Rosh Review family med

Réussis tes devoirs et examens dès maintenant avec Quizwiz!

A 55-year old woman presents with feeling "swollen all over." You do not notice any signs of edema, despite her claim that her rings no longer fit and that she has gone from a shoe size of 6 to a 7 1⁄2 over the last 2 years. A review of systems reveals that she is sweating more than usual, feels fatigued, and often has a dull headache and diffuse arthralgias. On examination she has no joint swelling, erythema, tenderness and no skin abnormalities are noted. This presentation is most consistent with which one of the following? A. Acromegaly Correct Answer B.Cushing's disease C.Polymyalgia rheumatica D.Scleroderma

Correct Answer ( A ) Explanation: Acromegaly usually has an insidious onset, with a time to diagnosis on the order of 6-10 years. The classic facial findings, which include enlargement of the supraorbital ridges, mandible and a widened nose, are sometimes difficult to identify without a reference to the patient's appearance from several years earlier. Patients with this condition sometimes present with nonspecific symptoms such as arthralgias, fatigue, and headache. More specific complaints such as visual field defects or increasing ring and shoe sizes should prompt a physician to consider this diagnosis. The condition is caused by a growth hormone-secreting pituitary adenoma in 98% of cases. In younger patients this disease results in gigantism. Blood testing for growth hormone and IGF-1 is usually adequate for initial testing. Because of the relatively high incidence of nonfunctioning, incidentally discovered pituitary adenomas, imaging studies should be obtained only after a firm biochemical diagnosis of acromegaly has been made. The sella turcica should be imaged first, since GH-secreting pituitary adenoma is the most common cause of acromegaly. Magnetic resonance imaging is the imaging study of choice. For pituitary adenomas, transsphenoidal surgery is usually considered the first line of treatment, followed by medical therapy for residual disease. Somatostatin and dopamine analogues and growth hormone receptor antagonists are the mainstays of medical treatment for growth hormone excess after surgery. Clinical findings in Cushing's disease (B) include weight gain, truncal obesity, striae, hypertension, glucose intolerance and infections. The pituitary tumors in Cushing's disease are usually microadenomas, which, by definition, are 10 mm or less in diameter. These tumors are most often discovered when clinical manifestations from hypersecretion of ACTH prompt an appropriate diagnostic work-up. The classic clinical presentation of scleroderma (D) is a young or middle-age woman with Raynaud phenomenon and skin changes accompanied by musculoskeletal discomfort and gastrointestinal symptoms. Polymyalgia rheumatica (C), the most common chronic inflammatory condition in older adults, primarily affects proximal muscles and joints, causing disability. Proximal myalgias and arthralgias typically develop over weeks to months, worsening at night and with movement. True muscle weakness, as opposed to limited effort because of pain or atrophy from disuse, is not associated with pure polymyalgia rheumatica.

Based on the National Cholesterol Education Program (NCEP) Adult Treatment Panel (ATP) III guidelines, what should be the goal LDL cholesterol for a high-risk patient with diabetes and coronary artery disease? A.LDL < 100 mg/dL B.LDL < 130 mg/dL C.LDL < 160 mg/dL D.LDL < 70 mg/dL

Correct Answer ( A ) Explanation: Based on the latest NCEP ATP III guidelines, the target LDL cholesterol in high-risk patients with a history of diabetes and coronary disease is < 100 mg/dL. Additionally, when a high-risk patient has low HDL-cholesterol or high triglyceride levels, the combination of niacin or a fibrate with an LDL-lowering drug should be considered. However, regardless of the drug regimen employed, patients with diabetes should maintain tight glycemic control, which in itself can help reverse the dyslipidemic profile that is prevalent in diabetics. As with all patients, lifestyle modification, including weight reduction and regular exercise, remains an important cornerstone of the treatment of dyslipidemia in patients with diabetes. The importance of dyslipidemia as a contributor to cardiovascular risk in patients with diabetes is reflected in the new guidelines of the NCEP ATP III. For the first time, diabetes is considered a coronary heart disease (CHD) risk equivalent, meaning that patients with diabetes have a risk of CHD that is similar to that of patients with clinically manifest CHD (> 20% risk of an event in the following 10 years). In addition, the presence of multiple CHD risk factors, the metabolic syndrome, and mixed hyperlipidemia (high triglyceride and low HDL-cholesterol levels) should be taken into account when estimating a patient's global risk. An LDL of < 160 (C) is never a goal of a diabetic patient. According to the NCEP ATP III guidelines, diabetic patients are candidates for cholesterol-lowering therapy if the LDL-cholesterol level is higher 130 (B). Reducing LDL cholesterol to less than 70 (D) is no longer the goal for a diabetic patient.

A 3-year-old girl presents with painful vesicles on her buccal mucosa and posterior pharynx and vesiculopapular lesions on the hands and feet. Which of the following is the most likely cause of her symptoms? A.Coxsackie A 16 B.Herpes simplex virus 1 C.Human herpes virus 6 D.Varicella zoster virus

Correct Answer ( A ) Explanation: Coxsackie A 16, an enterovirus, is the most frequent cause of Hand-Foot-and-Mouth disease (H-F-M). Hand, foot, mouth disease is characterized by several days of fever (often high and acute in onset), irritability, and decreased appetite. On physical exam, the oropharynx is inflamed with vesicles that can be present on the tongue, buccal mucosa, lips, palate, posterior pharynx and gingiva which can ulcerate leaving 4-5 mm shallow lesions with surrounding erythema. Lesions typically resolve in about one week. Its distinctive presentation also includes a vesiculopapular skin rash typically on the dorsum of the hands and feet but may also present on the palms and soles and buttocks and groin. The skin lesions are usually tender and approximately 3-7 mm in size. Complications are rare but include myocarditis and pericarditis. Treatment is supportive with cool fluids and pain relievers to prevent dehydration from poor oral intake due to pain. Herpes gingivostomatitis is an acute oropharyngeal infection and is the most common presentation of primary herpes simplex virus 1 (B) infection in children six months to five years. It is sudden in onset and is associated with high fever, mouth pain, drooling, and food and drink refusal. The gums become swollen and vesicles develop throughout the oral cavity and are usually more extensive than that of those with H-F-M and are limited to the anterior oral cavity unlike the posterior involvement of coxsackie. There is no skin rash associated with herpes simplex virus 1 infection. Human herpes virus 6 (C) is associated with roseola infantum (sixth disease). This self-limited disease occurs in infancy and early childhood. It begins with the abrupt onset of high fever usually lasting 72 hours. Upon defervescence, a 2-3 mm morbilliform rash develops on the trunk. Varicella zoster virus (D) is associated with both varicella (chicken pox) and zoster (shingles). Varicella is associated with skin lesions that typically present first on the scalp and face and spread downward to the trunk. They are pruritic and evolve from erythematous macules to papules to form a clear, fluid filled vesicle. This occurs over a 24-48 hour period then lesions begin to crust over as new vesicles are appearing. Zoster is associated with a vesicular rash in a dermatomal distribution.

An elderly man with congestive heart failure presents to the ED with a complaint of "not feeling normal." He is on several different medications. His ECG is shown above. Which of the following medications is the most likely cause of this ECG abnormality? A.Digoxin B.Furosemide C.Metoprolol D.Warfarin

Correct Answer ( A ) Explanation: Digoxin is a cardiac glycoside derived from the foxglove plant. It is used to increase the force of myocardial contraction in systolic heart failure and to decrease AV nodal conduction in atrial fibrillation. It works by inactivating the Na+ K+ ATPase pump on the cardiac cell membrane which leads to increased intracellular calcium and extracellular potassium. In individuals taking digoxin, there is a characteristic ECG pattern commonly referred to as the "digoxin effect." The presence of the "digoxin effect" on the ECG is not a marker of toxicity, rather it indicates that the patient is taking digoxin. The "digoxin effect" is characterized by (1) downsloping ST depression with a characteristic "slurred" appearance; (2) flattened, inverted, or biphasic T waves; (3) shortened QT interval. The overall morphology is commonly compared to the shape of a mustache. Furosemide (B) is a diuretic that is not directly related to changes in ECG morphology. However, in extreme cases it can lead to electrolyte abnormalities that are represented by ECG changes. Metoprolol (C) and other beta-blockers may cause a first-degree heart block (PR interval > 200 msec). Warfarin (D) does not affect the cardiac electrical pathway to produce ECG changes.

A 65-year-old man presents to your office with complaints of urinary frequency, hesitancy and nocturia. Which of the following findings would suggest a diagnosis of prostate cancer rather than benign prostatic hypertrophy? A.Asymmetric areas of induration on digital rectal exam B.Elevated serum prostate specific antigen C.Presence of hematuria on urinalysis D.Symmetric enlargement and firmness of the prostate on digital rectal exa

Correct Answer ( A ) Explanation: In the United States, prostate cancer is the most frequently diagnosed type of cancer in men after skin cancer. Prostate cancer is seen more commonly in African-American men and the likelihood of developing this type of cancer increases with age. It is uncommon in men younger than 50 years of age. Risk factors include a family history of prostate cancer, cigarette smoking and a diet high in animal fat. Most patients diagnosed with prostate cancer are asymptomatic and the cancer is found on digital rectal exam (DRE) or because of an elevated serum prostate specific antigen (PSA). A physical exam finding of asymmetric areas of induration or nodules on DRE is suspicious for prostate cancer and further workup should be initiated. Diagnosis is made with biopsy. Treatment decisions are determined after discussion with the patient about the severity of disease and quality of life related to treatment side effects. Treatment options include active surveillance, prostatectomy, radiation therapy and hormonal therapy. Elevated serum prostate specific antigen (B) and the presence of hematuria on urinalysis (C) may be seen in both prostate cancer and benign prostatic hyperplasia (BPH). Physical exam findings help to differentiate between the two conditions. The diagnostic approach to patients with suspected BPH includes complete history, physical exam, urinalysis, serum PSA and creatinine, and evaluation for urinary obstruction. Symmetric enlargement and firmness of the prostate on DRE (D) is a physical finding seen in patients with BPH.

An 88-year-old woman presents complaining of substernal chest burning and difficulty swallowing that started earlier today. She was taking her pills when one of them "got stuck in her throat." She has an extensive medical history and is on multiple medications. Which of the following medications is most likely responsible for her symptoms? A.Alendronate B.Lisinopril C.Metformin D.Simvastatin

Correct Answer ( A ) Explanation: Pill esophagitis occurs when a pill remains in contact with esophageal mucosa for a prolonged period, leading to irritation, ulceration, or stricture formation. It is associated with many medications, especially alendronate (a bisphosphonate used in the treatment of osteoporosis), tetracyclines, NSAIDs, potassium chloride, and ferrous sulfate. Treatment includes antacids to reduce esophageal inflammation. Patients should be advised to remain in an upright position while taking their medications and to drink at least four ounces of liquid with each pill. Lisinopril (B), metformin (C), and simvastatin (D), are not commonly implicated in pill esophagitis.

An 18-year-old woman presents after having a syncopal episode. She is complaining of a 2-day history of lower abdominal pain and vaginal spotting. Her BP is 86/42, HR is 128, RR is 18 breaths, and oxygen saturation is 99% on room air. She is drowsy, but answers questions appropriately. What is the most appropriate next step in management? A.Establish large-bore IV access and administer an IV fluid bolus B.Initiate rapid sequence induction and orotracheal intubation C.Perform a bedside urine pregnancy testing D.Perform an ultrasound of the abdomen to assess for free fluid

Correct Answer ( A ) Explanation: The patient is hypotensive and tachycardic. She is suffering from hypovolemic shock secondary to a ruptured ectopic pregnancy. Therefore she requires immediate intravenous access and volume resuscitation with Lactated Ringer's or normal saline. Emergency Department management of unstable patients includes rapid assessment of the ABC's (Airway, Breathing, Circulation). This patient is phonating, has a respiratory rate of 18 breaths per minute and an oxygen saturation of 99% on room air. There is no concern that her airway or breathing is in immediate jeopardy, therefore she would not require immediate rapid sequence induction and orotracheal intubation (B). Although a bedside pregnancy test (C) and abdominal ultrasound (D) would help make a diagnosis of ruptured ectopic pregnancy, the next step would be to resuscitate the patient.

A 41-year-old woman presents with fever and retrosternal chest pain of 28 hours duration. A loud friction rub is appreciated over the left sternal border. There is widespread ST-segment elevation seen on her ECG. Which of the following physical findings do you most expect to find in this patient? A.Chest pain lessens when she sits forward B.Chest pain worsens during forced expiration C.Normal breath sounds over the left chest field D.Tympanic percussion over the left chest field

Correct Answer ( A ) Explanation: The pericardium is a two layered sac (visceral and parietal pericardia) which surrounds the heart and great vessels. It normally contains 15-50 ml of straw-colored fluid. The pericardium can become inflamed (pericarditis), or become filled with fluid (effusion) or become constrictive (as an end result of inflammation). Acute pericarditis is most commonly caused by an infectious etiology (mainly viral, but possibly bacterial, tuberculous or fungal) or unknown source (idiopathic, 80% of which self-resolve within 1-3 days). It can also arise from neoplastic, autoimmune, radiation or uremic insult. In addition, pericarditis can be associated with acute transmural myocardial infarction or proximal aortic dissection. Pericarditis is marked by pleuritic chest pain that is less with sitting forward and worse with recumbency, effusion and tamponade, and friction rub, a rough sound heard best at the left lateral sternal border with the diaphragm. Viral, bacterial, tuberculous and fungal sources are possible. ECG, echo, chest radiograph, CBC, ESR and serum troponins are the preferred initial diagnostic tests. Treatment includes NSAIDs, corticosteroids and colchicine. Drainage and antibiotics are usually necessary if an infectious etiology is found. The pain of pericarditis is pleuritic; that is, it is worse with deep inspiration, not expiration (B). As effusion is common with pericarditis, an increase in pericardial sac volume can have a mass effect on surrounding tissue, namely the left posterior lung. This can lead to compressive atelectasis, decreased breath sounds (C) and dullness (D) to percussion (Ewart's sign).

Which of the following causes a painless change in vision? A.Acute angle closure glaucoma B.Central retinal artery occlusion C.Corneal abrasion D.Optic neuritis

Correct Answer ( B ) Explanation: Central retinal artery occlusion (CRAO) causes painless vision loss. CRAO occurs when a clot obstructs the ophthalmic artery, the first intracranial branch of the internal carotid artery, causing an ischemic stroke of the retina. It is most common in patients between the 5th and 7th decades of life. Hypertension, diabetes, vasculitis, connective tissue disorders, sickle cell disease and cardiac disease are the most common risk factors. Patients present with a complaint of a sudden and severe vision loss developing over seconds. Physical examination reveals decreased visual acuity and an afferent pupillary defect. The retina appears edematous and pale with comparative prominence of the fovea ("cherry-red spot") on fundoscopic examination. Treatment is directed at attempting to dislodge the embolus from the artery and restore blood flow. Digital global massage should immediately be started. Reducing intraocular pressure with timolol or acetazolamide may result in dislodgement of the clot by increasing the perfusion gradient. Emergent ophthalmologic consultation should also be obtained for surgical options. Acute angle closure glaucoma (A) and optic neuritis (D) are typically painful. Corneal abrasions (C) are painful and should not cause marked vision change.

A 25-year-old man presents to your office after having noticed a slightly raised red lesion with central clearing that increased in size over the past five days. He describes feeling flu-like symptoms and states that he had been camping in Connecticut recently. Based on the clinical presentation, which of the following is the best treatment option for this patient? A.Ceftriaxone 250 mg single dose intramuscular injection B.Doxycycline 100 mg oral tablet twice per day for 21 days C.Metronidazole 2 g single dose tablet orally D.Watch and wait

Correct Answer ( B ) Explanation: Doxycycline 100 mg oral tablet twice per day for twenty-one days is the correct treatment and dosing for the suspected diagnosis of Lyme disease with a known erythema migrans (bulls-eye) rash. Lyme disease is caused by the spirochete Borrielia burgdorferi. It is transmitted to humans through a tick bite. The disease is most common in the northeastern United States, with Connecticut having one of the highest prevalence rates. It is recommended to treat any patient with suspected Lyme disease with an associated erythema migrans rash. Ceftriaxone 250 mg single dose intramuscular injection (A) is the correct treatment for urethritis caused by Neisseria gonorrhoeae. Gonococcal urethritis typically presents with painful urethral discharge. Patient's with urethritis should be treated concurrently for Chlamydial infection since the two infections commonly occur together. Metronidazole 2 g single dose orally (C) is the treatment for a trichamonal infection. Trichamonas is a flagellated protozoan, which infects the genitourinary tact of males and the lower urinary tract of females. Associated clinical findings include a malodorous, frothy, yellow-green discharge and a wet-mount that reveals motile flagellates. Watch and wait (D) is not the treatment of choice. Though it is suggested to send blood work for a confirmatory laboratory titer, the proper treatment course is to treat any patients with symptoms suggestive of Lyme disease. Delayed treatment can result in central nervous system and cardiac complications such as Bells' palsy, encephalitis, pericarditis, and heart block.

Which of the following is the first line treatment for acute pericarditis? A.Citalopram B.Ibuprofen C.Ketorolac D.Prednisone

Correct Answer ( B ) Explanation: If there is a specific etiology of the pericarditis, then therapy should be directed at that etiology. Otherwise, nonsteroidal anti-inflammatory drugs (NSAIDs) are the mainstay of therapy for acute pericarditis. Ibuprofen has the best side effect profile, but other NSAIDs should be equally effective.The majority of patients have prompt resolution of symptoms without recurrent pericarditis when treated with NSAIDS alone. However, when used as an adjunct to NSAID therapy, colchicine reduces symptoms, decreases the rate of recurrent pericarditis, and is generally well tolerated. Citalopram (A) is a serotonin-reuptake inhibitor (SSRI) and has no role in the treatment of acute pericarditis. Ketorolac (C) is an NSAID with an excellent analgesic profile, but it has poor anti-inflammatory effects. Therefore, it is not recommended for the treatment of acute pericarditis. Prednisone (D) is a corticosteroid with anti-inflammatory properties that has use limited to the treatment of refractory pericarditis (patients who are intolerant or unresponsive to NSAIDS and/or colchicine). There is a risk of relapse of symptoms when the steroids are tapered or stopped.

Which of the following conditions is associated with low output heart failure? A.Anemia B.Dilated cardiomyopathy C.Pregnancy D.Thyrotoxicosis

Correct Answer ( B ) Explanation: Low output cardiac failure is more common than high output cardiac failure. Low output failure is typically associated with a dilated cardiomyopathy, which may occur as a result of poorly controlled chronic hypertension, ischemic heart disease, or valvular heart disease. Low output failure is characterized by a decreased cardiac output (systolic dysfunction), a decrease in left ventricular end-diastolic pressure, and an increased systemic oxygen extraction ratio. Anemia (A), pregnancy (C), and thyrotoxicosis (D) are associated with high output cardiac failure, which is caused by excessive demand for tissue perfusion resulting in hyperdynamic cardiac dysfunction with a supranormal cardiac output and decreased oxygen extraction ratio. Pulmonary congestion and peripheral edema occur as a result of elevated diastolic pressures. Over time, systolic myocardial dysfunction occurs, and patients develop classic heart failure indistinguishable from other end-stage cardiomyopathies.

A 12-year-old uncircumcised boy presents to the emergency department with a complaint of inability to retract his foreskin to its normal position. While he was showering, he retracted his foreskin to clean his penis. He called his mother for help when he could not get his foreskin back into place. Which of the following is the most likely diagnosis? A.Balanitis B.Paraphimosis C.Phimosis D.Priapism

Correct Answer ( B ) Explanation: Paraphimosis occurs in uncircumcised males when entrapment of the foreskin behind the glans penis causes venous and lymphatic congestion, resulting in inability to retract the foreskin to its normal position. Risk factors include genitourinary procedures, sexual activity, penile trauma, and males who forget to replace the foreskin after cleaning or urination. Paraphimosis typically causes significant pain and is a true urologic emergency. Treatment includes the timely reduction of the foreskin back over the glans penis. Urology consultation should be ordered for any signs of arterial compromise of the glans penis. Balanitis (A) is an inflammation of the glans penis often caused by poor hygiene. Phimosis (C) occurs when the foreskin cannot be retracted over the glans penis. Priapism (D) is a prolonged erection of the penis.

A 24-year-old woman presents to the clinic with mild otalgia and a chronic feeling that her right ear is "blocked." An otoscopic examination reveals a marginal tympanic membrane perforation. Keratin debris is prominent in this area. Which of the following risk factors likely contributed to the development of this patient's condition? A.Diabetes mellitus B.Prolonged eustachian tube dysfunction C.Recent upper respiratory infection D.Recent water exposure

Correct Answer ( B ) Explanation: Prolonged eustachian tube dysfunction is the most common cause of cholesteatoma formation, as the negative middle ear pressure draws the upper, flaccid portion of the tympanic membrane inward. The result is the formation of a squamous epithelium-filled sac which can fill with desquamated keratin and cause chronic infection. Cholesteatomas pose a significant danger to patients as they can eventually erode through the mastoid bone and destroy the ossicular chain. The inner ear and facial nerve may also become eroded, and intracranial involvement occasionally occurs. Because of the risk of devastating erosion, all patients with cholesteatomas should be referred to ENT for surgical marsupialization of the sac or its complete removal. Removal of the cholesteatoma may take precedence over preserving auditory function. Cholesteatomas should be suspected in patients with chronic mild otalgia, hearing loss, or chronic ear fullness, particularly with a long history of eustachian tube dysfunction. Diabetes mellitus (A) is not a risk factor for cholesteatoma formation. It is a risk factor for malignant otitis externa, which may cause infectious osteomyelitis to spread from the floor of the ear canal to other regions of the ear and skull. Recent upper respiratory infections (C) are unlikely to cause a cholesteatoma. They may, however, contribute to development of acute otitis media. Recent water exposure (D) does not contribute to cholesteatoma formation. It is more likely to cause acute otitis externa.

A 43-year-old woman presents to the ED with abdominal pain. She reports that the pain is epigastric and worse with food. She denies any fevers, chills, vomiting, or diarrhea. She recently visited a gastroenterologist who told her she had a bacterial infection in her stomach, however, she did not follow up or receive treatment. On exam, her abdomen is soft and nontender. Which of the following is the most appropriate treatment regimen for this patient's condition? A.Amoxicillin 1 g BID, metronidazole 500 mg BID, lansoprazole 30 mg BID for 10-14 days B.Clarithromycin 500 mg BID, amoxicillin 1 g BID, lansoprazole 30 mg BID for 10-14 days C.Clarithromycin 500 mg BID, amoxicillin 1 g BID, metronidazole 500 mg BID for 10-14 days D.Clarithromycin 500 mg BID, amoxicillin 1 g BID, metronidazole 500 mg BID, lansoprazole 30 mg BID for 10-14 days

Correct Answer ( B ) Explanation: This patient is presenting with peptic ulcer disease secondary to Helicobacter pylori infection. It is estimated that 30%-40% of the US population is infected with H. pylori, although many will never develop peptic ulcer disease. It is usually acquired through the fecal-oral route, typically in childhood. It is estimated that the majority of patients with duodenal and gastric ulcers are infected with H. pylori. Its presence causes mucosal inflammation and disrupts the normal defense mechanism of the stomach lining, leading to ulceration. Methods of testing include urea breath test, urea blood test, blood antibody testing, stool antigen testing, and biopsy during endoscopy. Triple therapy with clarithromycin 500 mg BID, amoxicillin 1 g BID (metronidazole 500 mg BID if allergic to penicillin), and a proton pump inhibitor (such as lansoprazole 30 mg BID) for 10-14 days is currently recommended for all patients with confirmed H. pylori infection. Amoxicillin 1 g BID, metronidazole 500 mg BID, lansoprazole 30 mg BID for 10-14 days (A) is incorrect as you need the addition of clarithromycin; clarithromycin 500 mg BID, amoxicillin 1 g BID, metronidazole 500 mg BID for 10-14 days (C) is incorrect as you have not used a PPI; and clarithromycin 500 mg BID, amoxicillin 1 g BID, metronidazole 500 mg BID, lansoprazole 30 mg BID for 10-14 days (D) is incorrect as you have treated the patient with too many medications.

A 43-year-old woman with a history of von Willebrand disease presents with epistaxis. Vital signs are normal and examination reveals oozing from the right nares despite applying pressure. Which treatment is indicated in this patient's management? A.Cryoprecipitate B.Desmopressin C.Factor VIII concentrate D.Fresh frozen plasma

Correct Answer ( B ) Explanation: This patient presenting with mild bleeding with a history of von Willebrand disease should have desmopressin (DDAVP) administered. von Willebrand disease is a common inherited bleeding disorder. von Willebrand factor (vWF) is involved in factor VIII activity in supporting platelet adhesion. Platelets have normal morphology but in the absence of factor VIII/vWF complex their ability to adhere is impaired. There are three types of von Willebrand disease: Type I - mild to moderate decrease in vWF, Type II - dysfunctional vWF, Type III - absolute lack of vWF. Patients predominantly present with mucosal surface bleeding including epistaxis and bleeding from the gums. Menorrhagia and gastrointestinal bleeding are common but hemarthrosis is rare. Diagnosis is classically made with an abnormal bleeding time. A reduction in vWF activity will also be seen. In patients with mild mucosal bleeding, desmopressin or DDAVP is sufficient for treatment. Desmopressin acts by increasing circulating levels of factor VIII and vWF. It can be administered intravenously or nasally. Cryoprecipitate (A) contains factor VIII, XIII, von Willebrand factor (vWF), fibrinogen and fibronectin and can be used in patients with severe bleeding. Fresh frozen plasma (D) should only be used in severe bleeding when factor VIII concentrate is not available. Factor VIII concentrate (C) is first line therapy in patients with vWF and severe bleeding if available.

A 3-year-old girl presents to the ED with her mom and grandmother for evaluation of vaginal spotting. The child has no other complaints and specifically denies sexual abuse or trauma when questioned alone. After watching a news special on sexual abuse, mom is concerned that her daughter may have been sexually assaulted, despite no specific concern. The child is acting appropriately and there are no external lesions or signs of trauma. Internal pelvic exam is difficult due to the patient's age. Which of the following statements is true? A.Obtain a pelvic x-ray prior to attempting a physical exam B.She is the victim of abuse; do no further evaluation without a trained nurse present and contacting authorities C.She may have a vaginal foreign body; consider a nasal speculum to attempt visualization and removal D.Vaginitis is an unusual diagnosis in this age group

Correct Answer ( C ) Explanation: Children often fear parental disapproval of a vaginal foreign body placement. This often leads to a delay in diagnosis until secondary signs (such as vaginal bleeding, foul smelling discharge, or purulent drainage) are noted by parents. The physical exam is often quite difficult owing to patient anxiety, small anatomic size, and parental concerns about sexual or physical abuse. However, a thorough vaginal exam is indicated in patients with vaginal bleeding or discharge. Use of a nasal speculum, procedural sedation, or trained assistant (such as a child life specialist) may facilitate this exam. Although plain films (A) may identify certain radiopaque objects, they are often of limited diagnostic value. A negative film does not disprove the presence of a radiolucent vaginal foreign body. Sexual and physical abuse (B) is always an important consideration when evaluating children with genitourinary or rectal complaints. When in doubt, ethical and legal requirements obligate the notification of appropriate child protection services. However, in this case, there is no evidence of abuse. Foreign body or vaginitis is a much more likely diagnosis. Vulvovaginitis (D) is a very common diagnosis in this age range owing to a lack of estrogen, developing toilet hygiene, and frequent use of baths, perfumed soaps, and bubbles.

Which of the following best characterizes diabetes insipidus? A.High urine osmolality, high serum osmolality B.High urine osmolality, low serum osmolality C.Low urine osmolality, high serum osmolality D.Low urine osmolality, low serum osmolality

Correct Answer ( C ) Explanation: Diabetes insipidus (DI) causes polyuria and an inability to concentrate urine because of a deficiency of antidiuretic hormone (vasopressin). ADH acts directly on the renal collecting system and distal tubules to reabsorb water from the urine back into the serum, hence the term "antidiuretic." There are several forms of DI, but the most common types are central and nephrogenic. Central (neurogenic) DI is due to a decrease in the release of ADH. Causes include neurosurgery, trauma, tumors, ischemia, infiltrative diseases, or idiopathic. Head trauma and brain injury can lead to DI. Nephrogenic DI is due to resistance to ADH at the level of the kidneys. DI is characterized by inappropriately dilute urine (low urine osmolality) in the setting of increased serum osmolality. Polyuria and polydipsia are also seen. The treatment for central DI is to treat the underlying lesion and administer intranasal DDAVP (desmopressin synthetic ADH). Nephrogenic DI is treated by treating the underlying condition. Thiazide diuretics and amiloride may help. The urine osmolality is low, not high (A and B) in DI. The serum osmolality is high, not low (B and D) in DI. High urine osmolality and low serum osmolality is consistent with syndrome of inappropriate ADH release (SIADH).

You are evaluating a patient with polyneuropathy. His nerve conduction study reveals a predominantly demyelinating injury process. Which of the following laboratory serum tests would best delineate this patient's diagnosis? A.Creatinine B.Hemoglobin A1C C.Hepatitis B panel D.Thyroid function studies

Correct Answer ( C ) Explanation: Electrodiagnostic testing is initially used in the evaluation of polyneuropathy, with the goal of defining a predominant injury process, either axonal nerve damage or demyelinating nerve damage. Specific lab tests should be reserved until after this determination, as a certain disease tends to cause a certain nerve injury pattern: Uremia/chronic renal impairment (A), diabetes mellitus (B), hyperthyroidism (D) are associated predominantly with axonal neuropathy.

You are seeing an elderly man with new onset of peripheral edema, head fullness and neck venous engorgement. Initial testing shows normal left heart function. You suspect cor pulmonale. Which of the following tests is the most accurate in confirming this diagnosis? A.Cardiac magnetic resonance imaging B.Electrocardiography C.Right heart catheterization D.Ultrafast ECG-gated computed tomography

Correct Answer ( C ) Explanation: Cor pulmonale is defined as an alteration in the structure and function of the right ventricle caused by a primary disorder of the respiratory system. Although the most common cause of right heart failure is left-sided heart disease, cor pulmonale is right heart dysfunction due to a lung, and not heart, problem. Just like systemic hypertension causes changes in left ventricular function, pulmonary hypertension causes changes in right ventricular function. This major underlying pathology is largely due to some kind of pulmonary vascular bed compromise, which can be primary pulmonary hypertension or thromboembolic disease, but more commonly anatomic compromise (COPD, interstitial lung disease and rheumatologic or connective tissue/collagen vascular disorders) or vasoconstrictive compromise (chronic hypoxic states and acidemia). When evaluating a patient with right heart failure, cor pulmonale is considered if pulmonary pathology is causative. However, if the etiologic evaluation is void of a pulmonary source, then the diagnosis of cor pulmonale cannot be made. In this situation, the clinician then tries to pinpoint a cardiac or blood disorder. Even when a pulmonary source is found to be a cause of right side heart failure, it is equally important to determine if there is such a coexisting non-pulmonary cause, such as increased blood viscosity, atrial and ventricular defects, congenital heart disease, cardiomyopathies and constrictive pericarditis. The general approach to evaluating a patient with suspected cor pulmonale begins with routine lab testing, chest radiography and electrocardiography (ECG). Further investigation of underlying pulmonary pathology is then accomplished via pulmonary function testing, ventilation/perfusion (V/Q) scanning and chest computed tomography. Right heart catheterization is the most accurate but invasive test to confirm the diagnosis of cor pulmonale. Cardiac magnetic resonance imaging (A) may provide valuable information about right ventricular size, shape, structure and function. This information, however, does not identify the lung as the site of pathology. ECG (B) changes of right ventricular hypertrophy and strain may suggest cor pulmonale. Again, these do not isolate the location of the underlying pathology. Ultrafast ECG-gated computed tomography (D) is used to study right ventricular function, mainly ejection fraction and wall mass. Its use is still experimental, but in the future, it may be used in following the progression of, not diagnosing the presence of, cor pulmonale.

A 7-year-old boy presents to your office for a well-child check. His mother has concerns regarding his car booster seat. Which of the following is the most accurate statement regarding car safety seats? AAll children should be restrained in the rear seats of vehicles for optimal protection until they have outgrown the highest weight or height allowed by the manufacturer of their belt-positioning booster seat, typically after 12 years of age. BAll children should be transitioned to belt-positioning booster seats as early as possible for optimal protection. CAll children should use a belt-positioning booster seat until the vehicle lap-and-shoulder seat belt fits properly, typically when they have reached 4 feet 9 inches in height and are between 8 and 12 years of age.Correct Answer DAll infants and toddlers should ride in a rear-facing car safety seat until they are 1 year of age or older.

Correct Answer ( C ) Explanation: Motor vehicle crashes continue to be a leading cause of death in children ages 4 and older. As such, the AAP released a policy statement with evidence-based guidelines for child restraint systems. These guidelines recommend that children should continue to use belt-positioning booster seats until the lap-and-shoulder seat belt fits properly; this generally occurs when the child is at least 4 feet 9 inches in height as well as at least 8 years of age. The middle seat is the safest seat. Children should not be sitting in the front seat until at least age 13, regardless of weight. The guidelines recommend that all children regardless of height or weight should be restrained in the rear seats (A) until age 13. There is greater safety advantage for children to remain in car safety seats for as long as possible before transitioning to booster seats (B) per the AAP policy statement. Children should ride in rear-facing car seats until age 2 years of age (D) or older, or children younger than 2 years who have outgrown the height or weight limit for their car safety seat.

What is the recommended treatment for polymyositis? A.Antibiotics B.NSAIDs C.Prednisone D.Radiation therapy

Correct Answer ( C ) Explanation: Polymyositis is an inflammatory myopathy that is classified by patient age at onset or by coexisting diseases, such as myositis associated with neoplasia or myositis associated with collagen vascular diseases (e.g., systemic scleroderma, systemic lupus erythematous). They have a bimodal distribution and are seen most often between age 10 to 15 and 45 to 60 years. Myositis is most common after age 50. The cause of inflammatory myopathies is unknown, but evidence suggests a genetic predisposition (associated with certain HLA markers) combined with an environmental insult, such as viruses, thereby initiating an autoimmune process. Patients usually experience progressive, symmetric, proximal muscle weakness with fatigue, malaise, and morning stiffness. Muscles often affected are those of the shoulder, neck, and pelvic girdle. Pulmonary (interstitial pneumonitis or fibrosis), cardiac (cardiomyopathy, congestive heart failure, arrhythmias), pharyngeal (dysphagia), and musculoskeletal (myalgias, arthralgias) symptoms might occur, although most patients do not experience synovitis. CK as well as aldolase, ALT, AST, and lactate dehydrogenase (LDH) levels might be elevated. ESR is elevated only half the time. Muscle biopsy can also be helpful in diagnosis. Prednisone, 1 mg/kg/day for up to several months, is the drug of choice; the earlier started in the disease process, the more effective it is. If prednisone is not sufficient, methotrexate, azathioprine, or another immunosuppressant is added. Antibiotics (A), NSAIDs (B) and radiation treatment (D) are not recommended in the primary treatment of polymyositis. One Step Further

A patient was treated for an upper respiratory tract infection 2 weeks ago. She now presents with one day of 7/10 facial pain with bending forward, as well as difficulty blowing her runny nose. Examination reveals frontal bone tenderness to percussion and nasal erythema and drainage. Her temperature is 100°F. Which of the following is the most appropriate management option for this patient? A.Antihistamines B.Computed tomography of the frontal sinuses C.Decongestants D.Otolaryngological referral

Correct Answer ( C ) Explanation: Rhinosinusitis is one of the most common reasons a patient seeks medical care. Inflammation of the sinus epithelium almost never occurs without inflammation of the nasal epithelium, as such, rhinosinusitis is a more correct term. Onset can be acute, subacute, chronic and recurrent acute. Acute cases can be further defined as viral or bacterial, however, most cases are due to associated viral upper respiratory infection (the common cold). Symptoms include an onset after an upper respiratory tract infection, facial pain or pressure, nasal obstruction, rhinorrhea (may be purulent) and maxillary toothache. Symptoms lasting less than 7-10 days points toward a viral etiology. If symptoms last longer than 7 days, consider a bacterial etiology. Treatment begins with analgesics, decongestants and saline nasal irrigation. If the presenting temperature is greater than 101°F, or symptoms are severe or worsening or last longer than 7 days, begin antibiotics like amoxicillin or trimethoprim-sulfamethoxazole. Computed tomography of the sinuses (B) is not considered routine evaluation. It is reserved in cases of recurrent acute or chronic presentation, and is used to define suspected anatomic abnormalities. Antihistamines (A) are not recommended for the treatment of acute rhinosinusitis. Referral to an otolaryngologist (D) is recommended if patients fail medication therapy or CT scanning reveals sinus abnormalities.

A 2-year-old boy presents from a rural Native American reservation with an episode of a 3 minute generalized seizure. On further questioning, he has had 3 days of diarrhea, tenesmus, and fever. He has no prior history of seizures. Which of the following is most likely causing his symptoms? A.Campylobacter B.Salmonella C.Shigella D.Yersinia

Correct Answer ( C ) Explanation: Shigella species are aerobic, gram-negative bacilli that are primarily transmitted person to person by the fecal-oral route or by ingestion of contaminated food or water. Children in daycare settings or crowded living conditions are most commonly affected. The onset of illness is several days after ingestion. Shigella infects the large intestine and causes watery or loose stools, fever, abdominal cramps or tenderness, tenesmus, and mucoid bloody stools. The characteristic bloody diarrhea appears after the fever subsides. Children under 2 years of age are more likely to develop high fever and seizures. Lab findings are notable for an increased number of bands on the CBC regardless of the actual white blood cell count. Due to high resistance to ampicillin and trimethoprim-sulfamethoxazole, azithromycin is the oral drug of choice for treatment in children. Rare complications include bacteremia, Reiter syndrome, hemolytic-uremic syndrome, toxic megacolon, intestinal perforation and toxic encephalopathy. Salmonella (B) is the most commonly implicated pathogen in food-borne gastroenteritis and usually causes a self-limiting disease but rarely can lead to complications including bacteremia, enterocolitis, meningitis, and osteomyelitis. Campylobacter (A) infections cause diarrhea, abdominal pain mimicking appendicitis or intussusception, malaise and fever but in neonates and young infants bloody diarrhea without fever may be the only manifestation. Yersinia (D) usually manifests as fever and diarrhea in young children and stool often contains leukocytes, blood and mucus. Seizures are not associated with Yersinia.

A 59-year-old man presents to the ED with 12 hours of emesis and abdominal pain. Vital signs are T 38.2°C, BP 110/79, and HR 109. On exam, you note a tender 2-x-2-cm bulge with erythema in the abdominal midline above the umbilicus. There is abdominal distension, and an occasional high-pitched bowel sound is heard. After placing an IV line and nasogastric tube, which of the following is the most appropriate course of management? AAdminister broad-spectrum antibiotics and attempt reduction BAdminister broad-spectrum antibiotics and obtain a plain radiograph CAdminister broad-spectrum antibiotics and prepare the patient for the OR DAdminister broad-spectrum antibiotics then obtain a CT scan of abdomen

Correct Answer ( C ) Explanation: The clinical scenario is consistent with a strangulated loop of bowel (emesis, abdominal pain, fever, erythema, abdominal distension, high-pitched bowel sounds) incarcerated in a ventral hernia. If the contents of a hernia can be returned to their natural cavity by manual reduction, the hernia is termed reducible; if they cannot, it is termed irreducible or incarcerated. Incarcerated hernias (third most common cause of SBO) are subject to inflammatory and edematous changes and are at risk for strangulation, which refers to vascular compromise of the incarcerated contents. When strangulation is not emergently relieved, necrosis and gangrene develop. The treatment for an incarcerated hernia that cannot be manually reduced is surgical fixation. If strangulation is suspected or shock is present, broad-spectrum antibiotics and fluid resuscitation are necessary, but manual reduction is contraindicated because the patient requires surgical management. The reintroduction of ischemic, necrotic bowel back into the peritoneal cavity can result in subsequent perforation and sepsis. If the hernia is incarcerated, but the patient does not yet show signs of strangulation, then an attempt at reduction should be made in the ED. Antibiotics should be administered, but the patient requires surgical intervention for the strangulated loop of bowel rather than manual reduction (A), radiograph (B), or CT scan (D).

A 13-year-old boy on chemotherapy for acute lymphoblastic leukemia presents with progressive lower back pain for 2 weeks. Per mom, he has had subjective fevers and a "bulge in his lower back" that is warm to touch. He is currently afebrile and has no focal neurologic deficits. An MRI is obtained as seen in the image above. Which of the following regarding this patient's condition is true? A.An appropriate antibiotic regimen is cefepime and metronidazole B.Direct extension of skin and soft tissue infections is the most common cause C.Erythrocyte sedimentation rate is a sensitive screening tool D.Most patients present with back pain, fever, and a focal neurologic deficit

Correct Answer ( C ) Explanation: The patient has a posterior epidural abscess with adjacent osteomyelitis and myositis. The most common cause of an epidural abscess is hematogenous spread of infection, not direct extension from skin or soft tissue infection. Major risk factors include diabetes, intravenous drug abuse, chronic renal failure, alcoholism, and immunosuppression. The most common organism involved is Staphylococcus aureus. Other organisms include streptococci, anaerobes, gram-negative bacilli, and Pseudomonas aeruginosa. Patients present with localized back pain with tenderness to percussion. Fevers are common (reported in up to 75% of patients). MRI is the diagnostic modality of choice, but erythrocyte sedimentation rate is a sensitive marker and may be used in conjunction with plain radiographs to screen for infectious spinal disease. Vancomycin is an important component of the antibiotic regimen because it covers methicillin-resistant Staphylococcus aureus (MRSA) and is usually combined with a 3rd-generation cephalosporin (A). Cefepime is a 4th-generation cephalosporin with pseudomonas coverage. Metronidazole or clindamycin can be added for anaerobic coverage. While the classic triad consists of back pain, fever, and neurologic deficits (D), only a small proportion of patients actually have all three components at presentation. The most common cause of an epidural abscess is hematogenous spread of infection, not direct extension from skin or soft tissue infection (B).

A 64-year-old man presents with a cough and shortness of breath for 2 weeks. His chest X-ray is shown above. Which of the following tests would indicate an infectious process as the cause of the above finding? A.Pleural fluid to serum LDH ratio < 0.6 B.Pleural fluid to serum protein ratio < 0.1 C.Pleural fluid to serum protein ratio > 0.5 D.Pleural fluid WBC 5,000 cells/mm3

Correct Answer ( C ) Explanation: The patient presents with a large pleural effusion that may be secondary to a pneumonia and a pleural fluid:serum protein ratio > 0.5 indicates the presence of an exudative effusion consistent with an infectious cause. Pleural effusions can result from a number of different pathologies. This includes CHF, malignancy, bacterial pneumonia, pulmonary embolism and pancreatitis. In developing countries, tuberculosis infection is the most common cause of pleural effusions. Pleural effusions can either be transudative (CHF, cirrhosis, pulmonary embolism) or exudative (malignancy, pneumonia, pulmonary embolism, pancreatitis). Patients will often present with dyspnea and pleuritic chest pain and dullness to percussion over the effusion. It is often difficult clinically to determine the cause of the effusion but testing of a sample of the fluid can differentiate exudative from transudative processes. Light's criteria (see table) is typically used for this purpose. If a single criterion is consistent with an exudative process, the fluid is exudative in origin. A pleural fluid:serum LDH < 0.6 (A) and a pleural fluid:serum protein < 0.1 (B) are both consistent with transudative processes. A pleural fluid white blood cell count greater than 10,000 cells/mm3 is indicative of an exudative effusion not a pleural fluid WBC 5,000 cells/mm3 (D).

A 25-year-old man with a history of hereditary angioedema presents with swelling of the tongue and lips for 30 minutes. The patient appears to have difficulty breathing if he lies flat but is comfortable when sitting up. Visualization of the mouth and posterior pharynx is completely obstructed by the tongue. What management should be initiated? A.Diphenhydramine B.Epinephrine 1:1000 intramuscular C.Fresh frozen plasma infusion D.Methylprednisolone

Correct Answer ( C ) Explanation: This patient presents with hereditary angioedema. Angioedema reactions, in general, are similar pathophysiologically to urticaria but involve deeper dermal and subcutaneous tissues. However, when angioedema presents in the absence of hives, it suggests involvement of bradykinin as opposed to mast cell degranulation. Hereditary angioedema (HAE) is due to a deficiency of C1 esterase inhibitor. In cases of severe angioedema, fresh frozen plasma (FFP) has been used with successful results. FFP replaces C1 esterase inhibitor and contains kininase II, which breaks down bradykinin. Epinephrine (B), diphenhydramine (A) and methylprednisolone (D) are effective in the treatment of allergic reactions and anaphylaxis but not in the treatment of angioedema. Epinephrine's alpha-agonist effects increase peripheral vascular resistance, decrease vascular permeability and reduce systemic hypotension. Beta-agonist effects lead to bronchodilation and increased cardiac inotropy and chronotropy. Diphenhydramine is a histamine 1 receptor blocker and blocks the peripheral effects of histamine. Steroids, like methylprednisolone, stabilize mast cells leading to decreased release of inflammatory mediators involved in allergic reactions.

Which of the following organisms is associated with pneumonia and bullous myringitis? A.Bordetella pertussis B.Haemophilus influenzae C.Streptococcus bovis D.Streptococcus pneumoniae

Correct Answer ( D ) Explanation: Although bullous myringitis is described as a classic finding in M. pneumoniae infections, it is not specific for mycoplasmal infection and is present in only a few cases. S. pneumoniae infection is most likely the cause of bullous myringitis in the setting of pneumonia. S. pneumoniae is a common cause of community acquired pneumonia. Bullous myringitis is an inflammation of the eardrum in which painful, fluid filled vesicles form. Patients with S. pneumoniae will present with typical signs of pneumonia. The presence of otalgia should prompt evaluation for bullous myringitis. Bullous myringitis will resolve with antibiotics directed at the S. pneumoniae pneumonia. B. pertussis (A) is the causative organism for pertussis or whooping cough. H. influenzae (B) is common causes of pneumonia but do not cause bullous myringitis. Streptococcus bovis (C) is associated with endocarditis and colorectal cancer.

A 38-year-old man who recently emigrated from Southeast Asia was treated several weeks ago at a local hospital for pulmonary tuberculosis. He was started on therapy including rifampin, isoniazid, pyrazinamide, and ethambutol. He now reports numbness in all of his extremities which began one week ago and has been progressing. On physical exam he has an uncoordinated gait and paresthesias on his hands and feet. What is the most likely cause of his symptoms? A.Vitamin A deficiency B.Vitamin B12 deficiency C.Vitamin B3 deficiency D.Vitamin B6 deficiency

Correct Answer ( D ) Explanation: The clinical presentation is consistent with a peripheral neuropathy, likely due to a vitamin B6 (pyridoxime) deficiency associated with isoniazid (INH) treatment. Peripheral neuropathy is a known complication of treatment with isoniazid, as is hepatitis. Concomitant administration of pyridoxine (vitamin B6) has been shown to prevent and decrease the symptoms associated with it including numbness, tingling, ataxia, and CNS difficulties. For prevention and treatment of isoniazid-induced peripheral neuropathy, pyridoxine is employed. The symptoms of vitamin A (beta-carotene) (A) deficiency include night blindness, xerophthalmia, and faulty bone formation. Vitamin B3 (niacin) (C) deficiency most notably causes pellagra: dermatitis, diarrhea, dementia, and eventually death. Vitamin B12 (cobalamin) (B) deficiency most commonly causes a megaloblastic anemia with neurologic symptoms.

A 47-year-old woman with a history of human immunodeficiency virus infection, hypertension, and hyperlipidemia, presents with a 1-day history of epigastric abdominal pain, nausea and vomiting. Vital signs are notable for BP 135/86 mm Hg, pulse of 104 beats/minute, RR 14 breaths/minute, temperature of 37.9℃, and oxygen saturation of 98% on room air. On examination, she is exquisitely tender in the epigastrium. Lipase is elevated at 4,500 U/L. Which of the following of her medications is the most likely cause of her symptoms? A.Atorvastatin B.Indinavir C.Lisinopril D.Tenofovir

Correct Answer ( D ) Explanation: The patient has clinical and laboratory evidence of acute pancreatitis. Common causes of pancreatitis include alcohol, gallstones, and medications. In particular, the nucleoside reverse transcriptase inhibitors used in the treatment of HIV are commonly associated with pancreatitis. Common culprits include tenofovir, didanosine, and stavudine. Treatment of medication-induced pancreatitis includes supportive care, bowel rest, and cessation of the offending agent. Atorvastatin (A) is an HMG-CoA reductase inhibitor, commonly known as a statin, used in the treatment of hyperlipidemia. Common adverse effects of statins include myopathy and rhabdomyolysis. Indinavir (B) is a protease inhibitor used in the treatment of HIV. Nephrolithiasis is a well-recognized complication of indinavir use. Lisinopril (C) is an antihypertensive medication in the ace-inhibitors class. Important adverse effects of ace-inhibitors include cough and angioedema.

An 18-year-old male undergoes an electrocardiogram as part of a sports physical for his college soccer team. The electrocardiogram was read as abnormal, and he was sent to the ED for evaluation. His electrocardiogram is shown above. Which of the following complications is known to result in patients with the suspected diagnosis? A.Atrial fibrillation B.Eisenmenger's syndrome C.Type 2 second-degree heart block D.Ventricular fibrillation

Correct Answer ( D ) Explanation: The patient's ECG is suggestive of Brugada syndrome. Brugada syndrome is a hereditary condition that leads to abnormal myocardial depolarization and predisposes otherwise young healthy individuals to ventricular fibrillation and sudden cardiac death. Several variants of Brugada syndrome exist, which involve mutations in the genes responsible for transmembrane sodium, calcium, and potassium ion channels in the heart. The incidence of Brugada syndrome is highest in Asian populations and is lower in groups originating from Western Europe. Most patients with Brugada syndrome are asymptomatic, and the disorder is identified incidentally via an abnormal ECG. Palpitations, near-syncope, and syncope may be reported. There may also be a family history of sudden cardiac death. Classic ECG findings of Type 1 Brugada syndrome include coved ST segment elevation followed by an inverted T wave in leads V1 to V3. If Brugada syndrome is suspected based on ECG findings or clinical history, patients should be referred to cardiology for further testing to rule out structural heart problems and confirm the diagnosis of Brugada syndrome. Medications with sodium channel blockade, such as flecainide and procainamide, should be avoided. The definitive treatment for Brugada syndrome is placement of an implantable cardioverter defibrillator device. Atrial fibrillation (A) is not associated with Brugada syndrome. Eisenmenger's syndrome (B) occurs when a left-to-right cardiac shunt, such as from a ventricular septal defect, switches direction to become a right-to-left cardiac shunt. Type 2 second-degree heart block (C) is not associated with Brugada syndrome.

A 23-year-old man with a history of opiate abuse presents with tachycardia, hypertension and mydriasis. Which of the following is true regarding management? A.Antiemetics should be witheld as they may cause serious side effects B.Consider ICU admission for the potential for symptoms to worsen C.Naloxone should be given D.Symptom control may be achieved with clonidine

Correct Answer ( D ) Explanation: This patient presents with opiate withdrawal and management should focus on the relief of symptoms. The onset of withdrawal symptoms depends on the opiate agent used. Heroin withdrawal typically occurs within 4-6 hours of discontinuation whereas methadone has a longer half-life and withdrawal may be delayed 24-48 hours. Withdrawal leads to sympathetic discharge and adrenergic hyperactivity. Symptoms include CNS excitation, tachypnea and mydriasis. Tachycardia and hypertension are common. Additionally, patients will often complain of nausea, vomiting and diarrhea. Physical examination may also reveal piloerection, yawning, rhinorrhea and lacrimation. Opiate withdrawal is not life-threatening. Supportive and symptomatic care can decrease the patient's discomfort. Clonidine has been used to mitigate symptoms as have antiemetics. Antiemetics (A) can safely be used to control nausea and vomiting associated with opiate withdrawal. ICU admission (B) is unnecessary as opiate withdrawal is not life-threatening. Naloxone (C) is used to reverse the effects of opiates.

A 9-year-old girl presents with scalp itching. Physical examination reveals the finding seen in the image above. What treatment is indicated? A.Clotrimazole B.Ivermectin PO C.Lindane D.Permethrin

Correct Answer ( D ) Explanation: This patient presents with pediculosis capitis or head lice and should be treated with permethrin. Pediculosis capitis is caused by lice infection. These parasitic lice infest and lay eggs at the base of the hair shaft. Transmission is from person-to-person. It is common in children but uncommon after puberty. Patients will present with intense pruritus, which coincides with the lice feeding and inspection reveals nits (immature lice) firmly attached to the base of hair shafts. The heaviest infection is typically seen behind the ears. Diagnosis is made on clinical grounds or with microscopy. The preferred treatment for pediculosis capitis is with permethrin, which kills the adult louse. Subsequently, the nits must be removed with a vinegar solution and fine-tooth comb. The scalp should be reexamined one week later and repeat therapy at this point may be required. Clotrimazole (A) is an antifungal medication that does not play a role in the treatment of pediculosis. Ivermectin PO (B) is a second-line therapy for pediculosis capitis and should be reserved for cases in which topical therapy fails. Ivermectin is given as a single dose and repeated 10 days later but it is a second line treatment. There is also a topical ivermectin hat can be used first line. In the past, lindane (C) was indicated as first line treatment but now is reserved for treatment failures.

An 81-year-old man with 10 years of coronary artery disease presents with chest pain and altered mental status. His ECG shows consecutive, large and wide QRS complexes. P waves cannot be appreciated. His pulse is 188 bpm. Which of the following is the most likely diagnosis? A.Atrial fibrillation B.Atrial flutter C.Ventricular fibrillation D.Ventricular tachycardia

Correct Answer ( D ) Explanation: Ventricular tachycardia is described electrocardiographically as "continuous PVCs", or more specifically, consecutive, fast, large and wide QRS complexes. A wide QRS complex clues one in that the underlying electrical problem is in the ventricles. Ventricular tachycardia can be further defined as monomorphic (QRSs are the same form) versus polymorphic (differing appearance of the QRSs), or sustained (>30 seconds) versus non-sustained (<30 seconds, self-terminates). In this tachydysrhythmia, the ventricles depolarize and contract so fast that cardiac output decreases, especially in the elderly or those with diseased myocardium or coronaries. This perpetuates ventricular ischemia leading to a precipitous decline if the dysrhythmia is not treated. Atrial fibrillation (A) and atrial flutter (B) both have narrow QRS complexes, either irregular or regular respectively. Ventricular fibrilation (C) is identified by a totally erratic appearance of unidentifiable waves. Discrete QRS complexes are absent.

Which of the following best describes a congenital cholesteatoma? A. Benign tumor of the ear canal B. Fat (lipid) deposits deep in the edge of the cornea C. Localized deposits of fat that collects in the skin of the upper and lower eyelids D. Plaques that begin in coronary artery walls

Correct Answer ( A ) Explanation: Congenital cholesteatoma usually appears as a white, round, cystlike structure medial to an intact TM. Cysts are seen most commonly in the anterior-superior portion of the middle ear, although they can present in other locations and within the TM or in the skin of the ear canal. Affected children often have no prior history of otitis media (OM). Congenital or acquired cholesteatoma should be suspected when deep retraction pockets, keratin debris, chronic drainage, aural granulation tissue, or a mass behind or involving the TM is present. Besides acting as a benign tumor causing local bone destruction, the keratinaceous debris of a cholesteatoma is a good culture medium and may become a focus of infection for chronic OM. Complications include ossicular erosion with hearing loss, bone erosion into the inner ear with dizziness, or exposure of the dura, with consequent meningitis or a brain abscess. Cholesteatoma should be removed surgically after CT scan and hearing evaluation, and appropriate antibiotic therapy. A second-look procedure 6-9 months after primary surgery is often recommended to prevent further recurrence. Congenital cholesteatoma is an aggressive disease and needs to be taken care of to prevent permanent damage to the middle and inner ear. None of the following describe a cholestetoma and all refer to other conditions commonly associated with hyperlipidemia. Corneal arcus (B) is a greyish-white ring (or part of a ring) opacity occurring in the periphery of the cornea, in middle and old age. It is due to a lipid infiltration of the corneal stroma, with age the condition progresses to form a complete ring. That ring is separated from the limbus by a zone of clear cornea. Xanthelasma are localized deposits of fats that collect in the skin of the upper and lower eyelids (C). They can be skin-colored to yellowish and are associated about half the time with elevated blood lipids (fats), including cholesterol and triglycerides, which may be a sign of diabetes. Removal usually requires electric cautery or surgery. Cholesterol plaques are the culprits of heart disease. Plaques begin in artery walls (D) and grow over years. The growth of cholesterol plaques slowly blocks blood flow in the arteries. Worse, a cholesterol plaque can suddenly rupture. The sudden blood clot that forms over the rupture then causes a heart attack or stroke.

In which of the following spondyloarthropathies is uveitis the most common extra-articular manifestation? A.Ankylosing spondylitis B.Fibromyalgia C.Psoriatic arthritis D.Reactive arthritis

Correct Answer ( A ) Explanation: Seronegative spondyloarthropathies share the characteristics of sacroiliac involvement, peripheral inflammatory arthropathy, an absence of rheumatoid factor, ligamentous and tendinous changes, and a genetic component related to the HLA-B27 marker. Patients with ankylosing spondylitis typically have back discomfort, with radiographic evidence of sacroiliitis. Uveitis is the most common extra-articular manifestation. Aortic root disease may also occur. Fibromyalgia (B) is characterized by idiopathic widespread pain with no associated extra-articular manifestations. Psoriatic arthritis (C) occurs in up to 20% of patients with psoriasis. Other than psoriasis, patients exhibit asymmetrical oligoarthropathy or symmetrical polyarthropathy, spondylitis, and arthritis mutilans. Reactive arthritis (Reiter's) (D) most commonly occurs in young men after an episode of urethritis or dysentery. It is characterized by asymmetrical polyarthritis, conjunctivitis that can progress to uveitis, and painless oral mucosal lesions that develop into painful ulcers

Which of the following patients should receive prophylactic antibiotics to prevent endocarditis? A.18-year-old pregnant woman with a history of a repaired congenital heart defect, with an impending vaginal delivery B.19-year-old woman with a history of endocarditis who is undergoing a dental extraction C.20-year-old man with a prosthetic heart valve who requires a Foley catheter due to urinary obstruction D.21-year-old man with a history of a heart transplant and valvulopathy who is undergoing suture repair of a facial laceration

Correct Answer ( B ) Explanation: This patient meets high-risk criteria (history of endocarditis) and requires antibiotic prophylaxis. The American Heart Association has published guidelines regarding prophylaxis for infective endocarditis in high-risk patients undergoing dental or invasive respiratory procedures. High-risk patients who are undergoing vaginal delivery (A), Foley catheterization (C), and suture repair (D) do not require antibiotic prophylaxis. In fact, prophylactic antibiotics are generally not needed with any genitourinary or gastrointestinal instrumentation and nearly all procedures that are performed in the ED.

Which of the following is the most common complication of this injury? A.Avascular necrosis B.Infection C.Malunion D.Osteoarthritis

Correct Answer ( A ) Explanation: The radiograph demonstrates a fracture at the waist or middle third of the scaphoid. The blood supply to the scaphoid penetrates the cortex on the dorsal surface near the tubercle waist area (distal aspect of scaphoid). Therefore, there is no direct blood supply to the proximal portion of the bone. Because of this tenuous blood supply, scaphoid fractures have a tendency to develop avascular necrosis (AVN). The more proximal the fracture, the greater the likelihood of developing AVN. Infection (B) is an uncommon complication of an isolated scaphoid fracture. Malunion (C) is a complication due to improper healing of the fracture. It mainly occurs when a scaphoid fracture goes unrecognized or there is early discontinuation of immobilization. This can lead to arthritis (D) over time due to misalignment from the abnormal motion and collapse of the bone fragments. If caught before arthritis has developed, surgery may be performed to try to improve scaphoid healing.

A 5-year-old boy presents with fever and rash for 2 days. Examination reveals a well appearing child with marked erythema to both cheeks as seen above. What management is indicated? A.Acetaminophen for feverCorrect Answer B.Cephalexin C.Mupirocin D.Topical hydrocortisone

Correct Answer ( A ) Explanation: This patient presents with symptoms consistent with erythema infectiosum and requires supportive care. Erythema infectiousum, or fifth disease is caused by infection with parvovirus B19. The disease is characterized by rash and mild systemic symptoms. The classic rash is deeply red on the face giving a "slapped-cheek" appearance with circumoral pallor. Additionally, a maculopapular, lacelike rash may be seen on the arms and progresses caudally. Rarely, parvovirus B19 has been associated with hepatitis. There is no specific treatment for erythema infectiosum and so management should focus on supportive care and parental reassurance. Acetaminophen or ibuprofen can be used to treat the fever. Cephalexin (B) is a first generation cephalosporin with activity against many streptococcus and staphylococcus species, which are not implicated in erythema infectiousum. Mupirocin (C) is used in the treatment of impetigo. Topical hydrocortisone (D) is useful in the treatment of allergic reactions but not in parvovirus B19 infections.

A 53-year-old man with a history of atrial fibrillation and hypertension presents with severe abdominal pain. He states that the pain has been there for 3 days but got more severe today. Over the last 3 days, he has been unable to eat because the pain occurs after eating. Vitals signs are T 99.7F, HR 123, BP 101/66, RR 24. Examination reveals an uncomfortable patient with diffuse mild abdominal tenderness to palpation without rebound or guarding. Stool guaiac is positive and the serum lactate is 4.8 mg/dL. A surgical consultation is requested. Which of the following represents the appropriate management? A.Obtain abdominal X-rays B.Obtain CT angiogram of the abdomen and pelvis C.Obtain CT scan of the abdomen and pelvis without IV contrast D.Obtain right upper quadrant abdominal ultrasound

Correct Answer ( B ) Explanation: This patient's presentation is highly suggestive of mesenteric ischemia, which is best diagnosed by CT angiogram. Once ischemia has progressed to infarction, mortality climbs to 70% underscoring the need for rapid diagnosis and management. Mesenteric arterial embolism is the most common cause of mesenteric ischemia. Risk factors for arterial embolus include coronary artery disease, valvular heart disease and arrhythmias - particularly atrial fibrillation. The classic presentation of mesenteric ischemia is that of an elderly patient who presents with intermittent abdominal pain that is increased with eating. The pain is out of proportion to examination (meaning that the patient complains of severe pain but the exam reveals mild to moderate tenderness and may lack peritoneal signs). Additionally, patients will often have guaiac positive stool testing especially as the bowel becomes more ischemic. An elevated lactate level is also highly suggestive of mesenteric ischemia and has a high sensitivity. Because of the high morbidity and mortality associated with this disease, early surgical consultation and definitive imaging is central to management. The study of choice is either conventional angiography or CT angiography (more easily accessible). Management focuses on aggressive resuscitation, supportive care and early diagnosis. Abdominal radiographs (A) are useful in eliminating other possible diagnoses including small bowel obstruction and bowel perforation but they are not sensitive in diagnosing either of these diseases or mesenteric ischemia. Angiography or CT angiogram should not be delayed while obtaining plain radiographs. Traditional CT scan of the abdomen and pelvis either with or without contrast (C) may show signs of ischemia like bowel wall edema or intramural gas but will not show where the vascular obstruction lies and is not sensitive for detecting mesenteric ischemia. A right upper quadrant ultrasound (D) is the diagnostic modality of choice for biliary disease (cholelithiasis, cholecystitis etc.) but has not been shown to be useful in diagnosing mesenteric ischemia.

A 48-year-old man presents with bilateral swollen lower extremities. Which of the following may lead to a false-negative result for proteinuria on a urine dipstick? A.Alkaline urine B.Dilute urine C.Hematuria D.Prolonged dipstick immersion in urine

Correct Answer ( B ) Explanation: Urine dipstick tests are often performed to evaluate for the presence of proteinuria as a surrogate for impaired renal function. This occurs through a color change of tetrabromophenol blue. There is an approximate relationship between the protein concentration and color intensity; however, reliably positive results occur only at concentrations above 30 mg/dL. As such, dilute urine can generate false-negative results for proteinuria. In contrast, alkaline urine (A), hematuria (C), and prolonged dipstick immersion in urine (D) can all generate false-positive results.

Which of the following tests can be used to confirm lactose intolerance? A.Anti-Gliadin antibodies B.Breath hydrogen test C.Sweat test D.Urea breath test

Correct Answer ( B ) Explanation: Lactose breath hydrogen test will show a rise in breath hydrogen >20 ppm within 90 min of ingestion of 50 g of lactose and is positive for lactase deficiency. This test is positive in 90% of patients with lactose malabsorption. Common causes of false-negative results are recent use of oral antibiotics or recent high colonic enema. The diagnosis can usually be made on the basis of the history and improvement with dietary manipulation. Lactose intolerance is a prototypical carbohydrate malabsorption disorder. Individuals with the condition may experience diarrhea, cramps, abdominal pain, and flatus following ingestion of milk products. Symptom severity depends on the amount of lactose ingested and the fat content of the product (ie skim milk empties from the stomach faster, causing more symptoms). Diagnostic workup may include confirming the diagnosis with hydrogen breath test and excluding other conditions listed in the differential diagnosis that may also coexist with lactase deficiency. Imaging studies are generally not indicated. A small bowel series may be useful in patients with significant malabsorption. Nearly 50 million people in the United States have partial or complete lactose intolerance. There are racial differences, with <25% of white adults being lactose intolerant but >85% of Asian Americans and >60% of African Americans having some form of lactose intolerance. Anti-Gliadin antibodies (A) are used in the diagnosis of celiac disease and not associated with lactose intolerance. Sweat test (C) is associated with the diagnosis of cystic fibrosis and urea breath test (D) is used to confirm eradication of H. Pylori infection.

A 12-year-old girl presents to the clinic with one day of cough, fever to 102°F, and extreme fatigue. She is awake and alert but appears tired. Her oxygen saturation is 95 percent, and respiratory rate is 15 breaths per minute. Lung auscultation reveals fine crackles in the left upper lobe. Which of the following is treatment of choice? A.Ampicillin B.Azithromycin C.Cefotaxime D.Clindamycin

Correct Answer ( B ) Explanation: The above patient's constellation of fever, cough, and focal lung findings are concerning for community-acquired pneumonia. The onset of symptoms was acute and severe, and the lung findings are focal, which is suggestive of a typical bacterial etiology. S. pneumoniae is the most frequent cause of "typical" bacterial pneumonia in children of all ages. However, in otherwise healthy children five years and older with CAP who are not ill enough to require hospitalization, M. pneumoniae and C. pneumoniae are the most likely pathogens. A macrolide is the empirical treatment of choice for typical bacterial pneumonia in otherwise healthy children > 5 years old being treated in outpatient setting. Among the macrolide antibiotics, clarithromycin and azithromycin have a more convenient dosing schedule and fewer side effects than erythromycin, but erythromycin is less expensive. Macrolide antibiotics may provide coverage for S. pneumoniae, which is the most frequent typical bacterial pathogen for all age groups. However, approximately 40 to 50 percent of S. pneumoniae isolates are resistant to macrolides. Failure to respond to macrolide therapy may indicate the development of a complication, a macrolide-resistant pathogen, or the need to alter therapy to provide better pneumococcal coverage. Given the significant resistance of S. pneumoniae to macrolides, fluoroquinolones (e.g., levofloxacin, moxifloxacin) are another reasonable alternative for the outpatient treatment of CAP. In addition to their excellent Gram-negative spectrum, the fluoroquinolones are active against a number of the pathogens responsible for CAP, including beta-lactam-susceptible and non-susceptible S. pneumoniae, M. pneumoniae, and C. pneumoniae. However, S. pneumoniae resistant to levofloxacin has been identified. Ampicillin (B) is used in neonatal infections. Cefotaxime (C) is an intravenous, third-generation cephalosporin used for inpatient treatment of community-acquired pneumonia for children with non-type-1 hypersensitivity reactions to penicillin. As the child is not hypoxemic, in respiratory distress, or toxic-appearing, she does not require inpatient treatment of her pneumonia at this time. A third-generation cephalosporin should also be used in the setting of confirmed beta-lactamase positive bacteria, such as Haemophilus influenzae or Moraxella catarrhalis or in the setting of complicated pneumonia without an identified pathogen. Clindamycin (D) may be used for treatment of typical bacterial pneumonias for children with type-1-hypersensitivity reactions to penicillin or for children with suspected aspiration pneumonia. Aspiration pneumonia typically involves the right middle or lower lobes and is more common in children with swallow dysfunction, such as those with static encephalopathy.

A 3-week-old male is brought to your office because of a sudden onset of bilious vomiting of several hours duration. He is irritable and refuses to breastfeed, but stools have been normal. He was delivered at term after a normal pregnancy, and has no health problems to date. A physical examination shows a fussy child with a distended abdomen. Radiography of the abdomen shows a "double bubble" sign. Which one of the following is the most likely diagnosis? A. Infantile colic B. Intussusception C. Midgut volvulus D. Necrotizing enterocolitis

Correct Answer ( C ) Explanation: Midgut volvulus may present in one of three ways: as a sudden onset of bilious vomiting and abdominal pain in a neonate; as a history of "feeding problems" with bilious vomiting that appears to be a bowel obstruction; or less commonly, as failure to thrive with severe feeding intolerance. The classic finding on abdominal plain films is the "double bubble" sign, which shows a paucity of gas (airless abdomen) with two air bubbles, one in the stomach and one in the duodenum. However, the plain film can be entirely normal. The upper gastrointestinal contrast study is considered the gold standard for diagnosing volvulus. Infantile colic (A) usually begins during the second week of life and typically occurs in the evening. It is characterized by screaming episodes and a distended or tight abdomen. Its etiology has yet to be determined. There are no abnormalities on physical examination and ancillary studies, and symptoms usually resolve spontaneously around 12 weeks of age. Necrotizing enterocolitis (D) is typically seen in the distressed neonate in the intensive-care nursery, but it may occasionally be seen in the healthy neonate within the first 2 weeks of life. The child will appear ill, with symptoms including irritability, poor feeding, a distended abdomen, and bloody stools. Abdominal plain films will show pneumatosis intestinalis, caused by gas in the intestinal wall, which is diagnostic of the condition. Intussusception (B) is seen most frequently between the ages of 3 months and 5 years, with 60% of cases occurring in the first year and a peak incidence at 6-11 months of age. The disorder occurs predominantly in males. The classic triad of intermittent colicky abdominal pain, vomiting, and bloody, mucous stools is encountered in only 20%-40% of cases. At least two of these findings will be present in approximately 60% of patients. The abdomen may be distended and tender, and there may be an elongated mass in the right upper or lower quadrants. Rectal examination may reveal either occult blood or frankly bloody, foul-smelling stool, classically described as "currant jelly." An air enema using fluoroscopic guidance is useful for both diagnosis and treatment.

A 6-week-old infant is brought to the Emergency Room with parental concern for one week of cough and congestion. Although her cough has worsened, she remains afebrile. Chest radiography reveals bilateral interstitial infiltrates and hyperexpansion. What is the treatment of choice? A.Amoxicillin B.Cefotaxime C.Erythromycin D.Reassurance and return precautions

Correct Answer ( C ) Explanation: The infant has classic findings of pneumonia caused by Chlamydia trachomatis. The onset C.trachomatis pneumonia typically occurs between two and nineteen weeks of life, and almost all affected infants have symptoms by eight weeks of life. Symptoms often begin with nasal congestion and progress to a "staccato cough" and tachypnea. A lack of fever is classic and helps to differentiate chlamydial pneumonia from other causes of pneumonia in this age group. Examination typically reveals rales, but wheezing is uncommon. Chest radiography shows hyperinflation and interstitial infiltrates. The treatment of choice is fourteen days of oral erythromycin. Audio of staccato cough 00:0000:00 Amoxicillin (A) and cefotaxime (B) are used to empirically treat typical pathogens of community-acquired pneumonia such as S.pneumoniae, M.catarrhalis, S.pyogenes, and H.influenzae. However, viral pneumonia is a more common cause of community-acquired pneumonia than typical bacterial pneumonia during infancy. Moreover, the combination of the patient's age, history (including lack of fever), and radiographic findings make community-acquired pneumonia less likely than C.trachomatis pneumonia. Reassurance and return precautions (D) are appropriate in the setting of both bronchiolitis and viral pneumonia. However, the patient's age, history, and radiography are concerning for C.trachomatis pneumonia.

A 46-year-old woman presents to her primary care provider with a complaint of severe itching on her hands and fingers. On physical examination, linear vesicles and pustules with some excoriation are seen in the web spaces of the fingers. Which of the following will confirm the suspected diagnosis? A.Gram stain and culture B.Patch testing C.Skin scraping and microscopy D.Wood's lamp and KOH prep

Correct Answer ( C ) Explanation: This patient has Scabies, which is caused by an infestation with Sarcoptes scabiei. It is usually spread by being in close contact with an infected individual. The classic symptom of scabies is generalized, severe pruritus, particularly at night. Burrows, vesicles and pustule particularly in the finger web and wrist crease are hallmark physical findings. Definitive diagnosis is made by visualization of mites, ova, or feces under microscopy. Gram stain and culture (A) should be performed on lesions suspected to have a bacterial origin; however given that this patient likely has scabies, gram stain would not provide any diagnostic value. Patch testing (B) is utilized to identify a causative agent and confirm a diagnosis of allergic contact dermatitis. Wood's lamp and KOH preps (D) are utilized to diagnose lesions caused by fungal infections of the skin.

A 32-year-old woman presents complaining of "bumps" in her vaginal area that she noticed two weeks ago. She has never had these in the past. She denies any pain. Physical exam reveals several flesh colored lesions with prominent papillae that are non-tender to palpation on the labia as shown below. Which of the following is the next best step in management? A. Referral to general surgery for excision B.Application of trichloroacetic acid C.Perform colposcopy with acetic acid application D.Perform shave biopsy for confirmation

Correct Answer ( C ) Explanation: This patient is suffering from condyloma acuminatum, or genital warts, caused by an infection with the human papilloma virus (HPV), commonly types 6 and 11. Performing a colposcopy with acetic acid application will ensure that there is no infection of the cervix with condyloma acuminatum. This is important because human papilloma virus (HPV) infection is strongly associated with the development of genitourinary and rectal cancer. HPV is a sexually transmitted infection that can be prevented with vaccination. All girls and boys who are 11 or 12 years old should get the recommended series of HPV vaccine regardless of sexual history. Symptoms include flesh colored to whitish, exophytic or papillomatous growths in the genital area. In women, this can affect the vagina, cervix, vulva, oropharynx, perineum, and perianal areas. HPV can be spread via sexual contact or from mother to newborn during passage through the birth canal. Treatment includes trichloroacetic acid solution, podophyllin, cryosurgery, surgical excision, or imiquimod cream application. Application of trichloroacetic acid (A) is a viable treatment option; however, before treatment is initiated, the extent of the infection should be established. Performing a shave biopsy (C) is often not necessary as the diagnosis can be made clinically. Referral to general surgery for excision (D) may be necessary for lesions that are large and need debulking.

An 8-year-old boy presents with fever for 3 days. He had a fever, cough and nasal congestion 2 days ago and this morning began with a rash. Examination reveals maculopapular, red lesions over the face, neck and chest. You also note conjunctivitis. He is otherwise well appearing. What management is indicated? A.Ceftriaxone B.Isolation of patient from family C.Supportive care D.Tetracycline

Correct Answer ( C ) Explanation: This patient presents with symptoms consistent with measles requiring supportive care. Measles is a highly contagious viral illness spread by infectious droplets. The incubation period for the virus is 10-14 days and patients are contagious 2 days prior to the onset of symptoms to 4 days after the rash appears. The rash is typically preceded by fever, which increases daily for 5-6 days, and malaise. Cough, coryza and conjunctivitis begin about 24 hours after the onset of fever. Koplik's spots, a pathognomonic finding, appear on the second day of illness. They are small, bright red spots with blue-white centers appearing the buccal mucosa. Rash follows on the fourth to fifth day of the illness. The rash is characterized by maculopapular lesions beginning on the forehead and face and spreading to the trunk, arms and legs. Treatment for measles focuses on supportive care and recognition of bacterial complications. Isolation of infected patients is usually not helpful as exposure usually occurs prior to identification of the disease. Additionally, patients are not contagious after the rash has been present for 5 days. Administration of human immune serum globulin (ISG) can modify the course of disease if given within 6 days of exposure. Live measles virus vaccine may prevent measles if given within 72 hours of exposure. Ceftriaxone (A) may be helpful if a post-measels bacterial pneumonia is suspected. Otherwise, it does not play a role in treatment. Isolation (B) is likely to be unhelpful, as the family has already been exposed during the infectious period. Tetracycline (D) is the treatment of choice for Rocky Mountain spotted fever which is characterized by a rash, which spreads from extremities centrally.

A 28-year-old woman at 31 weeks gestation comes to the emergency department after noticing a gush of clear fluid from her vagina. Which of the following features is most suggestive of preterm rupture of membranes? A.Microscopic evaluation of vaginal fluid revealing pseudohyphae B.Presence of bloody vaginal secretions C.Vaginal epithelial cells with a stippled appearance on microscopic evaluation D.Vaginal fluid pH > 6.5

Correct Answer ( D ) Explanation: A patient being evaluated for preterm rupture of membranes should undergo a sterile speculum exam with pH testing of vaginal fluid and evaluation for ferning. Normal vaginal fluid is acidic (pH ranging between 3.5 to 6). Amniotic fluid is alkaline. Therefore a color change of nitrazine paper to blue, indicating a pH of > 6.5, signals the presence of amniotic fluid. Of note, blood and semen can cause false positive results. Ferning, a distinctive pattern of amniotic fluid as it crystallizes on a glass slide, also indicates the presence of amniotic fluid. Blood may obscure ferning, and mucus may cause a false positive result. The cervix should also be visually inspected for effacement and dilatation. Other clues to premature rupture of membranes include fluid pooling in the vagina as it leaks from the cervix. The patient may be asked to cough or Valsalva to see if this induces more leakage of fluid. If preterm rupture of membranes is suspected, obstetric consultants should be contacted. Women with preterm rupture of membranes usually progress to active labor. Preterm labor before 23 weeks gestation is associated with very high fetal mortality rates. Women between 24 and 36 weeks of gestation with preterm labor are candidates for tocolysis to prolong the pregnancy to allow continued time for fetal development. The decision to use tocolytics must be made by an obstetrician who can discuss risks and benefits with the patient. Antenatal administration of corticosteroids is indicated for women prior to 34 weeks gestation to speed fetal lung maturity and is associated with improved outcomes. Microscopic evaluation of vaginal fluid revealing pseudohyphae (A) is characteristic of candidal yeast infection. Bloody vaginal secretions (B) may be suggestive of bloody show (loss of cervical mucous plug), but does not indicate preterm rupture of membranes unless the presence of amniotic fluid is confirmed. Vaginal epithelial cells with a stippled appearance on microscopic evaluation (C) is indicative of bacterial vaginosis.

A 26-year-old obese woman with a 2-year history of increased acne, abnormal hair growth, and menstrual abnormalities presents to her obstetrician for an infertility workup. A pelvic ultrasound reveals enlarged cystic ovaries. She desires to become pregnant. Which of the following is the first line treatment? A.Clomiphene citrate B.Gonadotropin therapy C.Spironolactone D.Weight reduction

Correct Answer ( D ) Explanation: Polycystic ovarian syndrome (PCOS) is characterized by androgen excess, insulin resistance, and gonadotropin abnormalities. PCOS is the most frequent cause of anovulatory infertility. The diagnostic criteria for PCOS are chronic anovulation and hyperandrogenism (established by hormone measurements or clinical findings such as acne or hirsutism) in women whom secondary causes have been excluded. Polycystic appearing ovaries on ultrasound examination are a nonspecific finding in PCOS. Important factors in the physical examination include the following: the presence of acne, balding, or clitoromegaly; the distribution of body hair; enlargement of the ovaries (based on a pelvic examination); and signs of insulin resistance. Weight reduction is the first line treatment for polycystic ovarian syndrome for women who desire to conceive. As little as 5% total weight reduction has been shown to improve the metabolic and reproductive abnormalities in PCOS. Clomiphene citrate (A) is the next line of treatment if weight loss does not lead to return of ovulation. Multiple studies have shown conflicting evidence comparing the efficacy of metformin and clomiphene for ovulation induction, however clomiphene remains the first line pharmacologic agent for ovulation induction in PCOS. Gonadotropin therapy (B) is used if clomiphene citrate fails. Low-dose gonadotropin therapy is recommended over high-dose therapy. Spironolactone (C) is the first-line agent to treat hirsutism in women with PCOS and has no role in ovulation induction. Combination oral contraceptives are also used to prevent hirsutism and menstrual abnormalities in patients whom do not wish to conceive, however they are not first-line.

A 62-year-old man reports to the ED with new-onset, crushing, left-sided chest pain, radiating to the left arm that began suddenly 35 minutes prior to arrival. The patient has a history of hypertension, hypercholesterolemia, diabetes mellitus, and a 60-pack-year smoking history. His EMS ECG demonstrates ST-segment elevation in leads II, III, and aVF. In the ED, his vital signs are BP 135/75, HR 98, and RR 18. What is the most appropriate next step? A.Arrange for the patient to have an emergent stress test B.Call the cath lab emergently and prepare the patient for transport C.Give the patient nitroglycerin and draw labs, including troponins D.Place the patient on a cardiac monitor, give the patient oxygen if hypoxic and administer aspirin

Correct Answer ( D ) Explanation: This patient has an acute myocardial infarction (MI). An acute MI is clinically characterized by left-sided, substernal, chest pain (often described as an "elephant on my chest," tightness, or pressure rather than pain itself) that radiates down the left arm or left jaw, diaphoresis, nausea/vomiting, and shortness of breath. These symptoms are a result of myocardial death due to coronary vessel occlusion or vasospasm, often as a result of rupture of an atherosclerotic plaque. The definition of an acute MI, as described by the European Society for Cardiology and American College of Cardiology (ACC) is a rise and fall of a cardiac biomarker (troponin) in addition to clinical symptoms, ECG changes, or coronary artery changes as noted on an interventional level. Risk factors for an acute MI include hypertension, hypercholesterolemia, diabetes, tobacco, male, increased age, and family history. This patient has ST-segment changes in leads II, III, and aVF, correlating to an inferior wall MI. In a patient with an acute MI, the first step in management is to place the patient on a cardiac monitor to recognize any dysrhythmias, establish a peripheral IV, give oxygen if hypoxic, and administer aspirin. An ischemic right ventricle becomes preload dependent because it can no longer pump blood to the left side of the heart. Administering nitroglycerin (C) (a preload reducer) to a patient having ischemia of the right ventricle can lead to severe hypotension. Because the right coronary artery often supplies the inferior aspect of the heart and the right ventricle, whenever there is evidence of ST-segment elevation in the inferior leads (II, III, aVF), it is important to exclude a right ventricular infarct by obtaining "right-sided leads." This is performed by placing the precordial leads on the right side of the patient's chest instead of the left. Lead rV4 is thought to be the most sensitive in identifying a right ventricular infarct. Similarly, labs, including serial cardiac biomarkers (troponin) should be drawn; however, this is not the most appropriate next step. It is important to notify the cath lab (B) of the patient's need for percutaneous intervention, but the first priority are the ABCs and administering an aspirin. The cath lab should be notified immediately after this is performed. Although a stress test (A) would be useful in a patient with cardiac risk factors, this patient has active cardiac symptoms and the ACC/AHA guidelines recommend such testing be performed when patients are free of ischemic or heart failure symptoms for at least 8-12 hours. In addition, this patient has evidence for a STEMI and should undergo percutaneous intervention in the cath lab.

A patient presents with back pain. Radiographic findings include a herniated nucleus pulposus between L5 and S1. What are you most likely to find on physical examination? A.Decreased lateral leg sensation B.Urinary incontinence C.Weak dorsiflexion of the foot D.Weak plantarflexion of the foot

Correct Answer ( D ) Explanation: Weak plantarflexion of the foot is a physical examination finding specific for an L5-S1 herniated nucleus pulposus. A herniated nucleus pulposus, also known as a herniated disc, occurs when there is a weakness in the outer annulus fibrosus and the inner nucleus pulposus herniates outward through the small tears. If not corrected, the nucleus pulposus continues to push outward causing impingement on the spinal nerves resulting in radiculopathy, or disease of a nerve. When a disc herniation occurs between the L5 and S1 vertebra, an S1 radiculopathy occurs resulting in pain radiating down the posterior aspect of the leg and foot along with weakness with plantarflexion due to weakness of the gastrocnemius muscle. The ankle reflex is also typically lost. Diagnosis is formulated through physical examination findings as well as confirmation from imaging. Treatment includes pain medications, physical therapy, and surgical interventions, such as a discectomy to remove the herniated nucleus pulposus from the impinged nerve. Decreased lateral leg sensation (A) is a finding of a nerve impingement at the L4-L5 level. An L5 nerve root impingement is one of the most common sites of radiculopathy of the spine. Patients present with low back pain that radiates down the lateral aspect of their leg and foot and can result in foot drop or decreased strength of foot dorsiflexion. Reflexes are typically spared with an L5 radiculopathy. Weak dorsiflexion of the foot (C) is a result of a herniated nucleus pulposus between L4 and L5. At this level, an L5 radiculopathy causes weakness in foot dorsiflexion, inversion, eversion, and toe extension. Weakness of the foot dorsiflexors results in foot drop because the patient can't lift the foot up. Urinary incontinence (B) is an alarming sign of sacral radiculopathy that is known as cauda equina syndrome. This is a rare, but emergent process in which extreme pressure or swelling compresses the nerve roots intrathecally. Nerves may be compressed at a higher level within the lumbar spine, but the amount of pressure causes symptoms of decreased perineal sensation, as well as urinary and bowel incontinence and sexual dysfunction. This is a life-threatening emergency, and treatment includes emergent decompression of the impinged nerves.

A 47-year-old man presents for evaluation of a whitish discoloration on his tongue for the last several days. He denies any fever or other systemic signs. On examination, the following findings are seen on the image above. You are unable to scrape the base of the plaque off with a tongue depressor. Which of the following tests is likely to be positive? A.Gram stain demonstrating large, ovoid, gram-positive yeast B.KOH prep C.Rapid HIV D.Rapid strep

This patient's tongue demonstrates the white corrugated plaques of oral hairy leukoplakia. This is a disease of the lateral tongue caused by infection of the squamous epithelium with Epstein-Barr virus. Most commonly the infection is localized to the lateral portion of the tongue, although in rare cases it may involve the floor of the mouth, palate or buccal mucosa. Clinically this may be difficult to differentiate from thrush caused by candida. However, the lesions of oral hairy leukoplakia cannot be scraped off the surface. Although caused by Epstein-Barr, it is rarely seen in conditions other than patients with HIV infection. A Gram stain demonstrating large, ovoid, gram-positive yeast (A) is consistent with candidiasis. A KOH prep (B) may be used to identify candida infections. On the prep, pseudohyphal elements will be seen microscopically. A rapid strep test (D) would not be positive in this individual. Strep characteristically has a white exudate over the tonsils and posterior pharynx. When using the rapid strep diagnostically, it has a high specificity, but poor sensitivity requiring follow-up culture for negative rapid tests.

A 58-year-old man presents with 3 months of headache and diplopia. He also reports that chewing tough foods has been progressively painful. Laboratory testing reveals a normocytic anemia and ESR of 88 mm/hour. A brain CT is normal. Which of the following is the most appropriate initial intervention? A. Intravenous methylprednisolone B. Oral methotrexate C. Temporal artery biopsy D. Ultrasonography

Correct Answer ( A ) Giant cell arteritis (GCA), as known as temporal arteritis, is the most common primary vasculitis of the elderly, and predominately affects those aged over 50 years. It is a syndrome of systemic inflammation which mainly affects the branches of the internal and external carotid arteries, but it can affect any vessel in the body. The most common symptoms are a temporal or occipital headache, jaw claudication (mandibular, tooth and tongue pain with chewing tough foods), visual changes (partial or complete blindness, diplopia, visual field cuts or amaourosis fugax) and arthralgias. It is associated with constitutional symptoms, anemia, an enlarged tender nodular erythematous temporal artery, ESR > 50 mm/hour and polymyalgia rheumatica. Temporal artery biopsy confirms the diagnosis. Since unilateral partial or complete blindness occurs in up to 20% of patients with GCA, immediate treatment is necessary when this condition is suspected. High-dose corticosteroids are the standard treatment, and typical options include 40-60 mg per day of oral prednisone, or intravenous methylprednisolone. High-dose therapy is usually continued for 2-4 weeks, and titrated against visual and headache improvement, not serial ESR monitoring. After symptoms lessen, steroids are slowly tapered over 6 months, with complete tapering off in 2 to 3 years. As such, the clinician must monitor for complications of long-term corticosteroid therapy. These include hypertension, diabetes, osteoporosis, steroid myopathy (proximal weakness), fluid retention, bruising, insomnia, restlessness, hypomania and hypercholesterolemia. Adjuvant methotrexate (B) is not routinely recommended for treating GCA as studies of its efficacy are inconclusive. If GCA is suspected, intravenous methylprednisolone should be started even before a temporal biopsy (C). Steroids will not affect the validity of a biopsy result until about 3-4 weeks of treatment, however, a delay in steroids may lead to blindness. There is some evidence of the usefulness of temporal artery ultrasonography (D) for detecting the best area of the artery to biopsy. However, a negative ultrasound does not rule out GCA, and furthermore, some studies have shown that temporal artery ultrasound sometimes cannot differentiate degenerative changes from inflammatory changes.

A 7-year-old girl presents with the rash seen above. Her mother states it appeared 7 days after hiking through the woods near their home in New Jersey. She has no known drug allergies. Which of the following is the most appropriate treatment for this patient? A. Amoxicillin B. Azithromycin C. Cephalexin D. Doxycycline

Correct Answer ( A ) Lyme disease is the most common vectorborne disease in the United States. It is a tickborne illness caused by the spirochete Borrelia burgdorferi. The early phase of the disease results in the characteristic rash seen above, erythema migrans. Prompt treatment of early disease can shorten the duration of symptoms and prevent progression to later stages of disease. Pregnant or lactating women and children younger than 8 years of age should receive amoxicillin. Advanced or severe disease should be treated with intravenous ceftriaxone or penicillin. Azithromycin (B), a macrolide antibiotic, is not recommended as a first-line agent for early Lyme disease. It should be reserved for those who cannot tolerate doxycycline or amoxicillin. Cephalexin (C) is a first-generation cephalosporin that is ineffective against Lyme disease. Doxycycline (D) is the drug of choice for men, nonpregnant and nonlactating women, and children older than 8 years of age. Doxycycline is a tetracycline that is readily bound to calcium deposited in newly formed bone or teeth in young children. When administered during pregnancy, it can be deposited in the fetal teeth, leading to fluorescence, discoloration, and enamel dysplasia. It can also be deposited in bone, where it may cause deformity or growth inhibition.

A 35-year-old woman is seen in the clinic with a chief complaint of frequent palpitations associated with symptoms of pre-syncope. She experiences these episodes two to three times per day. Her resting ECG in the office is normal sinus rhythm with heart rate 65/min. Her physical exam is unremarkable. Which of the following is the most appropriate next step in management? A.24-48 hour continuous ambulatory ECG monitor B.Electrophysiologic testing C.Implantable loop recorder D.Post-symptom event recorder

Correct Answer ( A ) Explanation: 24-48 hour continuous ambulatory ECG monitor, also known as a Holter-monitor, is the most appropriate next step in management. It is one of the most frequently used and cost-effective noninvasive tests used to evaluate cardiac rhythm abnormalities. The clinical utility of the ambulatory ECG recording lies in its ability to continuously examine the patient's cardiac rhythm over an extended period of time during normal routine activity, including any physical and psychological changes. Various rhythm recorders can be used to capture a dysrhythmia. Selection depends on the frequency and duration of symptoms. A Holter monitor is worn for 24-48 hours and is used for evaluation of more frequent symptoms. This patients symptoms occur daily, so a 24-48 hour monitor is likely to capture a possible dysrhythmia during an episode of palpitations or pre-syncope. Several other options to monitor cardiac rhythm abnormalities exist and include event recorders and implantable loop recorders. These monitors are most useful when symptoms are infrequent. A post-symptom event recorder (D) is held to the chest when symptoms occur. The advantage of a post-symptom recorder is the lack of electrode leads, making it more comfortable to carry. Implantable loop recorders (C) allow for continuous ECG monitoring to detect cardiac arrhythmias over months to years. This device is placed subcutaneously, similar to a pacemaker, and can be interrogated non-invasively. A looping event recorder records several seconds of the rhythm prior to the device being triggered and is useful in patients with syncope who many not be able to trigger the recording device. This patient's symptoms are frequent making these devices unnecessary. Electrophysiologic testing (B) is invasive and reserved for those at risk of sudden cardiac death. It is indicated in select, high risk patients with unexplained syncope, particularly those with structural heart disease. Electrophysiologic testing should be considered when noninvasive tests have failed to make a diagnosis. This patient is not likely to have structural heart disease or be at risk for sudden cardiac death based upon her history, physical exam findings and initial testing.

Which of the following signs has the greatest likelihood ratio for acute otitis media? A.Impaired mobility of the tympanic membrane. B.Red tympanic membrane C.Retracted tympanic membrane D.Ruptured tympanic membrane

Correct Answer ( A ) Explanation: A tympanic membrane with distinctly impaired mobility has an adjusted likelihood ratio of 31. A tympanic membrane with slightly impaired mobility has an adjusted likelihood ratio of 4. A tympanic membrane with normal mobility is unlikely to be consistent with acute otitis media. A distinctly red tympanic membrane (B) has an adjusted likelihood ratio of 2. A retracted tympanic membrane (C) has an adjusted likelihood ratio of 0.6. A ruptured tympanic membrane (D) is nonspecific for acute otitis media. Another sign that suggests acute otitis media is a cloudy tympanic membrane with an adjusted likelihood ratio of 34.

A 16-year-old boy with a history of asthma presents complaining of increasing episodes of evening and daytime symptoms. He is on a short acting inhaled beta agonist on an as needed basis. He presently needs to use his short acting beta agonist daily. Which of the following is the most appropriate addition to this patient's medication regimen? A.Inhaled corticosteroid Correct Answer B.Leukotriene inhibitor C.Long acting beta agonist inhaler D.Methylxanthine oxidase inhibitor

Correct Answer ( A ) Explanation: According to the stepwise approach for managing asthma by the National Asthma Education and Prevention Program, inhaled corticosteroids are indicated for mild to moderate persistent asthma. For most patients twice-daily dosing provides adequate control of asthma symptoms. Short acting beta agonists are helpful with controlling acute exacerbations by acting as bronchodilators; however they are not helpful in the long term management in patients with persistent asthma. Using an inhalation chamber, also known as a "spacer," along with rinsing of the month after inhaled corticosteroid use decreases local side effects and systemic absorption. Long acting beta agonist inhalers (C) should not be used in place of anti-inflammatory therapy. Salmeteol and formoterol are the two long acting beta 2 agonists available for asthma. They are indicated for long-term prevention of asthma symptoms, nocturnal symptoms, and for the prevention of exercised induce bronchospasms. Leukotriene inhibitors (B) are less desirable alternatives to inhaled corticosteroids. They are used to control smooth muscle airway contraction, increase vascular permeability and mucous secretion. Methylxanthine oxidase inhibitors (D) preparations may have beneficial effects in some patients, but their value is limited due to a narrow therapeutic window and modest efficacy.

A 57 year-old man presents to the office with a chief complaint left eye pain and seeing halos around lights. His left eye is seen above. Which of the following is the most likely diagnosis? A.Acute angle closure glaucoma B.Acute anterior uveitis C.Bacterial conjunctivitis D.Hyphema

Correct Answer ( A ) Explanation: Acute angle-closure glaucoma is an ophthalmologic emergency due to a closed anterior chamber angle. This is the only type of glaucoma that is curable. Patients present with acute onset of pain and vision loss that is most commonly unilateral. As intraocular pressure rises, patients may experience headache, nausea, vomiting, and abdominal pain that is sometimes mistaken as an acute abdominal process. The eye exam may reveal a red, tender globe that may be firm to touch. The cornea may be steamy or hazy and the pupil is mid-dilated and nonreactive. An ophthalmologic consult should be obtained immediately. Treatment incudes topical drops such as carbonic anhydrase inhibitors and beta-blockers. Permanent vision loss can occur if treatment is delayed. Acute anterior uveitis (B) presents with a sudden onset of unilateral eye redness, photophobia, pain and lacrimation. It is associated with systemic illnesses including anklosing spondylitis, Crohn's, ulcerative colitis, reactive arthritis and sarcoidosis. Bacterial conjunctivitis (C) is a self-limiting condition that is characterized by acute onset of redness in one or both eyes, a gritty or burning sensation and discharge that begins watery and eventually becomes purulent. Many patients awaken with their eyelids stuck together. Treatment consists of antibiotic eye drops or ointments. A hyphema (D) is usually secondary to globe trauma and is caused by hemorrhage from either the ciliary body or the anterior chamber.

A 74-year-old man is having a preoperative ECG performed. What is your interpretation of his ECG? A.Atrial fibrillation B.Atrial flutter C.Normal sinus rhythm D.Sinus tachycardia

Correct Answer ( A ) Explanation: Atrial fibrillation is an irregularly irregular rhythm due to uncoordinated atrial activation and random occurrence of ventricular depolarization. The atria are not contracting, but they do discharge electrical impulses to the ventricles. However, no single impulse depolarizes the atria completely, so only an occasional impulse gets through the AV node. It is the most common sustained dysrhythmia in clinical practice. Atrial flutter (B) is a rapid atrial rhythm, but due to nodal delay, ventricular response rate is slower. Therefore, atrial flutter always occurs with some sort of AV block so that not all impulses are conducted. The resulting block is often variable (2:1, 3:1, 4:1). P waves have a characteristic sawtooth pattern. In normal sinus rhythm (C) and sinus tachycardia (D), the SA node is the pacemaker that causes the atria to depolarize regularly and, thus, the ventricles to depolarize regularly. Therefore, the ECGs for both of these rhythms have P waves and QRS complexes that occur regularly. The difference between these two rhythms is with the rate. The rate of sinus rhythm is 60-99. The rate of sinus tachycardia is >100.

Which of the following is an acyanotic congenital cardiac defect? A.Coarctation of the aorta B.Ebstein's anomaly C.Hypoplastic left heart syndrome D.Pulmonary atresia

Correct Answer ( A ) Explanation: Coarctation of the aorta is an acyanotic congenital heart lesion. Acyanotic congenital heart defects (CHD) should be divided into obstructive lesions and lesions with left-to-right shunts and increased pulmonary blood flow. Acyanotic lesions typically present in the first 6 months of life with congestive heart failure. Obstructive CHD includes pulmonic stenosis, aortic stenosis and coarctation of the aorta. Left-to-right shunt CHD includes VSD, ASD, patent ductus arteriosus and endocardial cushion defects. Coarctation of the aorta accounts for approximately 8% of all CHDs. Ebstein's anomaly (B), hypoplastic left heart syndrome (C) and pulmonary atresia (D) are cyanotic heart lesions

You suspect a cardiac structural defect in a 6-month-old infant with peripheral edema and hypertension. Skin color is normal. A murmur is not appreciated. However, the blood pressure is markedly higher in the arms than legs. Which of the following is the most likely diagnosis? A.Aortic coarctation B.Great artery transposition C.Patent ductus arteriosus D.Tetralogy of Fallot

Correct Answer ( A ) Explanation: Congenital cardiac defects can be defined into two classifications: cyanotic and acyanotic. The most common acyanotic lesions are coarctation of the aorta, pulmonary stenosis, aortic stenosis, atrial septal defect, ventricular septal defect, atrioventricular canal and patent ductus arteriosus. Coarctation of the aorta is characterized by narrowing of the aorta anywhere along its length but most commonly around the left subclavian artery takeoff. This acyanotic lesion presents with congestive heart failure, cardiomegaly, hypertension and shock. The classic exam finding is blood pressure higher in the arms than legs, and pulses bounding in the arms and decreased in the legs. Rib notching (inferior aspect of the rib) is the classic radiographic finding in patients with coarctation of the aorta. This occurs as a result of dilatation of intercostal arteries. Surgical correction is usually required, and is most commonly performed between 2 and 4 years of age. Tetralogy of Fallot (D) and great artery transposition (B) are cyanotic, not acyanotic, congenital defects. Patent ductus arteriosus (C) can present with symptoms of congestive heart failure. However, the most common finding is a continuous, systolic and diastolic murmur, commonly called a machinery-like murmur. Although the pulse pressure is also commonly widened, the blood pressure is usually not asymmetric between arms and legs.

A 67-year-old African-American man established care with you two months ago after being told at a community health fair that his blood pressure was elevated. The blood pressure readings done in your office include 168/94, 158/98 and 162/96. You determine that the patient has primary essential hypertension. Lifestyle measures have already been instituted but you decide that medication is also warranted at this time. Which of the following is the best initial agent for blood pressure control in this patient? A.Amlodipine B.Clonidine C.Metoprolol D.Prazosin

Correct Answer ( A ) Explanation: Current studies suggest that the best initial choice for treating hypertension in African-American patients without diabetes or chronic kidney disease is either a calcium-channel blocker or a thiazide-type diuretic. When used as monotherapy, either of these agents is more effective at lowering blood pressure than drugs in other classes, though many patients require the addition of a second anti-hypertensive agent for adequate control. Clonidine (B) is a central alpha sympathetic agonist used for the treatment of hypertension. It is associated with several drug-drug interactions and orthostatic hypotension. Although it is used as an anti-hypertensive in some patients, it is not recommended as first-line therapy. Metoprolol (C) is a cardioselective beta-blocker used in the management of hypertension. Beta-blockers have been found to be comparatively less effective agents for the treatment of hypertension in African Americans, and in recent years have fallen out of favor as the best initial option for managing hypertension in other patients groups, as well. Prazosin (D) is one of several alpha-adrenergic receptor blockers which work to lower blood pressure by decreasing peripheral vascular resistance. It is not recommended as a first-line agent.

What is the most common location for spontaneous bleeding in children with hemophilia A? A.AnkleCorrect Answer B.Elbow C.Gastrointestinal tract D.Knee

Correct Answer ( A ) Explanation: Hemophilia A is an X-linked recessive disease that includes a range of clinical manifestations characterized by the activity of factor VIII. Severe hemophilia is characterized by spontaneous hemarthrosis. It differs from platelet disorders, such as von Willebrand disease that manifest as mucosal bleeding (epistaxis, gingival bleeding) and bullous hemorrhage of buccal mucosa. In patients with mild (>5% factor VIII activity) and moderate hemophilia (1%-4% factor VIII activity), usually an insult causes bleeding, whereas spontaneous bleeding is more common in severe disease (<1% factor VIII activity). Spontaneous intracranial hemorrhage can occur in all ages but is more common in neonates. Patients with hemophilia seldom bleed from small cuts or venipuncture, but they do hemorrhage out of proportion to larger injuries. Delayed bleeding is also common. Although bleeding may occur in any area of the body, the most common sites include joints, muscles, and the GI tract. However, 80% of hemorrhage occurs in the joints. In children, the most common joint is the ankle. In adults, the most common joint is the knee, followed by the elbow and ankle. The elbow (B), gastrointestinal tract (C), and knee (D) are all locations for spontaneous bleeding in children with hemophilia A but less common than the ankle is.

A woman complains of acute onset and rapidly progressive dark hair growth on her upper lip. She has also noticed a deepening of her voice, an increase in muscle bulk and a cessation of normal menses. Abdominal CT scanning reveals a large adrenal gland mass. Histopathologic evaluation shows the presence of malignant cells. An excess of which of the following class of hormones is most likely to be found in this patient? A. AndrogensCorrect Answer B. Catecholamines C. Eicosanoids D. Mineralocorticoids

Correct Answer ( A ) Explanation: Hirsutism, excessive terminal hair in androgen-dependent areas of a woman's body, affects up to 8% of women. It is usually a result of increased androgen levels. Underlying etiologies are many, including polycystic ovarian syndrome, late-onset adrenal hyperplasia and Cushing's syndrome. Adrenal neoplasia is also a cause, and a primary adrenal gland androgen-secreting neoplasia, typically a carcinoma of the androgen-producing cells, usually results in a rapid progression of hirsutism, development of male characteristics and cessation of menses. An excess of catecholamines (B), epinephrine and norepinephrine, can result from a pheochromocytoma. This adrenal medulla neoplasia causes hypertension, headache, palpitations, pallor and diaphoresis, not hirsutism and female masculinization. Eicosanoid hormones (C) include prostaglandins, leukotrienes and thromboxanes. These hormones are involved in the immunity and inflammation pathways, not the masculinization pathways. Mineralocorticoid (D) excess (hyperaldosteronism) causes hypertension, headaches and polydipsia.

A previously healthy 21-year-old man is found unresponsive by his roommates. On arrival to the Emergency Department, his pupils are pinpoint and oxygen saturation is 88% on room air. A blood gas reveals a pH of 7.25, PCO2 of 60 mm Hg, PO2 of 65 mm Hg, and a HCO3- of 26 mEq/L. Which of the following mechanisms is the most likely cause of his hypercapnia? A.Decreased respiratory drive B.Increased CO2 production C.Increased dead space ventilation D.Right-to-left shunt

Correct Answer ( A ) Explanation: Hypercapnia is defined as a PCO2 > 45 mm Hg. Hypercapnia is caused by alveolar hypoventilation. If ventilation is inadequate, CO2 is not exhaled at an appropriate rate and levels build up in the blood. The patient's elevated PCO2 and bicarbonate at the upper limit of normal indicates that he has an acute respiratory acidosis from hypoventilation. Hypoventilation results from a decreased respiratory rate, decreased tidal volume, or an increase in dead space ventilation. This can occur via numerous mechanisms: depressed central respiratory drive from a brainstem lesion, drug-induced depression of the respiratory center, neuromuscular disease (e.g. myasthenia gravis or Guillain-Barre syndrome), or conditions preventing adequate chest wall mobility (e.g obesity or tetanus). In this scenario of a young individual with pinpoint pupils, an opioid overdose causing respiratory depression is a likely cause of hypoventilation and resultant hypercapnia. Increased CO2 production (B) is incorrect. Hypercapnia is always caused by alveolar hypoventilation, not increased production. Increased dead space ventilation (C) is a cause of alveolar ventilation and can contribute to hypercapnia. However, in this clinical scenario opiate-induced respiratory depression is the more likely cause. Right-to-left shunt (D) causes hypoxemia, but not hypercapnia.

On a Sunday afternoon, a surgical oncologist and his family attend a football game in the city where he practices. While at the game, he runs into a physician colleague that works at the same institution. After some casual small talk, his colleague inquires, "Are you taking care of Mr. Clarke, my personal trainer? I heard through the grapevine that he has melanoma, and I didn't know if you had started him on any chemotherapy or performed any surgical intervention yet. Hopefully you'll be able to take very good care of him." In this situation, the surgical oncologist may confirm which of the following? A.No information at all B.Only that Mr. Clarke is his patient C.The patient's diagnosis D.The patient's treatment plan

Correct Answer ( A ) Explanation: In order to be in compliance with patient confidentiality, a physician must not discuss any information regarding a patient's care with a physician who is not actively involved in that patient's care. This not only includes the diagnosis, treatment and prognosis of a patient's care, but also the confirmation or denial of whether or not a person is, in fact, a patient of the physician in question. A physician has an ethical responsibility to his or her patients to respect and protect their confidentiality in every situation, including non-physician interactions as well as with physician colleagues who are not involved in the active care of the patient. The most appropriate course of action in a situation as described above would be to withhold information concerning the patient's condition and medical course. Physicians should not be dishonest or lie in order to protect patient confidentiality. Federal and state laws designed to safeguard patient confidentiality are inadequate against the rapid and innovative use of electronic health websites. Health professionals must be aware that this information is not always secure and that they are accountable for maintaining privacy for the patient. A physician should not divulge any information about a patient, including names, diagnoses (C), treatment plan (D), prognoses, and care providers (B) to any other physician not currently taking care of that patient.

A patient with irritable bowel syndrome complains mainly of lower abdominal pain. She denies diarrhea or constipation, and rarely has problematic flatulence. For this patient's abdominal pain, which of the following medications do you recommend? A.Desipramine B.Loperamide C.Lubiprostone D.Rifaximin

Correct Answer ( A ) Explanation: Irritable bowel syndrome is considered an intestinal motility disorder in which chronic diarrhea is the main symptom. Associated symptoms include abdominal pain, bloating, flatulence and constipation. Some dysmotility causes include improper secretion due to luminal or environmental stimuli, decreased pain thresholds (hence, increased pain sensitivity) and dysregulation of the brain-intestinal axis. Rome III diagnostic criteria are based on symptom presence on >3 days per month over the past 3 months plus 2 or more of the following: defecation decreases symptoms, a change in the frequency of stooling or a change in the shape of stool. Treatment regimens are geared toward which of the symptoms a patient with irritable bowel syndrome suffers from most: pain, diarrhea, bloating or constipation. The pain associated with irritable bowel syndrome can be treated with selective-serotonin reuptake inhibitors like citalopram, tricyclic antidepressants like desipramine, and antispasmodics such as atropine, hyoscyamine, dicyclomine or scopolamine. Loperamide (B), a opioid piperidine-derivative, is used to treat diarrhea, not pain, associated with irritable bowel syndrome. Lubiprostone (C) is a prostaglandin E1 derivative. Its chloride channel agonism is used in treating the constipation, not pain, associated with irritable bowel syndrome. Bloating and flatulence that are associated with irritable bowel syndrome can be treated with rifaximin (D), a semisynthetic antibiotic.

A 4-year-old boy presents to the office with 7 days of fever. On HEENT exam you note bilateral conjunctivitis, some fissuring of his lips and palpate a cervical lymph node. The cardiovascular and pulmonary exams are normal. Skin exam reveals a macular rash on his chest and skin desquamation on his feet. Which of the following is the most likely diagnosis? AKawasaki disease BLyme disease CMeasles DScarlet fever

Correct Answer ( A ) Explanation: Kawasaki disease typically presents in children younger than 4 years, most commonly at age 2. It is more common in males and occurs more frequently in the winter and spring months. The diagnosis requires the criteria found in the image below. Laboratory findings may reveal thrombocytosis, leukocytosis, and a normocytic anemia. Acute phase reactants (CRP, ESR, and alpha-1 antitrypsin) are often elevated. Antinuclear antibodies (ANA) and rheumatoid factor (RF) are not elevated in Kawasaki disease. The main complication is coronary artery disease. Treatment includes high dose aspirin (80 mg/kg/day) and IVIG. Aspirin maintenance (5 mg/kg/day) is continued for 2 months after high dose aspirin is administered for two days. A 2D-echcardiogram should also be performed at the time of diagnosis, 2-3 weeks and 6-8 weeks after the diagnosis is made. Lyme disease (B) is characterized by fever, arthraligias, carditis, and neurologic manifestations. The characteristic rash, erythema migrans, takes on a "bull's eye" appearance. Measles (C) is also associated with an exudative conjunctivitis but also includes cough, coryza, Koplik spots, cephalocaudal development of the typical rash, and leukopenia. Ocular findings are rare in Scarlet fever (D)

A 32-year-old previously healthy woman presents to your office with complaints of headache, cough and episodes of fever, shaking, chills and sweating. She tells you that she was recently on a mission trip to Haiti and did not see a medical provider before her trip. Lab work is ordered and testing identifies Plasmodium falciparum on blood smear. Which of the following is the most appropriate therapy? A.Chloroquine B.Doxycycline C.Isoniazid D.Supportive care

Correct Answer ( A ) Explanation: Malaria is a mosquito-borne infection that is potentially fatal. The infection is caused by the Plasmodium protozoa and is transmitted by an infected female Anopheles mosquito. Cases of malaria in the United States are seen in patients who have traveled to malaria-endemic countries. Malaria is most prevalent in rural regions of countries with tropical climates. Patients planning a trip to a malaria-endemic region should be advised to take malaria prophylaxis to prevent infection. Clinical presentation includes several fever spikes per day, headache, cough, malaise, arthralgia, myalgia, chills and sweats. Diagnosis is made when the parasite is seen on blood smear with light microscopy. Regions where chloroquine-sensitive malaria is found include Haiti, the Dominican Republic, and certain parts of Central America. When infection with chloroquine-sensitive malaria is likely, treatment with the antimalarial agent chloroquine should be administered. Doxycycline (B) is a tetracycline antibiotic used in the treatment of Lyme disease and Rocky Mountain spotted fever, both tick-borne illnesses. Isoniazid (C) is used as part of the treatment regimen for tuberculosis. Supportive care (D) is used to treat yellow fever and dengue fever, which are both mosquito-borne illnesses. Patients with malaria should be treated with supportive care to help with symptoms, but should also undergo a treatment regimen based on country visited and potential for chloroquine resistance.

Which of the following is an absolute contraindication to the measles, mumps and rubella vaccine? A.Anaphylactic reaction to neomycin B.Family history of seizure C.History of autism D.History of immune thrombocytopenia

Correct Answer ( A ) Explanation: Prevention against measles, mumps and rubella (MMR) is provided by two doses of a live-virus vaccine. The recommended schedule is the first dose at ages 12-15 months and the second dose at school entry, between ages 4-6 years. The MMR vaccine contains trace amounts of neomycin, therefore patients with a history of anaphylaxis to neomycin should not receive it. Other true contraindications include previous severe allergic reaction to any component of the vaccine, individuals who are immunocompromised, pregnancy, hematologic or solid tumors, and HIV infection with immunosuppression. A family or personal history of seizure (B) is a precaution to giving the MMR vaccine, but not an absolute contraindication. While there has been a great deal of discussion about the connection between vaccinations and autism, no scientific evidence has been found linking the two and history of autism (C) is not an absolute contraindication to the MMR vaccine. There is a risk of developing immune thrombocytopenia (D) after MMR vaccination, however this occurs rarely and is not an absolute contraindication to receiving the vaccine.

A patient asks you to explain the underlying cause of her newly diagnosed type 1 diabetes mellitus. Which of the following correctly describes the underlying pathology of this disease? A. Autoimmune destruction of pancreatic cells B. Excessive hepatic glucose production C. Myocyte insulin resistance D. Pancreatic glucagon deficiency

Correct Answer ( A ) Explanation: The pathological basis of type 1 diabetes mellitus (DM1) is an autoimmune destruction of the pancreas. It is marked by lymphocytic infiltration and destruction of the islets of Langerhans beta-cells. As these cells die off, insulin secretion decreases. When 80-90% of the beta-cells are lost, hyperglycemia ensues. Genetic predisposition and viral infections play a role in generating the autoimmune response. Nearly 85% of patients with DM1 have detectable levels of autoantibodies. Other autoimmune disorders, such as Grave's and Addison's disease, are more common in those with DM1. Excessive hepatic glucose production (B) is not a mechanism for DM1, but it is associated with DM2. Normally, insulin acts on the liver to suppress endogenous production of glucose. Myocyte, and other peripheral cell, insulin resistance (C) is largely responsible for the pathophysiology of DM2, not DM1. Glucagon is normally secreted by the pancreas in response to hypoglycemia. It raises blood glucose levels by inducing hepatic glycogenolysis and gluconeogenesis. In effect, its actions are opposite those of insulin. Glucagon under secretion or deficiency (D) would result in a hypoglycemic, and not hyperglycemic, state.

Which of the following is the most common infectious etiology of diarrhea in patients with AIDS? AClostridium difficile BCryptosporidium CCytomegalovirus DMycobacterium avium complex

Correct Answer ( B ) Explanation: As the CD4 count drops, patients with AIDS are at risk for opportunistic infections of the gastrointestinal tract. Worldwide diarrhea is a source of significant morbidity and mortality of HIV-positive patients because of poor access to antiretroviral therapy. HIV infects and damages the gut-associated lymphoid tissue (GALT) which leads to frequent infection in those with poorly controlled disease and lower CD4 counts. Medications also cause diarrhea as a side effect. Additionally, patients with longstanding disease develop malabsorption syndromes from the chronic inflammatory changes of the intestinal tract. In patients with AIDS, cryptosporidium is a common cause of diarrhea when the CD4 count falls below 100 cells/mm3. Clostridium difficile (A) is a cause of diarrhea in AIDS patients with the appropriate risk factors. Cytomegalovirus (C) is a viral infection that may lead to colitis and is of particular concern in AIDS patients with rectal bleeding and tenesmus. Mycobacterium avium complex (D) is a rare cause of diarrhea and typically only occurs once the CD4 count has fallen below 50 cells/mm3.

Which of the following hormones is involved in diabetes insipidus? A. Adrenocorticotropic hormone B. Antidiuretic hormone C. Glucagon D. Insulin

Correct Answer ( B ) There are two types of diabetes insipidus (DI), central and nephrogenic. The primary symptom of both types is the passage of large amounts of dilute urine. The posterior pituitary gland secretes antidiuretic hormone (ADH), which is the hormone involved in DI. Decreased secretion of ADH is characteristic of central DI. The majority of cases of central DI are idiopathic, although it may be caused by malignancy, neurosurgery or trauma. Patients with nephrogenic DI are unable to concentrate their urine because of resistance to ADH in the kidney. Nephrogenic DI can occur with lithium toxicity, renal disease, hypokalemia, hyperglycemia, pregnancy, and as a side effect of a number of other medications including amphotericin B, cidofovir, foscarnet and ofloxacin. Diagnosis of DI is confirmed through laboratory testing. Patients with suspected DI should have a 24-hour urine collection to assess urine volume, as well as serum electrolytes and glucose, urinary specific gravity, plasma and urine osmolality, and plasma ADH level. Treatment for central DI is fluid replacement and desmopressin. Patients may need hospitalization for monitoring and determination of fluid needs. Initial treatment for nephrogenic DI is a low-salt, low-protein diet, frequent voiding, and a thiazide diuretic. Patients taking lithium may have individualized treatment considerations. Adrenocorticotropic hormone (A) is secreted from the anterior pituitary gland and stimulates the release of steroid hormones from the suprarenal gland. Glucagon (C) and insulin (D) are hormones secreted by the pancreas. Glucagon is produced by alpha cells and increases glycogen breakdown and glucose release in the liver. This process raises blood glucose levels. Insulin is produced by beta cells and lowers blood glucose levels. It is secreted in response to high blood glucose levels in the body and is the hormone involved in diabetes mellitus types 1 and 2.

A 19-year-old Asian man presents with complaints of diarrhea, flatulence, and bloating that occur everyday after breakfast. His breakfast is usually fruit with cottage cheese and a glass of milk. He denies weight loss, constipation, steatorrhea, bloody stools, or tenesmus. Which of the following tests will most likely diagnose his condition? A.Colonoscopy B.Hydrogen breath test C.IgA endomysial antibody test D.Qualitative fecal fat assay

Correct Answer ( B ) Explanation: A hydrogen breath test is the most commonly used test to diagnose lactase deficiency leading to lactose intolerance. Lactose intolerance is a very common condition worldwide, as most people of non-European heritage experience a reduction in the concentration of their lactase enzymes during childhood that continues to progress into adulthood. This results in varying manifestations of gastrointestinal distress when patients are presented with dairy products. While some patients may be able to tolerate a small quantity of lactose-containing products if consumed with other foods, others require complete avoidance to remain distress-free. Patients may complain of diarrhea, bloating, cramping abdominal pain, and flatulence after eating lactose products. However, signs of nutrient malabsorption, systemic illness, or weight loss should not occur with isolated lactose intolerance. These findings should alert clinicians to assess for other gastrointestinal disorders. If a hydrogen breath test results in a rise of breath hydrogen beyond baseline after consuming 50 grams of lactose, lactose intolerance is likely. Two week of abstaining from any lactose-containing products should then be recommended. Resolution of symptoms during that time is confirmative of lactose intolerance. At that point, patients should be counseled to seek replacements for the calcium generally found in dairy products to prevent osteoporosis. Lactase-containing preparations may be beneficial. A colonoscopy (A) does not help in diagnosing lactose intolerance, the most likely disorder in this patient. It would be more helpful if weight loss, tenesmus, or bloody stools were present to indicate an underlying malignancy or inflammatory bowel disorder needing structural assessment. An IgA endomysial antibody test (C), along with an IgA tissue transglutaminase antibody test, is considered a very sensitive and specific test to diagnose Celiac disease. However, this patients symptoms relative to dairy consumption made Celiac disease a comparably less-likely causative condition. A qualitative fecal fat assay (D) is valuable in patients suspected of conditions for which steatorrhea is a presenting symptom. Steatorrhea was not observed in this patient's history.

A patient presents with nausea, vomiting, right-sided hemiplegia and non-occipital headache. His gaze is deviated to the left, but he denies loss of sensation. Thirty minutes later, he becomes stuporous and progresses into coma. The pupils are now fixed and dilated. Abnormal posturing is absent. A brain CT scan is ordered. You would expect to find intracerebral hemorrhage in which of the following sites? A.Left pons B.Left putamen C.Right cerebellum D.Right thalamus

Correct Answer ( B ) Explanation: A large putamen hemorrhage results in a rapid progression of hemiplegia, nausea, vomiting and headache over 30 minutes, which is quickly followed by ipsilateral deviation of the eyes, stupor, coma and mydriatic pupils ("blown-pupil", associated with brainstem compression and oculomotor nerve palsy). Acute management includes controlling intracranial pressure and hypertension. Surgical removal of clots is seldom successful, however, some neurologic function may be salvaged in those without coma or those with lobar clots. Pontine hemorrhages (A) are characterized by total paralysis, rapid coma, decerebrate rigidity (abnormal posturing marked by shoulder adduction, elbow extension, wrist pronation and flexion, digit flexion, leg extension and plantarflexion) and small (miotic), not mydriatic, reactive pupils. Cerebellar insults (C) are evidenced by vomiting, occipital headache, inability to stand, vertigo, and eye deviation to the opposite side of the lesion. In this scenario, the eye deviation makes sense, but not the presenting symptoms.Thalamic lesions (D) are marked by complete hemisensory loss. Also common is hemiparesis (bleeding extension into the internal capsule) and aphasia.

Aortic stenosis can lead to which of the following general patterns of left ventricular remodeling? A.Asymmetric left ventricular hypertrophy B.Concentric left ventricular hypertrophy C.Eccentric left ventricular hypertrophy D.Mixed concentric/eccentric left ventricular hypertrophy

Correct Answer ( B ) Explanation: Aortic stenosis increases afterload to the left ventricle resulting in concentric left ventricular hypertrophy. Pressure overload leads to concentric left ventricular remodeling with or without an overall increase in myocardial mass. Concentric remodeling is characterized by increased relative wall thickness or ventricular wall thickness as compared to cavity size. With concentric hypertrophy, cardiac sarcomeres are added in parallel and individual cardiomyocytes grow thicker. This pattern of hypertrophy is adaptive to the increased wall stress of the left ventricle and helps maintain normal systolic function of the left ventricle. However, as severity of stenosis and left ventricular remodeling progresses, abnormal diastolic function typically occurs. This usually precedes systolic dysfunction and results in increased left ventricular filling pressures and symptoms of angina and dyspnea. In contrast, pure regurgitant lesions, such as mitral valve regurgitation cause volume overload to the left ventricle. The remodeling pattern involves eccentric left ventricular hypertrophy; a compensatory mechanism that maintains ventricular compliance. This allows increased left ventricular volume without increased left ventricular filling pressure. With progression of regurgitation and left ventricular dilation, left ventricular contractility or systolic function is impaired. Patients with mixed stenotic and regurgitant lesions such as combined aortic stenosis and regurgitation, may develop concentric and eccentric hypertrophy. In this scenario the dominant lesion determines left ventricular remodeling, clinical presentation, and management. Eccentric left ventricular hypertrophy (C) is characterized by increased cardiac mass and chamber volume and is caused by volume overload or isotonic exercise. Relative wall thickness may be normal, increased, or decreased. With eccentric hypertrophy, sarcomeres are added in series and individual cardiomyocytes grow longer. This pattern of left ventricular remodeling is not seen in aortic stenosis. Mixed concentric/eccentric left ventricular hypertrophy (D) can occur in mixed stenotic and regurgitant lesions or following myocardial infarction. Myocardial infarction causes stretched, infarcted tissue which increases left ventricular volume, leading to combined volume and pressure load on noninfarcted zones. Asymmetric left ventricular hypertrophy (A) is typical of hypertrophic cardiomyopathy and is not associated with aortic stenosis.

A 45-year-old man with a history of paroxysmal atrial fibrillation presents to the ED with acute onset of severe pain and paresthesias in his right calf. On exam, you note lower extremity pallor and an absent dorsalis pedis pulse. Which of the following is the most likely diagnosis? A.Arterial atheroembolism B.Arterial thromboembolism C.Arterial thrombosis D.Arterial vasospasm

Correct Answer ( B ) Explanation: Arterial embolism can be divided into thromboembolic and atheroembolic causes. Most arterial thromboemboli originate in the left side of the heart and are frequently associated with a recent myocardial infarction, atrial fibrillation, or valvular abnormalities. Acute arterial thromboembolism results in the sudden loss of a previously present pulse. In general, patients with arterial thromboembolism have few physical findings suggestive of long-standing peripheral vascular disease and will have normal proximal and contralateral limb pulses. These patients typically do not have well-developed collateral circulation and are at high risk for limb ischemia. Arterial atheroemboli (A) refers to microemboli consisting of cholesterol, calcium, and platelet aggregates dislodged from proximal complicated atherosclerotic plaques. In the peripheral vascular system, atheroemboli characteristically present with cool, painful, and cyanotic toes ("blue-toe" syndrome). Arterial thrombosis (C) is a chronic condition associated with progressive development of complicated atherosclerotic plaques. Peripheral arterial thrombi are usually firmly attached to the damaged arterial wall and rarely embolize. Patients will exhibit signs of longstanding atherosclerosis in their lower extremities (atrophy, loss of hair growth, thickened toe nails) and often have a history of claudication. These patients have well-developed collateral circulation, which helps protect them from limb-threatening ischemia. Arterial vasospasm (D), such as Raynaud's disease, causes a sharp border between ischemic and normal tissue. This condition is characterized by intermittent attacks of triphasic color changes (pallor, cyanosis, rubor).

A 66-year-old man presents after a landscaping injury. He was pruning bushes with motorized shears when he slipped and cut his right thigh. He put a towel on the wound and was able to walk to the Emergency Department. He has no other injuries and his vital signs are within normal limits. He takes warfarin for atrial fibrillation. When you examine his wound, you note a seven-centimeter simple laceration with blood continuously oozing from the defect. Which of the following is the appropriate next step? A.Apply a tourniquet proximal to the wound B.Apply direct pressure to the wound C.Irrigate the wound D.Reverse his anticoagulation

Correct Answer ( B ) Explanation: Bleeding is a leading cause of mortality in patients suffering from trauma. Rapid identification and control of external hemorrhage is part of the primary trauma survey. Even with normal vital signs, a significant volume of blood can be lost. Control of the bleeding can prevent further blood loss. Arterial bleeding is under pressure and will spurt, while venous bleeding will generally be a continuous slow ooze. Direct pressure to the wound is an effective technique to gain early control of a bleeding injury. Pressure compresses the vasculature and gives time for a clot to form at the site of the injury. Applying a tourniquet proximal to the wound (B) is appropriate when a patient is exsanguinating from an extremity wound. In this case, the patient has active slow bleeding and direct pressure is an appropriate, less aggressive first step in bleeding control. Irrigating the wound (C) is an important step in preventing infection and promoting wound recovery, however it is important to control the bleeding first. If the bleeding continues, reversal of his anticoagulation (D) may be necessary. However, reversal is generally not necessary to control a relatively isolated, small wound.

Which of the following best describes the finding seen in the ECG above? A.Atrioventricular block B.Left bundle branch block C.Right bundle branch block D.Sinoatrial block

Correct Answer ( B ) Explanation: Bundle branch blocks are abnormal conduction abnormalities (not rhythm disturbances) in which the ventricles depolarize in sequence, rather than simultaneously, thus producing a wide QRS complex (> 120 msec) and a ST segment with a slope opposite that of the terminal half of the QRS complex. A left bundle branch block is a bifascicular block in which ventricular activation is by way of the right bundle branch. The impulse travels down the right bundle, activating the septum and the free wall of the right ventricle, and then continues on in the same direction to activate the free wall of the left ventricle. Because the dominant forces are traveling in the same direction, there is a tendency toward monophasic QRS complexes. The ECG in a LBBB will show a large wide R wave in lead I and a negative wave (QS or rS) in lead V1. AV block (A) produces PR interval lengthening, not R-R' complexes. The ECG in a RBBB (C) will show a wide S wave in lead I and a RSR' pattern in lead V1. Sinoatrial block (D) is represented electrocardiographically by a missed cycle or beat within otherwise normal appearing P waves and QRS complexes.

A 63-year-old man with a history of a cholecystectomy and appendectomy presents with abdominal cramping, vomiting and decreased bowel movements. Bowel sounds are decreased. Which of the following is true regarding this patient? A.Abdominal X-ray can be used to rule out the diagnosis of small bowel obstruction B.CT scan of the abdomen is highly specific in small bowel obstruction C.Serum lactate is highly sensitive early on in patients with small bowel obstruction D.Serum white blood count is always elevated in small bowel obstruction

Correct Answer ( B ) Explanation: CT scan of the abdomen has rapidly become the imaging modality of choice in diagnosing small bowel obstruction (SBO) as it is both highly sensitive and highly specific. Patients with SBO present with crampy abdominal pain with nausea, vomiting, constipation and abdominal distension. The pain can be generalized and often will come in waves. The rapidity of onset of symptoms depends on how proximal the obstruction is. More proximal obstructions will have a more rapid onset and peak of symptoms. CT scan of the abdomen and pelvis is the diagnostic modality of choice as it has a high sensitivity and specificity and can give additional information including location of obstruction, cause of obstruction and alternative diagnoses. Abdominal X-ray (A) has poor sensitivity and specificity. There are no lab tests (C, D) that aid in diagnosis.

A 27-year-old woman presents to your office with complaints of depression and thoughts of suicide. She is interested in starting both counseling and medication to address her symptoms. Which of the following is the safest medication to consider prescribing? A.Amitriptyline B.Fluoxetine C.Nortriptyline D.Venlafaxine

Correct Answer ( B ) Explanation: Depression and suicidal ideation are common complaints seen in the primary care setting. Any patient who reveals having thoughts of suicide should be assessed further to determine more details about the thoughts of suicide, as well as intent and plan. Risk factors for suicide include psychiatric illness, history of previous suicide attempts, individuals who have never been married, previous or active military service, childhood abuse, family history of suicide, and access to weapons. Women attempt suicide twice as often as men, but men are three times more likely to be successful. Management of a patient who is suicidal includes risk factor reduction, managing the underlying cause, close monitoring, and follow up. Determination of the lethality of the patient's current medication regimen is part of the risk reduction process. Selective serotonin reuptake inhibitors (SSRIs) seem to be safer in the case of an overdose than other agents. SSRIs, such as fluoxetine, are therefore the agents of choice in the treatment of depression for patients who are potentially suicidal. Tricyclic antidepressants, such as amitriptyline (A) and nortriptyline (C), are lethal if taken in high doses and their use should be avoided in patients who are suicidal. Venlafaxine (D) is a serotonin norepinephrine reuptake inhibitor that is dangerous in overdose and should also be avoided in patients deemed a high risk for suicide.

Which of the following will help to classify heart failure as being systolic or diastolic? A.Afterload B.Ejection fraction C.Heart rate D.Preload

Correct Answer ( B ) Explanation: Ejection fraction is the percentage of blood that is ejected from the ventricle during systole. A normal ejection fraction is 55% or greater. Systolic dysfunction typically results from ischemic heart disease and myocardial cell death. It results in impaired contractility with an ejection fraction < 40%. Cardiac output is dependent on resistance (afterload) to emptying the ventricle. Diastolic dysfunction typically results from chronic hypertension and left ventricular hypertrophy. It results in impaired relaxation and ventricular filling with a normal ejection fraction. Output is dependent on ventricular filling (preload). Afterload (A) is the force needed to overcome both the volume of blood in the ventricle and the peripheral vascular resistance during ventricular contraction. The afterload is not specific for different types of heart failure. Heart rate (C) can also be variable in different types of heart failure. Preload (D) is the force or volume stretching the myocytes before contraction. It also can be thought of as the volume in the ventricle at the end of diastole just before the ventricle contracts. Output is dependent on preload in diastolic dysfunction, but it poorly differentiates diastolic from systolic heart failure.

Which of the following sources of fluoride has the highest risk of causing dental fluorosis? A.Community water fluoridation with 0.7 ppm fluoride B.Fluoride drops and tablets C.Fluoride gel and varnish D.Tooth brushing with 1000 ppm fluoride toothpaste

Correct Answer ( B ) Explanation: Fluoride is highly beneficial in primary and secondary prevention of dental decay. It is incorporated into tooth enamel and also helps incorporate calcium and phosphate into enamel. While insufficient fluoride intake can increase the risk of dental caries, too much fluoride can result in fluorosis. Fluorosis can result from systemic fluoride consumption >0.05 mg/kg/day during enamel formation. This high fluoride consumption can be caused by residing in an area of high fluoride content in the drinking water (>2.0 ppm), swallowing excessive fluoridated toothpaste, or inappropriate fluoride prescriptions. Excessive fluoride during enamel formation affects ameloblastic function, resulting in inconspicuous white, lacy patches on the enamel to severe brownish discoloration and hypoplasia. Therefore, topical fluoride through low levels in water and toothpaste is favored to systemic fluoride through drops and tablets. Community water fluoridation is considered a highly successful public health achievement. The US Department of Health and Human Services recently updated their optimal fluoride concentration recommendation from 1 ppm to 0.7 ppm (A) to account for the fluoride exposure from other sources. Fluorosis can occur at levels >2.0 ppm. Fluoride gel and varnish (C) are very effective in preventing caries as they leave a fluoride-calcium compound on tooth enamel that releases fluoride when the pH decreases from plaque. Varnish is preferred over gel as it adheres better to the teeth and requires little training to be applied. Starting in the first year of life, fluoride varnish should be applied to the teeth of children twice yearly. Toothbrushing with fluoridated (1000 ppm) toothpaste (D) twice a day is recommended for proper hygiene. Fluoride remains in the saliva after brushing at low concentrations for two to six hours. Fluoride levels less than 1000 ppm in toothpaste do not consistently prevent caries.

Which of the following describes a grade 3 ankle sprain? A.Complete ligamentous rupture with concomitant distal fibular fracture B.Complete ligamentous rupture with considerable swelling, pain, and significant laxity C.Partial tear with mild laxity and moderate pain, tenderness, and instability D.Partial tear without laxity and only mild edema

Correct Answer ( B ) Explanation: Grade 3 sprain is a complete rupture resulting in considerable swelling, increased pain, significant laxity, and often an unstable joint. Ankle sprain involves stretching or tearing of the ligaments of the ankle. There are three grades of ankle sprain as determined by the extent of ligamentous injury. This injury is a common cause of morbidity in the general population, and the ankle is the most commonly injured joint complex among athletes. Patients who play sports experience approximately one ankle sprain for every 1000 person-days of competition. It is estimated that more than 23,000 ankle sprains require medical care in the United States per day. Eighty-five percent of all ankle sprains occur on the lateral aspect of the ankle, involving the anterior talofibular ligament and calcaneofibular ligament. There are only 3 grades of ankle sprains and complete ligamentous rupture with concomitant distal fibular fracture (A) would be a fracture and not a sprain. Grade 2 ankle sprain (C) is a partial tear with mild laxity and moderate pain, tenderness, and instability. Grade 1 (D) is a partial tear without laxity and only mild edema.

A previously healthy 32-year-old man presents to your office with a complaint of tingling in both feet. He says that the tingling sensation started 3 days ago and now his lower legs feel weak and he is having difficulty walking. In addition, he is having trouble chewing his food. Physical exam shows absent patellar and Achilles reflexes. Which of the following is the most likely diagnosis? A.Diabetic neuropathy B.Guillain-Barré syndrome C.Multiple sclerosis D.Myasthenia Gravis

Correct Answer ( B ) Explanation: Guillain-Barré syndrome is an autoimmune disorder that affects the peripheral nervous system. The classic presentation involves symmetric, progressive muscle weakness along with weak or absent deep tendon reflexes. Facial weakness occurs in more than 50% of individuals. Diagnosis is generally based on clinical signs and symptoms, but may be confirmed by cerebrospinal fluid analysis which will reveal elevated protein with a normal white blood cell count. Treatment includes supportive care, admission to the ICU for monitoring of respiratory and cardiac function, plasma exchange and IVIG. Diabetic neuropathy (A) is one of the most common complications of diabetes and affects the peripheral nervous system in a symmetric, distal to proximal pattern. However, individuals do not lose their deep tendon reflexes. Multiple sclerosis (C) is an autoimmune inflammatory disease involving the central nervous system. Women are more commonly affected than men. Symptoms include vision changes and transient, intermittent muscle weakness. Myasthenia gravis (D) is an autoimmune disorder in which acetylcholine receptors are blocked by antibodies. It is classically described as presenting with fluctuating muscle weakness, ptosis and diplopia. Symptoms are usually worse later in the day.

Which of the following clinical scenarios can be defined as a hypertensive emergency? A.A 25-year-old pregnant woman in her second trimester with a blood pressure of 155/100 mm Hg with a normal urinalysis B.A 55-year-old man with a blood pressure of 185/90 mm Hg whose creatinine has increased from 1.0 to 2.5 mg/dL within 36 hours C.A 59-year-old asymptomatic man requesting a medication refill and is found to have a blood pressure of 210/110 mm Hg and an ECG consistent with left ventricular hypertrophy (LVH) D.A 63-year-old woman with a history of poorly controlled hypertension who presents with a finger laceration and is noted to have a blood pressure of 200/105 mm Hg

Correct Answer ( B ) Explanation: Hypertensive emergency is generally defined as a markedly elevated blood pressure in the setting of acute end-organ damage of the cardiovascular, neurologic, or renal organ system. This condition is a true medical emergency and warrants early reduction of blood pressure (preferably within one hour of identification of the condition) with titratable intravenous medications. It is important to understand, however, that an elevated blood pressure in response to an acute condition is often physiologic; aggressive lowering of the pressure in these conditions (e.g., ischemic stroke) may actually increase morbidity and mortality. Renal failure can be seen as both a consequence and cause of hypertension. Uncontrolled hypertension may cause acute kidney injury and can accelerate the progression of injury in patients with chronic renal failure. Acute worsening of kidney function as seen in this patient whose creatinine increased acutely from 1 to 2.5 mg/dL—in the setting of elevated blood pressure—should be considered a hypertensive emergency and warrants immediate treatment. Hypertension is one of the most common complications in pregnancy. Hypertension in pregnancy is defined as a systolic pressure > 140 mm Hg or a diastolic pressure > 90 mm Hg. Severe hypertension is classified as a systolic pressure > 160 mm Hg or diastolic > 105 mm Hg. Pre-eclampsia is defined as hypertension occurring after 20 weeks gestation with proteinuria or any signs or symptoms of end-organ damage (e.g., elevated LFTs) and should be considered a hypertensive emergency. Hypertension occurring after 20 weeks gestation without these signs or symptoms (A) is termed gestational hypertension. Patients who are hypertensive, but asymptomatic, and show no evidence of acute end-organ damage (C and D) in most cases, do not need acute lowering of their blood pressure. Patients with chronic hypertension may have an altered autoregulatory range; rapid normalization of their elevated blood pressure may in fact lead to hypoperfusion and ischemia. In response to chronic hypertension, the heart is remodeled in a cycle starting with increased wall stress that leads to hypertrophy and impaired diastolic function. When LVH is present in the setting of hypertension, early follow-up should be arranged because such patients are at increased risk for MI, heart failure, stroke, and sudden death.

You diagnose lactation mastitis in a postpartum 17-year-old woman. She is currently breast feeding her healthy newborn. She has no allergies. In addition to local ice packs and ibuprofen, which of the following is the most appropriate treatment? A.Aztreonam B.Dicloxacillin C.Metronidazole D.Penicillin

Correct Answer ( B ) Explanation: In lactating women, mastitis typically presents within a few weeks postpartum, and occurs in 2-10% of breastfeeding women. The infection is almost always unilateral. The most common causative agent is Staphylococcus aureus, which actually originates mainly from the newborn's pharynx. The antibiotic of choice for non-severe disease is a penicillinase resistant agent, such as dicloxacillin or cephalexin. If the patient has beta lactam sensitivity, clindamycin is recommended. If there is concern for maternal methicillin-resistant staphylococcus aureus (MRSA) colonization, trimethoprim-sulfamethoxazole or clindamycin is recommended. If the patient is unstable, inpatient intravenous vancomycin should be initiated after local and blood cultures are obtained. Aztreonam (A) and Penicillin V (D) are antibiotics that do not properly cover staphylococcal infections. Although metronidazole (C) is a common treatment of many gynecologic conditions, its cidal properties are ineffective against Staphylococcus aureus. However, it should be administered if mastitis is associated with subareolar abscess and nipple retraction.

In which of the following patients is a zoster vaccination indicated? A.A 26-year-old pregnant woman B.A 61-year-old man who recalls having chicken pox as a child C.A 7-year-old boy with asthma D.A 75-year-old woman with chronic lymphocytic leukemia

Correct Answer ( B ) Explanation: No additional testing is indicated in this patient. She most likely has a benign systolic ejection murmur. This is a common clinical situation and reassurance is appropriate. A systolic murmur is present in up to 60 % of patients, with 90 % being associated with a normal echocardiogram. A murmur is characterized by its intensity or grade, timing, configuration, frequency or pitch, and location. The most common causes of a mid-systolic murmur are innocent flow murmurs, an increase in flow rate across a normal semilunar valve, and aortic valve sclerosis. Innocent murmurs are typically systolic ejection murmurs located at the base of the heart, short and soft, grade 1/6 to 2/6, without radiation, with a normal S1 and S2, normal cardiac impulse, and no evidence of any hemodynamic abnormality. The benign characteristics of the murmur in this patient include a grade < 3/6, mid-systolic timing, lack of radiation, and the absence of additional abnormal heart sounds. The remainder of the physical examination and the electrocardiogram are normal, without any evidence of cardiac enlargement or dysfunction. Signs of more serious cardiac disease include an S4, grade >3/6 intensity, any diastolic murmur and fixed splitting of S2. Transthoracic echocardiography (D) is the primary test for diagnosis and assessment of valvular heart disease. The appropriateness of this test is guided by history and physical examination. It is indicated in symptomatic patients, in those with a systolic murmur grade 3/6 or greater, diastolic murmurs, continuous murmurs (begins after s1 and beyond s2), holosystolic murmurs, late systolic murmurs, murmurs associated with ejection clicks, or murmurs that radiate to the neck or back. This patient's murmur does not have any of these characteristics. Transesophageal echocardiography (C) may be useful in patients with poor imaging by transthoracic study or to evaluate the feasibility of surgical repair when surgery is planned. Cardiac magnetic resonance imaging (A) is indicated if both transthoracic and transesophageal echocardiograms are equivocal in a more suspicious murmur. It is good for the assessment of the aorta and cardiac chamber size and function.

You are caring for a patient with rheumatoid arthritis. A rheumatologist refills this patient's medications, but does not order the proper laboratory testing used to monitor for adverse side effects. You decide to begin such monitoring. Which of the following is most commonly serially monitored in patients on disease-modifying antirheumatic drugs (DMARDs)? A.Chest radiograph B.Complete blood count C.Rheumatoid factor D.Thyroid function test

Correct Answer ( B ) Explanation: Rheumatoid arthritis (RA) is treated with a variety of medications, from salicylates to disease-modifying anti-rheumatic drugs (DMARDs). Several, if not all, of these medications carry significant side effects and toxicities. As such, routine monitoring of potential side effects or pathology is recommended and necessary. Common side effects of these medications include nausea, vomiting, diarrhea and rash. However, one of the most common DMARD adverse events is bone marrow suppression, especially with sulfasalazine, methotrexate, gold preparations, leflunomide, cyclosporine and azathioprine. Therefore, it is important to monitor marrow health with serial complete blood counts. Hepatitis and hepatic toxicity can occur with chronic methotrexate and leflunomide. As such, clinicians should obtain liver function tests at baseline and periodically throughout treatment. Leflunomide and cyclosporine are associated with hypertension. As such, routine monitoring of blood pressure is important in patients on these medications. Gold preparations are associated with significant kidney damage, namely proteinuria. It is recommended to check serial urinalyses, even on a weekly basis, in patients taking gold sodium thiomalate. Checking a PPD test for tuberculosis is also recommended. Repeated chest X-rays (A) are not routinely recommended in monitoring the effects of DMARD medications. Rheumatoid factor (C) is not a very sensitive or specific marker of rheumatoid arthritis. However, a screen for this antibody is routinely performed, but commonly negative (especially in early disease), to help secure a diagnosis of RA, not monitor for DMARD side effects. Eventually, nearly 70-80% of patients with RA will have a positive RF. DMARDs are not commonly associated with thyroid disease (D).

A 60-year-old man presents to the office with a history of osteoporosis. Which of the following statements is correct as it relates to men with osteoporosis? A.Anorexia is a common finding B.Back pain with vertebral compression is the most common presenting complaint C.Gonadotropin releasing hormone analogues for prostate cancer lessens the chances of developing osteoporosis D.Secondary causes includes hyperthyroidism and coronary artery disease

Correct Answer ( B ) Explanation: The clinical presentation of osteoporosis in men is often different from that in women, especially in relation to the time to diagnosis after symptoms begin. Back pain with vertebral compression is the most common presenting complaint. Bone density measurements are less frequently obtained in men, but they are performed after symptoms occur, a far different situation than in postmenopausal women. Lack of androgens results in a skeletal deficit. Peak bone mass is clearly reduced in androgen-insufficient young male patients whether the condition results from idiopathic hypogonadotropic hypogonadism, Klinefelter's syndrome, or constitutional delayed puberty. Anorexia (A) in male patients is extremely rare, and the athlete's triad so characteristic of women runners (i.e., exercise, hypogonadism, and low bone mass) is infrequent in men. The use of long-acting gonadotropin-releasing hormone analogues (C) that block androgen production and are administered in the treatment of prostate cancer is also associated with significant bone loss and fractures in later life. Secondary causes (D) of osteoporosis dominate this disease in men. Hypogonadism and hypercortisolemia are the major etiologic factors in male osteoporosis and must be considered regardless of the phenotypic presentation. However, certain other conditions, including gluten enteropathy, gastric resection or bypass, and ethanol abuse, are more common in men than women with osteoporosis.

A 26-year-old woman is brought in by her husband for evaluation. He states that over the last week, she has exhibited a number of concerning symptoms including tremors and seizures. On entering the room, the patient is noted to be resting comfortably. When asked about the tremor, the patient begins to exhibit coarse diffuse tremors. During the evaluation the patient has generalized shaking but is able to respond to questions. Further discussion with the patient's husband reveals that they are currently going through a separation and there has been a lot of stress at home. Which of the following disorders accounts for the patient's symptoms? A.Conversion disorder B.Factitious disorder C.Hypochondriasis D.Somatization

Correct Answer ( B ) Explanation: This patient exhibits signs and symptoms of factitious disorder. Factitious disorder is characterized by falsified general medical or psychiatric symptoms. Patients deceptively misrepresent, simulate, or cause symptoms of an illness or injury in themselves, even in the absence of obvious external rewards such as financial gain, housing, or medications. Symptoms may develop after an identifiable psychosocial stress or as part of a pattern of general life (i.e. this is the way the patient deals with life events). Symptoms can be both psychological and physical. If the patient admits to producing the symptoms, they would not be included in the factitious category. This patient presents with volitional tremors and pseudoseizures that are characterized by general shaking but preserved cognitive function. Conversion disorder (A) describes the presence of a single unexplained symptom affecting voluntary or sensory function suggestive of a neurologic or medical condition. Patients are typically oblivious to or express no concern about the symptom. Hypochondriasis (C) is a preoccupation with having a serious medical condition. Somatization (D) is the unexplained presence of physical symptoms (multiple) that result in seeking treatment and significant social impairment.

A 35-year-old woman presents for several days of anterior neck pain radiating to her ears, accompanied by dysphagia and "restlessness." She endorses a low-grade fever and fatigue that is "lingering" from her recent flu-like illness. Physical exam shows a tender, symmetrically enlarged thyroid. Laboratory studies show a normal complete blood count, suppressed thyroid stimulating hormone, low antithyroid antibody titers, and a high erythrocyte sedimentation rate. In addition to propranolol, the most appropriate treatment plan includes which of the following medications? A. Ampicillin B. Aspirin C. Levothyroxine D. Methimazole

Correct Answer ( B ) Explanation: This patient has subacute thyroiditis which is treated with oral propranolol to manage her symptoms of hyperthyroidism and aspirin to manage pain. Subacute thyroiditis, or deQuervain thyroiditis, is a common, transient condition that usually presents after a recent viral illness, such as an upper respiratory infection. Young and middle-aged women are the typical population. Symptoms include an acute, painful enlargement of the thyroid gland with dysphagia, fever, and fatigue. Pain often radiates to the jaw or ears. Approximately 50% of patients will develop symptoms of thyrotoxicosis, including palpitations, diaphoresis, vomiting or diarrhea, and a high fever. Laboratory findings will include a suppressed TSH, elevated free T4 levels, and markedly elevated erythrocyte sedimentation rate. Symptoms of hyperthyroidism can remain for several weeks and may be followed by a 4-6 month period of hypothyroidism. Normal thyroid function generally restores within 12 months. Long-term thyroid replacement medication is required in 5% of patients. Ampicillin (A) given parenterally may be an appropriate choice in suppurative thyroiditis. However, suppurative thyroiditis is distinguished from sub-acute thyroiditis by an elevated leukocyte count, which is not present in this patient. Levothyroxine (C) is the necessary treatment for hypothyroidism, not hyperthyroidism. Subacute thyroiditis usually allows of return of normal thyroid function within 12 months and levothyroxine is rarely needed. Methimazole (D) is a treatment option for hyperthyroidism, such as that caused by Graves disease. However, the hyperthyroidism of subacute thyroiditis will typically resolve spontaneously in 4-6 weeks; a beta-blocker is usually sufficient to manage these temporary hyperthyroidism symptoms.

Which of the following disorders causes a normal anion gap metabolic acidosis? A. Cyanide exposure B. Diabetic ketoacidosis C. Diarrhea D. Salicylate ingestion

Correct Answer ( C ) Diarrhea is a common cause of normal anion gap metabolic acidosis. Metabolic acidosis is defined as a reduced serum bicarbonate concentration. Normal anion gap metabolic acidosis is thought to be less immediately dangerous than anion gap metabolic acidosis. Normal anion gap metabolic acidosis can be caused by a variety of conditions including rapid infusion of 0.9% saline, renal tubular acidosis, ingestion of acetazolamide and calcium chloride and hypoaldosteronism. Cyanide (A), diabetic ketoacidosis (B) and salicylate ingestion (D) are all causes of increased anion gap metabolic acidosis.

A six-year-old girl presents with a rash on her face, which she has had for the past seven days. It consists of erythematous plaques with yellow crusts as shown above. Further physical examination is unremarkable, and vital signs are stable. Which of the following is the most appropriate treatment? A. Oral erythromycin B. Topical hydrocortisone C. Topical mupirocin D. Topical terbinafine

Correct Answer ( C ) Impetigo is a rash characterized by erythematous plaques with honey-colored crusts. It is most commonly caused by Staphylococcus aureus and group A Streptococcus. Risk factors include poor hygiene, malnutrition, and atopic dermatitis. Treatment of uncomplicated facial impetigo consists of topical mupirocin. Diffuse impetigo can be treated with oral erythromycin or oral cephalexin. Oral erythromycin (A) is reserved for diffuse impetigo. As impetigo is the result of a bacterial infection, topical hydrocortisone (B) and topical terbinafine (D) have no role in treatment.

A 10-year-old boy presents with a two day history of sore throat, fever and headache. He denies cough, significant rhinnorhea or head congestion. Physical exam is remarkable for enlarged, erythematous tonsils with a pharyngeal whitish exudate. He has marked lymphadenopathy over his anterior cervical lymphoid chain. What is the most likely diagnosis? A. Mononucleosis B. Sinusitis C. Strep throat D. Tonsillolithiasis

Correct Answer ( C ) Explanation: "Strep throat" caused by Group A Beta-hemolytic streptococcus (Streptococcus pyogenes or GAS) is a common etiology of acute pharyngitis especially in children ages 5 to 15. It is characterized by inflammation of the pharynx or tonsils (tonsillar exudates) associated with symptoms of fever, malaise and sore throat, as well as the absence of other URI symptoms such as nasal congestion and cough. Cervical lymphadenopathy is often found on exam, as is a whitish exudate over the pharynx and tonsils. A rapid streptococcal antigen test is recommended in order to determine if treatment with antibiotics is warranted, as other conditions which do not require antibiotic treatment may mimic streptococcal pharyngitis. A throat culture to rule-out GAS infection is recommended in children if rapid antigen testing is negative (90% sensitivity), in order to limit transmission and prevent complications such as rheumatic fever. Other complications of strep throat may include acute glomerulonephritis, peritonsillar abscess, bacteremia, sinusitis and pneumonia. Penicillin-based antibiotics (benzathine penicillin IM or oral penicillin VK) are the treatment of choices. For penicillin allergic patients, azithromycin is an alternative. Mononucleosis (A) is an infection caused by Epstein-Barr virus and is most common in 15 to 24 year-olds. Patients usually experience a several day prodrome of fever, chills, malaise and anorexia followed by the onset of throat pain, fever and lymphadenopathy. The diagnosis is supported by heterophile (Monospot) antibody testing. Treatment is supportive and the illness is generally self-limited, though some patients may experience residual symptoms of malaise and fatigue for months following initial diagnosis. Sinusitis (B) refers to inflammation of the mucous membranes lining the paranasal sinuses. Patients present complaining of cough, facial tightness, headache, reduced ability to smell and nasal congestion with purulent nasal and postnasal discharge lasting 7-10 days. Sinusitis is usually caused by a virus, leading to decreased clearance of secretions within the mucosa and entrapment of bacteria which may lead to secondary bacterial infection. The three most common bacterial agents involved in sinusitis are Haemophilus influenzae, Streptococcus pneumoniae and Moraxella catarrhalis. Treatment of viral sinusitis is supportive; antibiotics are indicated only if the clinical course strongly suggests bacterial secondary infection. Tonsillolithiasis (D) is a condition caused by chronic inflammation of the tonsils in which small concretions develop within one or both of the tonsils. Symptoms of this condition may include halitosis, foreign body sensation, dysphagia and odynophagia, otalgia, and neck pain, though many patients are asymptomatic. Definitive treatment is surgical removal, though not required if symptoms are not bothersome

Which of the following medications is considered first line therapy for absence seizure? A. Carbamazepine B. Diazepam C. EthosuximideCorrect Answer D. Phenytoin

Correct Answer ( C ) Explanation: Absence seizures are sometimes referred to as petit mal seizures. The hallmark of absence seizures is an abrupt and sudden-onset impairment of consciousness, interruption of ongoing activities, a blank stare, possibly a brief upward rotation of the eyes. If the patient is speaking, speech is slowed or interrupted; if walking, he or she stands transfixed; if eating, the food will stop on its way to the mouth. Usually, the patient will be unresponsive when addressed. In some cases, attacks are aborted when the patient is called. The attack lasts from a few seconds to half a minute, and evaporates as rapidly as it commenced. Absence seizures generally are not followed by a post-ictal state. This is in contrast to the majority of seizure disorders. Electroencephalography (EEG) shows generalized spike-and-slow wave discharges. The traditional treatment of absence seizures is with ethosuximide. An alternative agent is valproic acid. Carbamazepine (A) is contraindicated in the treatment of absence seizures as it can induce seizure activity. Diazepam (B) is a benzodiazepine that is used in the acute management of generalized tonic-clonic seizures. Phenytoin (D) is used in the chronic management of generalized seizures.

A 42-year-old man presents to the Emergency Department with nausea, vomiting, and right upper quadrant abdominal pain. He drinks alcohol daily. Which of the following laboratory results would be most consistent with alcoholic hepatitis? A. Alkaline phosphatase 350 U/L B. Aspartate transaminase 1000 U/L and alanine transaminase 1200 U/L C. Aspartate transaminase 250 U/L and alanine transaminase 120 U/LCorrect Answer D. Mean corpuscular volume 60 f

Correct Answer ( C ) Explanation: Acute hepatitis can be the result of an infectious process (most commonly viral), toxic injury, or alcohol. Alcoholic hepatitis can range from subclinical disease to acute liver failure. Patients present with nausea, vomiting, and RUQ abdominal pain. On examination, they frequently have a tender, enlarged liver, and possibly jaundice. Laboratory studies include a macrocytic anemia and thrombocytopenia. The WBC count is often elevated as is the prothrombin time and bilirubin. Liver transaminases, alanine transaminase (ALT) and aspartate transaminase (AST) are typically elevated 2-10 times normal. Unlike hepatitis due to other causes, AST is predominantly elevated, often with a AST:ALT ratio of 2:1. While patients with alcoholic hepatitis may have elevations of alkaline phosphatase, levels over three to four times normal (alkaline phosphatase 350 U/L) (A) are more typically seen with obstructive etiologies. Serum transaminases over ten times normal (aspartate transaminase 1000 U/L and alanine transaminase 1200 U/L) (B) are unusual in alcoholic hepatitis and would point to a toxic or viral etiology. A mean corpuscular volume of 60 mL (D) would indicate a microcytic anemia. Chronic alcoholics will have a macrocytic anemia.

A businessman, who frequently travels in airplanes, complains of painful ear popping every time he flies. He has tried yawning, swallowing, chewing gum and pinching his nose, but nothing seems to relieve the pain. He has even tried using ibuprofen two hours prior to flight. Which of the following would you recommend? A.Myringotomy B.Nifedipine C.Oxymetazoline D.Ranitidine

Correct Answer ( C ) Explanation: Barotitis media, barotrauma, or ear popping, are all names for eustachian tube dysfunction, a condition that occurs when the tube does not open properly during swallowing or yawning. This tube, connecting the middle ear to the pharynx, equalizes pressure inside the ear to the atmospheric pressure. Blockage of the tube can arise from congenital stenosis, abnormal peristaltic function, abnormal ciliary function, adenoid hypertrophy, nasal congestion and tumors. Risk factors include ear or sinus infections, allergies and rapid altitude changes. Symptoms include ear fullness, otalgia, tinnitus, hearing impairment and vertigo. Diagnosis is mainly clinical, but further evaluation may involve an audiogram, tympanogam or otolaryngologist referral. Treatment includes chewing gum, multiple swallows and exhalation through closed nostrils. If refractory, medications may be necessary, and include analgesics, oral antihistamines and nasal decongestants or steroids. Oxymetazoline is an over-the-counter nasal spray decongestant.

A woman suffers from an acute attack of vertigo, nausea, and vomiting. You suspect viral labyrinthitis. Which of the following medications is the best choice in treating the vertigo? A.Acyclovir B.Phentermine C.Prednisolone D.Prochlorperazine

Correct Answer ( C ) Explanation: Commonly a complication of otitis media, upper respiratory infection or meningitis, labyrinthitis is an abnormality of the inner ear vestibular system, which consists of three semicircular canals, vestibule, utricle and sacule. When the vestibular system is damaged, injured or inflamed, balance deficits and vertigo arise. However, since an anatomic connection between the vestibular system and cochlea exist, tinnitus and hearing impairment also occur. The classic clinical picture of labyrinthitis is acute, profound, incapacitating vertigo, with nausea, vomiting and nystagmus. Tinnitus and hearing impairment usually accompany the onset of vertigo. These symptoms resemble those of Meniere's disease, however in Meniere's disease, symptoms are intermittent and possibly less severe. Furthermore, the pathology is different between the two conditions. Labyrinthitis is mainly due to inflammation, while an excess of inner ear fluid causes Meniere's disease. Treatment of suspected viral labyrinthitis includes bed rest, hydration, antiemetics and vestibular depressants. Treatment with corticosteroids during the acute period of vertigo has been shown to improve the recovery of peripheral vestibular function in patients with acute labyrinthitis. Benzodiazepines are also effective. Currently, the role of antiviral therapy, like acyclovir (A), in the treatment of labyrinthitis is not established. Studies to date do not show an improvement over placebo. Phentermine (B) is used as an appetite suppressant and plays no role in the treatment of labyrinthitis. Prochlorperazine (D), an antiemetic, is commonly used in the treatment of acute viral labyrinthitis. Its dopamine-antagonism and anticholinergic effects help relieve nausea and vomiting, but do not diminish the distressing sensation of vertigo.

A 26-year-old woman presents for her annual exam and is inquiring about birth control. She is current on her immunizations and her last pap smear was 2 years ago. She is in a monogamous relationship with her boyfriend and does not have any immediate plans for pregnancy. She has a history of migraines with aura but does not require any prescription medication. Which of the following birth control methods is the best option for her? A. Behavioral methods such as the withdrawal method and periodic abstinence B. Etonogestrel/ethinyl estradiol vaginal C. Medroxyprogesterone acetate D. Norelgestromin/ethinyl estradiol transdermal

Correct Answer ( C ) Explanation: Contraception counseling should be routinely performed in all women of child-bearing age at every annual visit. There are many options such as behavioral, barrier and pharmacological methods. However, pharmacological therapy has the highest rate of pregnancy prevention and should be recommended in all women, unless there are contraindications. This patient has a history of migraines with aura which is considered a contraindication to estrogen use. Other contraindications to estrogen are history of deep vein thrombosis, breast cancer within the past 5 years, cigarette smoking in women more than 35 years of age who smoke more than 15 cigarettes per day, ischemic heart disease, stroke, active liver disease, major surgery with prolonged immobilization and poorly controlled hypertension. In these cases, progesterone only therapy (medroxyprogesterone) should be used in the form of the intrauterine device, the injection or the subcutaneous implantation in the arm.

A family brings in their 3-month-old infant who was just adopted from Honduras. They are not aware of the infant's birth history and were told that the child was healthy and lived in an orphanage for the first three months of life. They are concerned that he has significant bruising over his back. On examination, a large blue patch is present over the sacrum. Similar patches are present over the bilateral shoulders. The remainder of the examination is benign. Which of the following is the most likely diagnosis? A.Child abuse B.Coagulopathy C.Dermal melanosis D.Nevus of Ota

Correct Answer ( C ) Explanation: Dermal melanosis, also known as a Mongolian spot, represents a benign persistence of dermal melanocytes in neonates. It presents as large blue patches that are most commonly located over the sacrum and shoulders. Rarely, the face and flexor surfaces of the extremities may be involved. It is more common amongst all non-Caucasian races. The natural history of dermal melanosis is a gradual fade during the first two years of life. Most lesions resolve completely during grade school. Extra-sacral locations are more likely to persist into adulthood. Persistent lesions remain benign, and no treatment is indicated. Child abuse (A) should be suspected in any infant with significant bruising in unusual locations, as well as in any child who does not walk ("those who don't cruise, don't bruise.") However, the distribution and description of this child's blue patches are consistent with dermal melanosis rather than child abuse. Coagulopathy (B) is a consideration in any child with unusual ecchymoses, purpura, petechiae, or bleeding. However, this infant's bluish discoloration over the sacrum and shoulders are consistent with dermal melanosis rather than true bruising. Nevus of Ota (D) is a rare dermal melanocytosis that preferentially involves the distribution of the first two trigeminal nerves. In contrast to dermal melanosis, its appearance is speckled or mottled and may be grey or blue-black in color. Underlying CNS melanosis or ocular melanoma may also occur, necessitating yearly ophthalmologic examination.

Which of the following antibiotics is associated with spontaneous tendon rupture? A.Amoxicillin B.Doxycycline C.Levofloxacin D.Sulfamethoxazole

Correct Answer ( C ) Explanation: Fluoroquinolone drugs, including levofloxacin, have been associated with spontaneous tendon ruptures. Fluroquinolones are a commonly prescribed antibiotic class. The class includes ciprofloxacin, moxifloxacin and levofloxacin. The fluoroquinolone class of drugs has a number of side effects, the most serious of which are prolongation of the QTc and spontaneous tendon rupture. Tendon rupture appears to be more common in older patients. The overall risk is between 0.1 - 0.4%. These drugs are discouraged for use in pregnant women and children secondary to their effect on cartilage. Amoxicillin (A), doxycycline (B) and sulfamethoxazole (D) are not associated with an increased rate of spontaneous tendon rupture.

A 66-year-old woman has been on estrogen-replacement therapy for 9 years. Her social history is significant for 20 years of alcohol abuse and 30 years of tobacco use. Her family history is positive for familial adenomatous polyposis. Which of the following organs is most likely to undergo malignant transformation due to this patient's alcohol abuse? A.Breast B.Colorectal C.Liver D.Lung

Correct Answer ( C ) Explanation: Hepatocellular carcinoma, also known as malignant hepatoma, is a primary cancer of the liver. It is rare in the United States, where most of liver cancer is secondary from a metastatic site. However, in countries where hepatitis is endemic, such as China, this cancer represents the most common of all liver cancers. The most common causes are viral hepatitis (hepatitis B and C) and alcoholic cirrhosis. Other risk factors include aflatoxin exposure, hemochromatosis, Wilson's disease and type 2 diabetes mellitus. Risk of developing this cancer in someone infected with hepatitis B or C is 20-25%, while those with alcoholic cirrhosis have upwards of an 80% chance of developing hepatocellular carcinoma. Alpha-fetoprotein is elevated in the majority of cases but it is nonspecific. Triple or quad-phase liver CT scan aides in the diagnosis. Biopsy provides a definitive diagnosis. Ultrasound plays an important role in the surveillance of high-risk patients, however, contrast enhanced abdominal CT with triple phase scanning remains the primary diagnostic tool. Treatment options include surgical resection, transplantation, chemotherapy and radiotherapy. Prolonged estrogen-replacement therapy is a major risk factor for breast cancer (A). Up to 25% of patients with colorectal cancer (B) have a family history of familial adenomatous polyposis. Tobacco use is a major risk factor for lung (D), not liver, cancer.

A patient is found to be hyponatremic. Laboratory evaluation reveals low serum osmolality, urine sodium concentration >20 mmol/L and a fractional excretion of sodium (FENa) >1%. He appears to be "fluid overloaded." Which of the following is the most likely cause of this hyponatremia? A.Cirrhosis B.Congestive heart failure C.Hypertensive nephropathy D.Syndrome of inappropriate ADH release (SIADH)

Correct Answer ( C ) Explanation: Hyponatremia is defined as sodium less than 135 mEq/L. Hyponatremia can occur in a hypovolemic, euvolemic, or hypervolemic state. Hypervolemic hypo-osmolar hyponatremia is is associated with fluid overload. The etiology is usually from a perceived low intravascular volume by the kidneys and active water reabsorbtion in excess to sodium retention. If urine sodium is low (<20) causes include liver failure, cirrhosis, hepatorenal syndrome, nephrotic syndrome, and CHF. If urine sodium is high (>20) causes include acute or chronic renal failure, such as that caused by hypertensive nephropathy. Treatment of hypervolemic hypo-osmolar hyponatremia is dialysis. Cirrhosis (A) and congestive heart failure (B) is often the cause of hypervolemic hypo-osmolar hyponatremia when the urine sodium is low (<20). SIADH (D) results in euvolemic hyponatremia with urine osmolality greater than serum osmolality. The excess ADH causes total body water to increase thereby diluting total body sodium. Despite the increased total body water, these patients typically do not show evidence of edema or heart failure as the increased water is intracellular not intravascular.

Which of the following conditions most clearly results in symptomatic hemorrhoids? A.Pancreatitis B.Peptic ulcer disease C.Pregnancyr D.Ulcerative colitis

Correct Answer ( C ) Explanation: Swollen lower rectum vessels are called hemorrhoids. This area contains several vessels which tend to engorge and prolapse, and as such, hemorrhoids are one of the most common causes of any anorectal complaint. However, hemorrhoids are frequently misdiagnosed by many medical practitioners. They do not represent true varicosities, but rather, are comprised of arteries, veins, smooth muscle and connective tissue, all of which are covered by anal canal epithelium. They occur normally in asymptomatic individuals, but are hence called "hemorrhoids" when they cause symptoms. The most likely associated etiology is pregnancy. Other potential etiologies include anorectal varices and poor venous return, which commonly occurs during prolonged sitting on the toilet. Other risk factors include genetic predisposition, colon cancer, inflammatory bowel disease, hepatic disease, rectal surgery, chronic diarrhea and spinal cord injury. Pancreatitis (A) is not a direct risk factor to developing symptomatic hemorrhoids. However, hepatic causes of pancreatitis may be an indirect risk factor for symptomatic hemorrhoids.Peptic ulcer disease (B) is not directly linked to symptomatic hemorrhoids. Although constipation can be seen in cases of ulcerative colitis (D), ulcerative colitis is not directly associated with symptomatic hemorrhoids. In contrast, Crohn disease is more commonly associated with the development of symptomatic hemorrhoids.

A 17-year-old man presents after being thrown a far distance off of a horse. Which of the following is consistent with an anterior cord syndrome? A.Isolated motor function loss B.Loss of all motor and sensory function C.Loss of pain and temperature, loss of motor function D.Upper greater than lower motor weakness

Correct Answer ( C ) Explanation: In order to fully understand the different syndromes of injuries to the spinal cord, it is imperative to understand the location of the tracts of the cord. The posterior columns carry tracts responsible for ipsilateral position and vibratory sensation. The lateral spinothalamic tract carries fibers for contralateral pain and temperature. The lateral corticospinal tract is responsible for ipsilateral motor function. Syndromes may be incomplete depending on how much of the cord is affected by the injury. In the anterior spinal cord syndrome, just the posterior columns are preserved and so patients lose all pain and temperature sensation as well as motor function. Most cases of anterior cord syndrome follow aortic surgery, but it has also been reported in the setting of hypotension, infection, vasospasm, or anterior spinal artery ischemia or infarct. In trauma, typically hyperflexion of the cervical spine causes the injury to the spinal cord. Loss of all motor and sensory function (B) occurs with a complete transection of the spinal cord. Most commonly this occurs after a significant trauma. Isolated motor function loss (A) is not a classic syndrome and would result from a small area of injury on the cord just involving the corticospinal tract. Upper greater than lower motor weakness occurs (D) with a central cord syndrome. Sensory involvement is variable although burning dysesthesias in the upper extremities may occur. Most commonly the syndrome occurs after a fall or motor vehicle accident. Anterior cord may also be known as ventral cord syndrome.

What is the most common sexually transmitted infection in the United States? A.Chlamydia B.Gonorrhea C.Human papillomavirus D.Syphilis

Correct Answer ( C ) Explanation: More than 50% of sexually active individuals will contract human papillomavirus (HPV) at some point in their lives. There are over 100 types of HPV, many of which are asymptomatic and unrecognized. Two high-risk types of the virus, HPV 16 and 18, are known to cause cervical and anogenital cancers in men and women. HPV 6 and 11 are low-risk types that cause genital warts. Two HPV vaccines are available and are recommended for both men and women aged 11-12 years old. HPV2, Cervarix® for women only and HPV4, Gardasil® for both men and women. The vaccinations can also be given to females ages 13-26 years and males ages 13-21 years who were not vaccinated earlier. Of all diseases that are reportable to the Centers for Disease Control, chlamydia (A) is the most common and gonorrhea (B) is the second most common. However, HPV is the overall most common sexually transmitted disease. Both chlamydia and gonorrhea are major causes of pelvic inflammatory disease in women. Syphilis (D) rates are lower than those of chlamydia, gonorrhea and HPV, but have been increasing since 2000. Syphilis infection is considered to be a major health problem among men who have sex with men and there is a high rate of co-infection with HIV in this population.

A 33-year-old woman with no medical problems presents with chest pain and shortness of breath. The symptoms worsened over the previous two days and increase with exertion. The patient denies cough, but last week reports fevers, chills, coughing and myalgias. Her ECG demonstrates sinus tachycardia without ST segment changes. Vital signs are T 100.7°F, HR 120, BP 108/60, RR 16, 100% saturation on room air. Which of the following is the most likely diagnosis? AAcute coronary syndrome BMediastinitis CMyocarditis DPulmonary embolism

Correct Answer ( C ) Explanation: Myocarditis is typically caused by a viral infection, most commonly coxsackie B virus, adenovirus, parvovirus B19 and Echovirus. Trypanasoma cruzi (Chagas disease) is the most common etiology worldwide. Myocardial necrosis occurs likely as a result of direct invasion of the offending organism as well cytotoxic effects of the host's immune system. Individuals develop flulike symptoms and in adults chest pain and shortness of breath. One of the hallmark signs of myocarditis is tachycardia out of proportion to fever. Depending on the time of presentation, patients may have symptoms of heart failure as the left ventricular ejection fraction is impaired as a result of the myocarditis. Troponin is often elevated as the disease progresses. The gold standard for diagnosis is endocardial biopsy. Management is supportive. ACE inhibitors help reduce myocardial inflammation. Acute coronary syndrome (A) is a possibility in this patient but is less likely given her age and absence of medical history. It is also atypical for a person with acute coronary syndrome to have a fever and viral symptoms. Mediastinitis (B) may present with fever, chest pain and shortness of breath. However, there is often a history of either severe retching or a procedure that caused esophageal perforation. Patients with air in the mediastinum from a perforation may have Hamman's sign, a crunching sound on auscultation. Pulmonary embolism (D) does cause chest pain and shortness of breath. Classically the chest pain is pleuritic and flulike symptoms are not associated with the diagnosis. Patients with a pulmonary embolism may have a low-grade fever.

Which of the following clinical findings differentiates rheumatoid arthritis from osteoarthritis? A.Involvement of the proximal interphalangeal (PIP) joints B.Polyarticular involvement C.Presence of constitutional symptoms D.Symmetric joint involvement

Correct Answer ( C ) Explanation: Osteoarthritis (OA) and rheumatoid arthritis (RA) share a number of features but constitutional symptoms are only seen in RA. RA is a chronic inflammatory disease. Patients often present with fever, weakness and musculoskeletal pain lasting for weeks to months. Arthritis involves symmetric joints of the hands, wrists and elbows. Morning stiffness is common and usually last for about an hour. Inflammation may affect other organs leading to hepatitis, scleritis, myocarditis, pericarditis and pleuritis. Atlantoaxial subluxation may also occur and cervical spine precautions should be maintained during intubation. Both RA and OA often involve the PIP joints (A) and can be monoarticular or polyarticular (B). Symmetric joint involvement (D) is common in both RA as well as OA.

A 32-year-old woman complains of a band-like pressure around her forehead that radiates down to the back of her neck. These headaches occur twice a week on average and last for approximately 1 hour in duration. Her neurological exam is within normal limits and she has no other associated symptoms. What is the best initial abortive treatment? A.Amitriptyline B.Caffeine C.Ibuprofen D.Promethazine

Correct Answer ( C ) Explanation: Tension-type headaches cause pain that is mild or moderately intense and is described as tightness, pressure, or a dull ache. The pain is usually experienced as a band extending bilaterally back from the forehead across the sides of the head to the occiput. Patients often report that this tension radiates from the occiput to the posterior neck muscles. In its most extensive form, the pain distribution is "cape like," radiating along the medial and lateral trapezius muscles covering the shoulders. Tension-type headaches can last from 30 minutes to several days and can be continuous in severe cases. In addition to its characteristic distribution and intermittent nature, the history obtained from patients with tension-type headache discloses an absence of signs of any serious underlying condition. Patients with tension-type headache do not typically report any visual disturbance, fever, stiff neck or recent trauma. Treatment goals for patients with tension-type headache should include recommending effective over the counter analgesic agents and discovering and ameliorating any circumstances that may be triggering the headaches. Research confirms that NSAIDs, such as Ibuprofen, and acetaminophen are effective in reducing headache symptoms. Patients with chronic tension-type headache should limit their use of analgesics to two times weekly to prevent the development of chronic daily headache. Repeated use of analgesics, especially ones containing caffeine or butalbital, can lead to "rebound" headaches as each dose wears off and patients then take another round of medication. Common features of chronic daily headache associated with frequent analgesic use are early morning awakening with headache, poor appetite, nausea, restlessness, irritability, memory or concentration problems, and depression. If the patient requires analgesic medication more frequently, adjunctive headache medications can be initiated. Smoking cessation is an important issue to address in patients with chronic tension-type headache. The number of cigarettes smoked has been "significantly related" to the headache index score and to the number of days with headache each week. Analgesics can be augmented with a sedating antihistamine, such as promethazine (D) and diphenhydramine or an antiemetic, such as metoclopramide and prochlorperazine. If this regimen is inadequate, the patient can try acetaminophen or aspirin combined with caffeine (B) and butalbital. A wide variety of prophylactic agents have been researched in the management of chronic tension-type headache. Amitriptyline (A) is the most researched of the prophylactic agents for chronic tension-type headache. It is typically used in doses of 10 to 75 mg, one to two hours before bedtime to minimize grogginess on awakening.

A 40-year-old woman complains of recurrent chest pain that occurs shortly after she wakes up in the morning. She has a history of migraine headaches and Raynaud's phenomenon. She admits to tobacco abuse and smokes 1 pack-per-day. Her chest pain is not reproducible with palpation. An ECG during an episode reveals ST-elevation in multiple leads and cardiac biomarkers are normal. Which of the following is the most likely diagnosis? A.Costochondritis B.Panic disorder C.Prinzmetal angina D.Unstable angina

Correct Answer ( C ) Explanation: Prinzmetal angina, or variant angina, is the most likely diagnosis in this patient. Prinzmetal angina is characterized by spontaneous episodes of angina in association with ST-segment elevation on ECG. The cause is a transient, abrupt, and marked reduction in the luminal diameter of an epicardial coronary artery due to spasm which leads to transient myocardial ischemia. Unlike most other causes of chest pain and ST-segment elevation, the ST-segment returns to baseline rapidly upon resolution of symptoms. This is a diagnosis of exclusion. Patients with variant angina are often younger and exhibit fewer classic cardiovascular risk factors, with the exception of cigarette smoking. Variant angina may be associated with other vasospastic disorders such as Raynaud's phenomenon and migraine headache or its treatment, such as sumitriptan. A history of drug abuse, such as cocaine, may also be present. Costochondritis (A) is musculoskeletal chest wall pain that causes more diffuse pain, in which multiple areas of tenderness are found and can usually be reproduced with palpation. Panic disorder (B) is a particularly common cause of chest pain; however it is not associated with ECG changes and is less consistent with this clinical picture. Unstable angina (D) is a presentation of acute coronary syndrome characterized by new, worsening or persistent chest pain, but in the absence of positive cardiac biomarkers. This can be difficult to distinguish from Prinzmetal angina; however, it would be less likely in a younger patient and it is not associated with other vasospastic disorders such as migraine headaches or Raynaud's phenomenon.

A 72-year-old woman was hospitalized for atrial fibrillation with rapid ventricular rate. She is being discharged today on amiodarone for long term control of her dysrhythmia. Which of the following diagnostic studies should be followed as an outpatient? A. Coagulation studies B. Complete blood counts C. Pulmonary function tests D. Renal panels

Correct Answer ( C ) Explanation: Pulmonary function tests should be monitored for patients on chronic amiodarone therapy. Amiodarone is a class III anti-arrhythmic drug used to treat many common dysrhythmias. Baseline pulmonary function tests and a chest radiograph with annual pulmonary function tests thereafter are recommended. Pulmonary toxicity generally correlates more closely with the total cumulative dose than the serum levels. It usually occurs months to years after initiation. Several types of pulmonary toxicity may result from chronic amiodarone therapy; however, the most common is a chronic interstitial pneumonitis. A non-productive cough and dyspnea are present in the majority of affected individuals. If signs or symptoms develop the patient should discontinue amiodarone immediately. Other complications of chronic amiodarone therapy include, but are not limited to, thyroid dysfunction, both hypo- and hyperthyroidism, symptomatic hepatitis, corneal microdeposits, optic neuropathy, or dermatologic manifestation such as photosensitivity and blue-gray skin discoloration. Thyroid function tests and hepatic panels are recommended every six months. Dermatologic physical exams and ophthalmologic eye evaluations are recommended as needed for signs or symptoms. These complications are treated by either reducing the dose and discontinuation. Coagulation studies (A), complete blood counts (B) and renal panels (D) are not routinely monitored with amiodarone therapy; however, these studies are important for the monitoring of many different drugs. A few examples include coagulation studies for patients on warfarin, complete blood counts for patients on the antipsychotic, clozapine, and renal panels for patients on angiotensin converting enzyme inhibitors. These are just a few examples.

A 16-year-old girl returns from a summer abroad volunteering at an HIV clinic in Zimbabwe. She is going to volunteer at her local hospital, which requires tuberculosis skin testing. Her skin test is read as 14 mm in diameter. Chest X-ray is negative and she is asymptomatic so she is started on a 9-month course of isoniazid. What vitamin should she concurrently be started on? A.Cobalamin B.Niacin C.PyridoxineCorrect Answer D.Thiamine

Correct Answer ( C ) Explanation: Pyridoxine (vitamin B6) supplementation is recommended when taking isoniazid because isoniazid alters the activity of vitamin B6. The tuberculosis skin test is measured 48 to 72 hours after placement. A positive test is induration of 5 mm or more in patients with HIV or other immunocompromising conditions, 10 mm or more in patients younger than age 4, with certain medical conditions (e.g. diabetes, renal failure), in health care workers or in others with high-risk exposure, and 15 mm or more in patients with no risk factors. Patients with positive skin tests with a negative chest X-ray and no symptoms should be started on treatment with isoniazid for latent tuberculosis. Cobalamin (vitamin B12) (A) is important for DNA synthesis and deficiency can lead to pernicious anemia and abnormal neurological symptoms. Niacin (vitamin B3) (B) is used to treat hypercholesterolemia and pellagra. Thiamine (vitamin B1) (D) deficiency can cause neurological symptoms as well as seen in Wernicke's encephalopathy and Korsakoff syndrome.

A 14-year-old boy presents to the Emergency Department for a rash. He reports the abrupt onset of fever, headache, and myalgias three days ago. This morning, he developed a blanching, red, macular rash on his wrists and palms that now involves his extremities and trunk. What is the most likely cause of his symptoms? A.Measles virus B.Neisseria meningitidis C.Rickettsia rickettsii D.Staphylococcus aureus

Correct Answer ( C ) Explanation: Rocky Mountain spotted fever (RMSF) is a febrile, tick-borne illness caused by Rickettsia rickettsii. Patients present with abrupt onset of fever, headache, myalgias, and nausea followed three to five days later with a blanching, macular rash that initially is found on the wrists and ankles before spreading centripetally. The rash later becomes petechial. Despite its name, RMSF is relatively rare in the Rocky Mountain states and is found primarily in the southeastern United States. Carried mostly by the American dog tick (Dermacentor variabilis) and the Mountain wood tick (Dermacentor andersoni), R. rickettsii is an obligate intracellular bacteria that damages endothelial cells. This in turn starts a cascade of reactions that result in widespread vascular lesions that manifest as the clinical features of the disease. Without treatment, mortality is near 25%. Management includes supportive care and doxycycline. The measles virus (A) causes symptoms of cough, coryza, conjunctivitis, fever, and rash. The rash typically starts on the head and spreads down to involve the trunk. Neisseria meningitidis (B) can also cause a fever, headache, and rash. However, the rash associated with N. meningitidis typically spares the palms and soles. Staphylococcus aureus (D) can cause toxic shock syndrome which is characterized by fever, hypotension, and a diffuse erythematous macular rash that later desquamates.

A 28-year-old man presents with scrotal swelling and pain. Examination reveals a left, nontender, swollen spermatic cord that feels like a "bag of worms." The swelling reduces when the patient moves to a supine position. There are no skin lesions, hernia or testicular abnormalities. In addition to scrotal ultrasonography, you may consider ordering further imaging with attention to which of the following organs? A.Ascending colon B.Descending colon C.Kidney D.Urinary bladder

Correct Answer ( C ) Explanation: Scrotal mass and pain are common complaints in the primary care office. Etiologies range from incidental benign conditions to serious cancers to surgical emergencies. Any patient reporting scrotal swelling needs to be evaluated immediately. A careful history and physical examination, including transillumination, is necessary. The evaluation begins with determining which structure is swollen: skin, epididymis, testicle or spermatic cord. Dilation of the pampiniform venous plexus along the spermatic cord is referred to as varicocele. This condition is suspected in any swelling of the spermatic cord, especially when palpation reveals a nontender "bag of worms" consistency. There are primary and secondary types of varicocele. Primary varicocele is usually idiopathic in nature. Secondary varicocele can occur due to abdominal mass compression of the renal veins (more common with right sided varicocele), or superior mesenteric artery compression of the left renal vein (aka "Nutcracker Syndrome"; most commonly due to renal cell carcinoma and retroperitoneal fibrosis/adhesions). Patients with varicocele usually complain of scrotal heaviness and a "dragging" aching pain. It mostly occurs in tall, thin men between the ages of 15 and 30 years, and affects the left side more than the right. The "wormy" mass or swelling typically reduces when the patient is supine. Varicoceles, although not life threatening may cause decreased testicular artery flow and infertility, and therefore should be evaluated further with ultrasonography and possibly abdominal imaging. Colonic (A and B) and bladder (D) masses are unlikely to cause venous compression and subsequent varicocele.

A healthy 6-year-old boy presents to the ED with bloody diarrhea. He was in his usual state of health until one week ago when loose, watery stools (up to 10 per day) were noted. He was seen by his pediatrician four days ago but has since developed increasing amounts of blood and pus in his stools along with a low-grade fever. Mom states there is no recent travel, antibiotic use, or known sick contacts. His vitals are heart rate 118 beats per minute, oxygen saturation 100% on room air, and rectal temperature of 38.3°C. Your physical exam reveals a mildly tender abdomen without localization, rebound, guarding, or peritoneal signs. You note grossly bloody stool on rectal exam. A brief discussion with his pediatrician confirms your suspicion of an invasive bacterial diarrhea; a stool culture was positive for Shigella. Which of the following statements is true regarding this condition? A.Antibiotics should be avoided because this is a severe case and the patient is at highest risk of developing hemolytic uremic syndrome B.Antidiarrheal agents (such as diphenoxylate and atropine) are indicated, given the frequency of loose stools C.Extraintestinal manifestations such as hallucinations, confusion, and seizures may occur D.Oral rehydration should be avoided; IV fluids should be initiated

Correct Answer ( C ) Explanation: Shigella species cause an invasive diarrhea that rarely infects infants younger than three months old and is most common between two and three years of age. Infection is typically transmitted by person-to-person (fecal-oral) contact or through ingestion of contaminated material. Clinical illness varies from mild to severe, with some patients exhibiting abdominal cramps and tenderness. Dysentery (diarrhea with significant blood, pus, and mucus) occurs in approximately 33% of patients. Some patients may also develop extraintestinal manifestations such as reactive arthritis, seizures, and hallucinations. Although antibiotics (A) are generally not indicated for the treatment of mild illness, they should be considered in more severe cases. Hemolytic uremic syndrome (HUS) is a potentially fatal complication that develops when antibiotics are administered to children (but not adults) with enterohemorrhagic E. coli (O157.H7). HUS results from the release of Shigella-like toxin by dying E. coli (not Shigella itself as which occurs from Shiga toxin). For this reason, it is recommended that empiric antibiotic therapy be held in pediatric patients with infectious diarrhea pending culture results. In this case, the causative organism is already known, and antibiotic therapy can be safely administered. Antidiarrheal agents (B) are not recommended because they can worsen bacterial invasion of the bowel wall and prolong the infection and carrier state. If dehydration is a concern, oral rehydration (D) should be attempted prior to initiating IV fluids because the patient is not vomiting and is able to tolerate oral intake.

An 18-year-old girl presents to the ED with left ankle pain. Earlier in the day she was playing softball and slid into second base and "twisted her ankle." On exam, you note moderate swelling, tenderness, and pain with passive range of motion of the ankle. You do note some abnormal motion when stressing the joint. Which of the following is the most likely diagnosis? A.First-degree sprain B.First-degree strain C.Second-degree sprain D.Second-degree strain

Correct Answer ( C ) Explanation: Sprains are classified as ligamentous injuries resulting from an abnormal motion of a joint. In such cases, there is injury to the ligamentous fibers of a supporting joint. Sprains are graded according to the severity of pathologic findings; however, clinically the grades are often indistinct. A second-degree sprain is a partial tear of a ligament (more than first-degree). Clinically, there will be moderate hemorrhage and swelling, tenderness, painful motion, abnormal motion, and loss of function. Although there may be some laxity with stressing of the joint, an absence of end points will be seen only with complete ligament rupture (i.e., third-degree sprains). A first-degree sprain (A) is characterized by minor tearing of ligamentous fibers with mild hemorrhage and swelling. There is minimal point tenderness. Stressing the ligament produces some pain, but there is no opening or abnormal joint motion. A strain is an injury to a musculotendinous unit resulting from violent contraction or excessive forcible stretch. Sometimes the term "pulled muscle" is used interchangeably with muscle strain. A first-degree strain (B) is a minor tearing of the musculotendinous unit, characterized by swelling, local tenderness, and minor loss of function. With a second-degree strain (D), more fibers are torn but without complete disruption. There is also greater swelling, ecchymosis, and loss of strength.

A 72-year-old man presents to the Emergency Department with chest pain. During triage, he collapses and nursing staff cannot feel his pulse. The patient is taken to a treatment room where cardiopulmonary resuscitation is initiated and the above cardiac rhythm is noted. What is the next best step? A.Administration of amiodarone B.Cardioversion C.Defibrillation D.Two minutes of chest compressions

Correct Answer ( C ) Explanation: Sudden cardiac death is a leading cause of mortality. It occurs in a bimodal age distribution: infancy and individuals > 45 years or age. There is a slight predominance in men. In a sudden episode of cardiac death, the initial rhythm is most likely ventricular fibrillation, or ventricular tachycardia that degenerates into ventricular fibrillation. In a witnessed cardiac arrest, if the initial rhythm is amenable to defibrillation, a shock should be administered as soon as possible. Defibrillation is the best chance of recovering a perfusing rhythm from a nonperfusing rhythm. The longer a heart is in a nonperfusing rhythm, the less likely it is to respond to defibrillation and cardiac life support. Two minutes of chest compressions (D) is the standard cycle length for chest compressions. In the unwitnessed cardiac arrest, two minutes of chest compression should be initiated to "prime" the heart to receive defibrillation. However, in a witnessed arrest an immediate shock should take precedence. Administration of amiodarone (A) is a pharmacologic attempt to achieve a stable cardiac rhythm. It is indicated for persistent pulseless ventricular tachycardia or ventricular fibrillation after defibrillation and epinephrine have not resulted in a perfusing cardiac rhythm. Cardioversion (B) is indicated in an unstable (but not pulseless) cardiac rhythm (e.g. atrial fibrillation with rapid ventricular response) in an attempt to recover a normal sinus rhythm for the patient. Cardioversion does not play a role in the pulseless patient.

Which of the following is the most likely cause of phenylketonuria? A.Conversion of phenylalanine to tyrosine B.Conversion of tyrosine to phenylalanine C.Deficiency of phenylalanine hydroxylase D.Excess of phenylalanine hydroxylase

Correct Answer ( C ) Explanation: The majority of patients with phenylketonuria (PKU) are deficient in phenylalanine hydroxylase (PAH) due to an autosomal recessive disorder. Newborns with the disorder are asymptomatic until given phenylalanine, at which point symptoms develop. Patients with PKU who are undiagnosed and do not receive treatment develop intellectual disability and epilepsy. Elevated serum phenylalanine provides the diagnosis and treatment is dietary restriction of phenylalanine. The recommended uniform screening panel for newborns in the United States includes screening for PKU to allow for early intervention in patients with the disorder. Phenylalanine hydroxylase is a hepatic enzyme that catalyzes phenylalanine to tyrosine (A). The catabolic pathway for the disposal of dietary phenylalanine starts with conversion of phenylalanine to tyrosine, not conversion of tyrosine to phenylalanine (B). Excess phenylalanine hydroxylase (D) is not a factor in PKU; the disorder is caused by a deficiency of the enzyme.

A 29-year-old woman complains of mouth sores. On examination, you note white plaques covering the tongue and buccal mucosa. When scraped off using a tongue blade you note an erythematous base. Laboratory testing reveals a positive rapid HIV. Which of the following is the most likely diagnosis? A. Herpes simplex virus B. Kaposi sarcoma C. Oral candidiasis D. Oral hairy leukoplakia

Correct Answer ( C ) Explanation: The patient has oral candidiasis, also known as thrush. Candidiasis is a mucocutaneous fungal infection that affects up to 80% of AIDS patients. Characteristic white plaques cover the tongue and buccal mucosa, which can be easily scraped off using a tongue blade. Although similar in appearance, the plaques associated with oral hairy leukoplakia cannot be scraped off, distinguishing it from oral candidiasis. Candida can also affect the esophagus, resulting in painful swallowing and ultimately malnutrition. Oral candidiasis is associated with CD4 counts below 500 cells/mm3 and esophageal candidiasis with more advanced disease (CD4 counts below 100 cells/mm3). Oral candidiasis is managed with clotrimazole or nystatin suspension. Oral fluconazole and amphotericin B are reserved for refractory or recurrent disease. Herpes simplex virus infection (A), another cause of painful oral lesions, is characterized by vesicular lesions on the mucosal surfaces. Diagnosis is confirmed by identifying multinucleated giant cells in mucosal scrapings, PCR or culture. Kaposi sarcoma (B) is an AIDS-defining illness causing tumors on the skin, mouth, gastrointestinal tract, and respiratory tract. Oral Kaposi sarcoma appears as painless, well-circumscribed, raised, and violaceous lesions on the mucous membranes of the mouth. Biopsy is diagnostic. Oral hairy leukoplakia (D), caused by the Epstein-Barr virus (EBV), appears as white patches on the tongue, which cannot be scraped off.

A previously healthy 12-year-old girl presents with right ear pain for the past two days. On examination, there is erythema and edema of the external auditory canal with debris and crusted material visible in the external canal. She reports pain when the auricle is gently pressed. Which of the following pathogens is the most likely causative agent? A.Aspergillus B.Moraxella catarrhalis C.Pseudomonas aeruginosa D.Streptococcus pneumoniae

Correct Answer ( C ) Explanation: The patient has otitis externa. Otitis externa refers to infection and inflammation of the external ear. Factors which predispose to otitis externa include local trauma (often from scratching or attempted cerumen removal) and elevation of the local pH (frequent contact with water from swimming in pools or freshwater lakes). The most common organisms causing otitis externa are Pseudomonas aeruginosa, Staphylococcus aureus, Enterobacteriaceae, and Proteus species. Fungal species (like Aspergillus and Candida) cause otomycosis and are seen in immunosuppressed hosts, those with recent long-term antibiotic therapy, or in hot humid climates. The clinical presentation of otitis externa includes pruritus, pain, erythema, edema, and tenderness of the external auditory canal. Clear or purulent otorrhea may be present and crusting on the external canal is common. Infection may spread to the tragus, auricle, and periauricular soft tissues. Local lymph nodes may be inflamed. The treatment for otitis externa includes analgesia, cleansing of the external canal, and topical antimicrobials with or without steroids. Non-ototoxic topical antibiotics should be selected, particularly if swelling interferes with middle ear visualization and tympanic membrane rupture cannot be excluded. If edema of the external canal obstructs the lumen, an ear wick should be used to enhance distal delivery of the topical drops. Systemic antibiotics are not usually indicated, unless fever or periauricular extension is present. Aspergillus (A) and other fungal causes of otitis externa are usually seen in the immunocompromised or those with previous long-term antibiotic therapy. Moraxella catarrhalis (B) and Streptococcus pneumonia (D) are common causes of otitis media, not otitis externa.

A 23-year-old man is admitted for a femur fracture. An admission ECG is shown above. What management is indicated? A.Atropine B.Beta blocker C.Reassurance D.Transvenous pacing

Correct Answer ( C ) Explanation: The patient's ECG shows a 1st degree AV block; a normal variant finding in 2% of the population that requires no specific management. 1st degree AV block is defined as prolonged conduction of atrial impulses. The PR interval > 0.20 seconds (200 msec) and there is no loss of any atrial impulse. All impulses result in a ventricular response. Although it can be a normal variant, it can be seen in pathologic conditions like Lyme disease and should be followed up. However, no specific management is needed. Nodal depressing agents should be avoided or used with caution. Atropine (A) is required for patients with symptomatic bradycardia. Beta blockers (B) are AV nodal blockers and should be used with caution in patients with 1st degree AV blocks. Transvenous pacing (D) is required for higher degree AV blocks (2nd degree type II and 3rd degree heart block).

Which of the following statements is true regarding this condition? A.Associated with recent infection B.Most commonly occurs in boys ages four to eight years C.Pain can be referred to the knee D.Radiographs can be normal

Correct Answer ( C ) Explanation: The radiograph represents slipped capital femoral epiphysis (SCFE), a disorder seen in early childhood characterized by chronic slipping of the femoral epiphysis of the hip. SCFE is the most common cause of hip disability in adolescents. Clinically, the child develops hip pain referred to the thigh or knee. Internal rotation of the hip induces pain. It is more common in obese, African American males. Other risk factors include hypothyroidism, growth hormone deficiency, and renal osteodystrophy. Transient tenosynovitis (A) is a cause of pediatric limp typically preceded by a viral infection. Legg-Calve-Perthes disease (B) is an idiopathic avascular necrosis that most commonly occurs in boys aged four to eight years, whereas SCFE most commonly occurs in obese boys with an average age of 12 to 16 years. Radiographs are not normal (D) and will demonstrate epiphyseal slippage. In subtle cases, MRI may be needed for confirmation.

A 21-year-old woman presents with shortness of breath, rash and nausea after an insect bite. Her vitals are T 97.7°F, HR 128, BP 85/56, RR 28, oxygen saturation 93%. Exam reveals diffuse hives and posterior pharyngeal swelling. Which of the following should be immediately administered? A.Epinephrine 1:10,000, 0.3 mL IM B.Epinephrine 1:10,000, 10 mL IV C.Epinephrine 1:1000, 0.3 mL IM D.Epinephrine 1:1000, 0.3 mL IV

Correct Answer ( C ) Explanation: This patient presents with anaphylactic shock from an insect bite and requires immediate administration of epinephrine. Epinephrine is potentially life-saving in severe anaphylactic reactions. During an anaphylactic reaction, mast cells degranulate leading to release of histamines and other immune mediators. These mediators lead to the hallmark symptoms of allergic reactions including hives, nausea and vomiting, airway edema, bronchoconstriction and hypotension. Epinephrine acts immediately on adrenergic receptors to reverse these symptoms. Inhaled beta agonists can also be given to rapidly reverse bronchoconstriction. Many of the other treatments in anaphylaxis have a delayed onset of action. Diphenhydramine (H1 receptor blocker) acts by blocking the effect of histamine on H1 receptors mainly in the skin. Ranitidine, famotidine and other H2 blockers can mitigate some of the gastrointestinal symptoms caused by histamine as well as some of the cutaneous manifestations. Steroids mainly act by stabilizing mast cells from further degranulation but this effect is usually delayed 4-6 hours after administration. Steroids also increase the expression of beta-receptors in the lung increasing the efficacy of inhaled beta-agonists. Epinephrine should be given as 300 - 500 mcg IM in the anterolateral thigh. This dose equates to 0.3 - 0.5 mL of the 1:1000 preparation. Giving epinephrine 1:1000, 0.3 mL IV (D) is a massive overdose and can cause dysrhythmias and cardiac ischemia. Cardiac epinephrine (A & B) (1:10,000 concentration) comes in vials of 10 mL for a total of 1000 mcg of epinephrine. This preparation of epinephrine is reserved for use in patients with cardiac arrest (ventricular fibrillation, pulseless electrical activity or asystole) and should not be given to patients with a blood pressure. 0.3 mL of cardiac epinephrine IM is a 10-fold underdose as this amount only contains 30 mcg of epinephrine.

After return of spontaneous circulation from a ventricular fibrillation arrest, mortality can be determined by calculating a Cardiac Arrest Score, which takes into account which of the following factors? A.Left ventricular ejection fraction B.Pre-hospital pharmacologic treatment C.Systolic blood pressure D.Time to presentation to the emergency department

Correct Answer ( C ) Explanation: Witnessed out-of-hospital cardiac arrest patients can be stratified by using the 3-criteria Cardiac Arrest Score developed by Thompson and McCullough. The criteria are: (1) Systolic blood pressure in the emergency department; (2) time from loss of consciousness to return of spontaneous circulation; (3) neurologic responsiveness. Criteria are SBP > 90 mm Hg (1 point; < 90 mm Hg = 0 points), time from loss of consciousness to return of spontaneous circulation < 25 minutes (1 point; > 25 minutes = 0 points) and positive neurologic responsiveness (1 point; unresponsiveness = 0 points). A score of 0 equates to 90% in-hospital mortality and 3% chance of neurologic recovery. A score of 3 equates to 18% in-hospital mortality and an 89% chance of neurologic recovery. Left ventricular ejection fraction (A), pre-hospital pharmacologic treatment (B) and time to presentation to the emergency department (D) are not criteria in the Cardiac Arrest Score. However, they do play a role in considering the overall prognosis of cardiac arrest.

Which of the following is an example of primary prevention? A. Eliminating dust and pollen for an asthmatic B. Ophthalmoscope exam in diabetics C. Papanicolaou test D. Varicella vaccine

Correct Answer ( D ) Prevention traditionally has been divided into three categories: primary, secondary, and tertiary. Primary prevention targets individuals who may be at risk to develop a medical condition and intervenes to prevent the onset of that condition. Examples include childhood vaccinations, water fluoridation, smoking cessation, and education about safe sex. The varicella vaccine is an example of primary prevention. Secondary prevention includes procedures that detect and treat pre-clinical pathological changes and thereby control disease progression. Screening procedures, such as mammography to detect early stage breast cancer, are often the first step, leading to early interventions that are more cost effective than intervening once symptoms appear. Screening is usually undertaken by health professionals, such as routine blood pressure checks or via public health screening programs such as osteoporosis screening. Colonoscopy and Papanicolau test (C) are other common examples of secondary prevention. Tertiary prevention targets individuals with a known disease, with the goal of limiting or preventing future complications. It aids in softening the impact caused by the disease on the patient's function, longevity, and quality of life. Examples include screening diabetics for microalbuminuria, rigorous treatment of diabetes mellitus, and post-myocardial infarction prophylaxis with beta-blockers and aspirin. An annual ophthalmoscope examine in diabetic patients (B) to screen for retinopathy is an example of tertiary prevention. Tertiary prevention can include modifying risk factors, such as assisting a cardiac patient to lose weight, or making environmental modifications to reduce an asthmatic patient's exposure to allergens (A).

A 56-year-old woman presents with burning pain and tingling on the palmar surfaces of digits 1-3 of her right hand. The pain has been present for several months and awakens her at night. There is no atrophy or weakness of her hand. However, volar wrist percussion causes a shock-like pain extending to the palmar surfaces of digits 1-3. Which of the following tests proved positive in this examination? A.Finkelstein B.Phalen C.Spurling D.Tinel

Correct Answer ( D ) Explanation: A Tinel test is considered positive when percussion of the volar wrist in the region of the median nerve produces tingling or a shock-like pain in the palmar surfaces of digits 1-3. This test is generally sensitive for clinically diagnosing carpal tunnel syndrome, an entrapment neuropathy due to some form of compression or narrowing of the carpal tunnel that impinges the median nerve. Other physical exam findings may include a positive Phalen sign in which flexion of both wrists to 90 degrees for one minute causes pain or tingling in the median nerve distribution. The carpal compression test, in which the clinician applies steady, direct pressure over the carpal tunnel to elicit tingling, may be even more sensitive for the condition. Carpal tunnel syndrome generally presents with pain and tingling in the median nerve distribution and may initially bother the patient only during sleep. A history of performing repetitive activities may be noted. Diabetes mellitus and fluid retention during pregnancy can also contribute to carpal tunnel compression. A work-up for carpal tunnel syndrome may include an ultrasound to observe flattening of the median nerve, or electromyography and nerve conduction studies to differentiate it from a more proximal neuropathy. A trial of wrist extension splinting, as well as NSAIDs and corticosteroids, may help. Surgical carpal tunnel release can be considered for symptoms beyond 12 months of conservative treatment. A Finkelstein test (A) requires sharp, ulnar deviation of the thumb while the fingers are flexed around the thumb into a fist. Pain at the wrist is suggestive of deQuervain tenosynovitis. A Phalen test (B) is positive when flexion of both wrists to 90 degrees for one minute causes pain or tingling in the median nerve distribution. Like A Tinel test, a positive Phalen test suggests carpal tunnel syndrome. A Spurling test (C) requires the patient to rotate and extend the neck to one side while the clinician gently applies an axial load to the neck. Nerve root compression is likely when this test produces cervical radiculopathy.

Which of the following is a complication of an untreated mallet finger injury? A. Boutonniere deformity B. Dupuytren's contracture C. Jersey finger D. Swan-neck deformity

Correct Answer ( D ) Explanation: A mallet finger is caused by a forced flexion of the distal interphalangeal joint (DIP) leading to rupture of the extensor tendon at its insertion at the base of the distal phalanx or bony avulsion of the tendon insertion site. Appropriate treatment includes splinting the distal interphalangeal joint in full extension or slight hyperextension while allowing the proximal interphalangeal joint to have full range of motion. In untreated or under treated cases, a swan neck deformity occurs. Other causes include rheumatoid arthritis and connective tissue disorders (Ehlers-Danlos syndrome). A swan neck deformity is characterized by hyperextension of the proximal interphalangeal joint (PIP) and flexion of the distal interphalangeal joint (DIP). Dupuytren's contracture (B) refers to a thickened and fibrous palmar fascia which is due to fibrous proliferation. It occurs commonly in men over age 40 years, but it also is common in those of Northern European descent, diabetics, alcoholics and epileptics. Boutonniere deformity (A) occurs from forced flexion at the proximal interphalangeal joint (PIP), causing a tear of the central portion of the extensor tendon at the proximal interphalangeal joint (PIP). Patients are unable to fully extend at the proximal interphalangeal joint (PIP) with the wrist and metacarpalphalangeal joints fully extended. Jersey finger (C) occurs in cases such as a football player grabs another player's jersey, avulsing the profundus tendon from its bony insertion.

An 11-month-old previously healthy boy presents in January with a two-day history of rhinorrhea, cough, and wheezing. Physical exam is significant for bilateral wheezing and crackles heard at the lung bases. The patient's pulse oximetry is 96 percent on room air. Which of the following is the most appropriate next step in management? A.Administer racemic epinephrine B.Albuterol nebulizer treatment every four hours C.Chest X-ray D.Observation and supportive care

Correct Answer ( D ) Explanation: Bronchiolitis is a lower respiratory tract viral infection that is caused by damage of the epithelial cells lining the small airways, which leads to acute inflammation, increased mucous production, and bronchospasm. Bronchiolitis peaks between December and March. The clinical presentation includes rhinorrhea, cough, wheezing, low-grade fever, and shortness of breath. Bilateral wheezing and crackles are heard on pulmonary exam. Poor feeding and respiratory distress (tachypnea, nasal flaring, and hypoxemia) are indicators of increased severity. Oxygen saturation less than 95 percent, PO2 less than 65, PCO2 greater than 40, atelectasis on chest X-ray, and respiratory rate greater than 70 are also all indicators of severe disease. Bronchiolitis most commonly occurs between two and seven months of age, but can occur in those up to two years old. The most common virus associated with bronchiolitis is respiratory syncytial virus (RSV). Other causes include parainfluenza virus, metapneumovirus, and adenovirus. Symptoms can last up to five days and generally worsen by day three before subsiding. Most patients can undergo observation and supportive care. Treatment is symptomatic and includes cool mist. Hospitalization is determined by the patient's comorbidities, reliability of parents, duration of symptoms, and condition on presentation (hypoxia, tachypnea, dehydration, age). Racemic epinephrine (A) is the treatment for laryngotracheobronchitis (croup) caused by parainfluenza virus and is characterized by a barking cough and inspiratory stridor. Bronchodilators (B), such as albuterol, are not routinely given when managing bronchiolitis. Bronchiolitis is usually a clinical diagnosis, and a chest X-ray (C) is not required to make the diagnosis. Chest X-ray findings typically include hyperinflation and patchy infiltrates. Focal findings are not consistent with bronchiolitis.

Which of the following statements is true regarding febrile seizures? A.Administering acetaminophen and ibuprofen during a febrile illness have been shown to decrease the likelihood of seizure recurrence B.Children who have had a simple febrile seizure have the same rate of epilepsy as those who have not had a febrile seizure C.Older children with a febrile seizure are more likely to have a recurrence than younger children with a febrile seizure are D.Treatment with long-term anticonvulsants does not lower the long-term risk of developing epilepsy

Correct Answer ( D ) Explanation: Children with simple febrile seizures have a 2% to 3% chance of developing epilepsy, compared with a 1% rate of epilepsy in the general population. Children with complex febrile seizures have a significantly higher risk. Treatment with long-term anticonvulsants does not affect the long-term risk of developing epilepsy and is rarely warranted. Administering acetaminophen and ibuprofen (A) have not been shown to decrease the likelihood of seizure recurrence. About 30% of children with a simple febrile seizure have a recurrence; of these, one-half will have a third event. Children with simple febrile seizures have a 2% to 3% chance of developing epilepsy (B), compared with a 1% rate of epilepsy in the general population. The younger the age at onset of a febrile seizure, the more likelihood of recurrence (C).

Which of the following is most closely associated with the development of acute cor pulmonale? A.Acute bronchitis B.Health care associated pneumonia C.Heart failure D.Pulmonary embolism

Correct Answer ( D ) Explanation: Cor pulmonale is defined as an alteration in the structure and function of the right ventricle caused by a primary disorder of the respiratory system. Pulmonary hypertension is the common link between lung dysfunction and the heart in cor pulmonale. Cor pulmonale is usually a chronic condition, but acute pulmonary embolism (more common) and acute respiratory distress syndrome are associated with acute cor pulmonale. The underlying pathophysiology in massive pulmonary embolism causing cor pulmonale is the sudden increase in pulmonary resistance. In chronic cor pulmonale, RV hypertrophy (RVH) generally predominates. In acute cor pulmonale, right ventricular dilation mainly occurs. Acute bronchitis (A) is not associated with cor pulmonale. Heart failure (C) results from an abnormality in systolic or diastolic cardiac function. Chronically increased left heart pressures in poorly controlled heart failure can lead to cor pulmonale, but in most patients, this does not occur. Pneumonia (B) can increase cardiac demands but generally does not lead to pulmonary hypertension or right ventricular dilation.

A patient with sarcoidosis presents with progressive peripheral edema. Echocardiography reveals restrictive cardiomyopathy. In addition to diuresis and heart rate control, which of the following is also appropriate? A.Buspirone B.Melphalan C.Phlebotomy D.Prednisone

Correct Answer ( D ) Explanation: Infiltrative myocardial deposition usually results in restrictive cardiomyopathy (RCM). Evaluation begins with the typical heart failure work-up of chest radiograph, ECG, echocardiogram, cardiac MRI and possibly cardiac catheterization. A chest radiograph may show enlarged atria and normal ventricles. ECG may reveal low voltages and tachyarrhythmias. Echocardiographic findings typical of RCM include bilateral atrial enlargement, normal or thickened ventricular walls, decreased right > left ventricular walls, mural thrombi and diastolic dysfunction. A cardiac MRI may show myocardial inflammation or infiltrates. Endocardial biopsy may be necessary to confirm a diagnosis. Treatment of RCM is difficult due to the relative refractoriness of many of the underlying pathologies. Specific augmentative treatments are aimed at the underlying disorder. One example is corticosteroids, like prednisone or prednisolone, used to control sarcoidosis and Loeffler's endocarditis. Gentle diuresis may be helpful. Anticoagulation is usually considered. Serotonin agonists are contraindicated in patients with restrictive cardiomyopathy, as these drugs, like buspirone (A), cisapride and sumatriptan, can cause decreased compliance of the endomyocardium, and actually lead to restrictive cardiomyopathy. Chemotherapy, like melphalan (B) and cyclophosphamide, is used to decrease the amount of abnormal cells in amyloid restrictive cardiomyopathy. Phlebotomy (C) and chelation therapy are used to control the complications of hemochromatosis.

A 45-year-old man presents after swallowing a pen. A review of his previous visits shows two prior evaluations for the same thing. He reports this is because of post-traumatic stress disorder. An X-ray confirms the presence of a pen in the esophagus. What is his most likely diagnosis? A.Borderline personality disorder B.Drug-seeking behavior C.Malingering D.Munchausen syndrome

Correct Answer ( D ) Explanation: Munchausen syndrome is a form of factitious disorder in which patients feign illness for some sort of personal gain. Factitious disorders are different from somatic disorders and malingering because of intention. Patients will often have objective physical findings as a result of their self-harm. In some cases, they may induce a serious illness in order to have prolonged hospitalization in the patient role. They are willing to undergo procedures in order to maintain their sick role. Borderline personality disorder (A) is characterized by a pattern of instability in interpersonal relationships and self-image. Patients have extremely labile moods and are at risk for suicide, eating disorders, post-traumatic stress disorder and substance abuse. Patients classically split providers identifying some as good and some as bad subsequently pinning them against each other in the course of their care. Drug-seeking behavior (B) is a challenging diagnosis in which a patient feigns illness as a type of malingering in order to obtain drugs. Typically the patient has active issues with substance abuse. Malingering (C) is separate from somatization disorders and factitious disorders because the patient simulates a disease for some secondary gain. Factitious patients will do things creating actual objective findings.

A 60-year-old man with a history of hypertension and paroxysmal atrial fibrillation is scheduled to undergo an elective dental extraction. His paroxysmal atrial fibrillation is rate controlled with a beta blocker and he is on chronic anticoagulation with warfarin. Anticoagulation must be discontinued for the procedure. His medications include metoprolol tartrate, lisinopril and warfarin. In addition to discontinuing warfarin, which of the following is the most appropriate next step in management? A. Bridge with aspirin B. Bridge with intravenous unfractionated heparin C. Bridge with low molecular weight heparin D. No bridging agent is necessary

Correct Answer ( D ) Explanation: No bridging agent is necessary. Interruption of oral anticoagulation in patients with atrial fibrillation is sometimes necessary for invasive procedures, and it must be determined whether bridging is indicated. If the patient has a low short-term risk as determined by a CHA2DS2-VASc score of 0-1 and the duration of interruption is less than 1 week, then bridging is not needed. This patient has a CHA2DS2-VASc score of 1 due to his history of hypertension; therefore, no periprocedural bridging is necessary. In this scenario it would be recommended to simply discontinue warfarin approximately 5 days before the procedure with no bridging agent. Warfarin can often be resumed 12-24 hours after the procedure if there is no active bleeding. If the patient has a high short-term risk with a CHA2DS2-VASc score of > 2, recent stroke, mechanical or rheumatic mitral valve or if the interruption is more than 1 week, then use of a bridging agent should be considered more strongly. In this setting, it would be appropriate to bridge with intravenous unfractionated heparin (B) or low molecular weight heparin (C). For atrial fibrillation patients with moderate risk features with a CHADS2 score of 3-4, a history of remote transient ischemic attack or stroke, or a mechanical aortic valve, management is individualized according to risks and benefits and is patient and provider dependent. It would not be appropriate to bridge with aspirin (A) as aspirin has no role in bridging.

What is the most common cause of upper gastrointestinal bleeding? A. Aortoenteric fistula B. Boerhaave's syndrome C. Esophogeal varices D. Peptic ulcer disease

Correct Answer ( D ) Explanation: Peptic ulcer disease (PUD) is responsible for almost half of all UGIB. PUD involves both gastric and duodenal ulcers. H. pylori bacteria is thought to be the cause of 70-80% of duodenal ulcers and 60-70% of gastric ulcers. The bacteria causes inflammation of the stomach and duodenal mucosa and disrupt natural defense mechanisms leading to ulceration. NSAIDs are the second leading cause of PUD. Patients typically present with epigastric pain that is burning or gnawing in nature. Pain may also be described in the chest and back. The pain usually occurs a couple of hours after meals and at night and is typically improved or relieved by eating. Aortoenteric fisula (A) is an uncommon cause of UGIB in patients without a history of a AAA repair. Boerhaave's syndrome (B) may be seen after recurrent vomiting and patients are typically toxic. Esophogeal varices (C) are common cause of UGIB in patients with a history of liver cirrhosis.

A 61-year-old man presents to the clinic for urinary urgency and frequency, nocturia, and dribbling urine consistently after he voids. He endorses a weak stream and a feeling that his bladder won't empty fully. You plan to start treatment for benign prostatic hyperplasia if his physical exam confirms it. Which of the following physical exam findings is most indicative of benign prostatic hyperplasia? A.A warm, boggy, prostate gland that is tender on palpation B.Asymmetric enlargement of the prostate gland with unilateral induration C.Normal size and texture of the prostate gland D.Smooth, firm, elastic enlargement of the prostate gland

Correct Answer ( D ) Explanation: Smooth, firm enlargement of the prostate gland is characteristic of benign prostatic hyperplasia (BPH). BPH is the most common benign tumor in men, with the incidence estimated at 50% of men by 60 years. Between ages 55-75, the likelihood of developing obstructive voiding symptoms rises significantly. Symptoms include hesitancy and straining to urinate, decreased force and caliber of the urine stream, and the need to urinate twice within 2 hours due to incomplete emptying. Post void dribbling can occur due to the residual urine in the bladder after voiding. Men may also develop irritative bladder symptoms including urgency, frequency, and nocturia. Though prostate size does not correlate with symptoms, a rectal exam will usually show some degree of smooth, firm enlargement. Induration should not be present, as this would be more concerning for malignancy. A distended bladder may be palpated. The decision to get a serum prostate-specific antigen level is controversial, as it is difficult to use this value to determine whether a patient has BPH as opposed to cancer. The American Urological Association symptoms index should be used to gauge severity of BPH and guide treatment. Recurrent urinary tract infections, gross hematuria, bladder stones, or kidney disease merit surgical resection and require referral to urology. Otherwise, mild symptoms can be managed with observation. Moderate to severe symptoms merit medical therapy, for which many options exist. Alpha-blockers will reduce symptoms by improving urine flow at the bladder neck. The 5-alpha reductase inhibitors, interrupts the processing of testosterone and can actually decrease the size of the prostate and improve symptoms. Six months are required to see improvement. Phosphodiesterase-5 inhibitors, like tadalafil, can offer improvement as well. Combination therapy is usually beneficial and tolerated. Asymmetric enlargement of the prostate gland with unilateral induration (B) is concerning for prostate cancer. This finding should alert the clinician to the need for further work-up, including a serum prostate-specific antigen, transrectal ultrasound, and biopsy. A warm, boggy prostate gland that is tender on palpation (A) is a typical finding in acute or chronic bacterial prostatitis. In the acute setting, this requires a urine culture with antibiotics guided by culture findings. A normal size and texture of the prostate gland (C) may occasionally be present in patients with underlying BPH. However, a normal prostate exam should alert the clinician to explore other causes of the obstructive and irritative voiding symptoms, such as urinary tract infections, urethral strictures, or neurogenic bladder.

A 52-year-old man with a history of hypertension presents with a concern that he's "developing breasts." On exam, glandular enlargement beneath his areola is present bilaterally in a symmetric manner. It is mildly tender. Which of the following daily medications most likely contributed to his condition? A. Clonidine B. Hydrochlorothiazide C. Lisinopril D. Spironolactone

Correct Answer ( D ) Explanation: Spironolactone is well-established as a contributor to the development of gynecomastia due to its effect on androgen expression. Many medications are implemented in causing gynecomastia, which common offenders being antipsychotics, antifungal medications, high active antiretroviral therapy, and some atypical antidepressants. Fortunately gynecomastia caused by medication use usually resolves with discontinuation of the causative medication. If a medication is not found to be responsible for gynecomastia, other causes should be pursued. Serum beta-hCG levels should be drawn to look for a testicular tumor or other malignancy that may be producing exogenous hormones. Similarly, a prolactin level should be checked to rule out a prolactinoma. Additionally, any male athlete should be asked about taking anabolic steroids as using these will lead to gynecomastia in 50% of users. Other causes to explore based on individual patient profiles may include liver disease, obesity, and primary or secondary hypogonadism. Of note, in teenagers a mild degree of pubertal gynecomastia can be normal and will usually resolve without treatment in 1-2 years. However, any male breast tissue that is asymmetric, unusually firm, forming in a location other than below the areola, or accompanied by nipple bleeding, discharge, or retraction should raise concern for a malignancy. In these patients, a needle biopsy with cytologic examination will help differentiate benign gynecomastia from a tumor or other inflammatory process. When necessary, true glandular gynecomastia usually responds to selective estrogen receptor modulators, such as raloxifene or tamoxifen. Surgical correction may be needed in patients for whom medication therapies are ineffective. Clonidine (A), hydrochlorothiazide (B), and lisinopril (C) are incorrect choices as these are not particularly well-known for causing gynecomastia in patients.

Which of the following can cause an exudative pleural effusion? A. Cirrhosis B. Congestive heart failure C. Nephrotic syndrome D. Systemic lupus erythematosus

Correct Answer ( D ) Explanation: Systemic lupus erythematosus can cause an exudative pleural effusion. A pleural effusion is an accumulation of fluid in the pleural space and can either be exudative or transudative. Exudates result primarily from pleural and lung inflammation which result in increased capillary and pleural membrane permeability. It can also be caused by impaired lymphatic drainage of the pleural space. Disease in virtually any organ can cause exudative pleural effusions by a variety of mechanisms, including infection, malignancy, immunologic responses, lymphatic abnormalities, noninfectious inflammation, iatrogenic causes, and movement of fluid from below the diaphragm. Some examples include pneumonia, tuberculosis, pancreatitis, connective tissue disorders such as lupus or rheumatoid arthritis, and malignancy, more commonly breast, lung, lymphoma or leukemia. Transudative pleural effusions result from imbalances in hydrostatic and oncotic pressures in the chest. Congestive heart failure (B) accounts for up to 90% of all transudative pleural effusions. Nephrotic syndrome (C) and cirrhosis (A) are also causes of transudative pleural effusions.

Which of the following is most characteristic of acute conjunctivitis? A.Ciliary injection B.Diminished visual acuity C..Pain and Photophobia D.Serous or clear discharge

Correct Answer ( D ) Explanation: The most common cause of a red eye is acute conjunctivitis, most commonly due to viral infection. Patients often complain of an itchy, irritated eye with serous or clear discharge. Purulent (creamy white or yellow watery) discharge suggests a bacterial cause. Scanty, white, stringy exudate occurs most often with allergic conjunctivitis. It is important to examine both eyes because many patients with conjunctivitis in one eye have signs of early conjunctivitis in the other. Conjunctival injection is characterized by individually visible vessels in the conjunctiva branching from the sclera toward the cornea. Inspect the palpebral conjunctiva carefully with magnification to determine whether lymphoid hyperplasia (cobblestone appearance) exists. The type and quantity of discharge are assessed by pulling down the lower lid. The absence of discharge should prompt investigation to other causes of a red eye. Ciliary injection (A) appears as engorgement of the deep vessels around the limbus. The significance of ciliary injection is that the deep ciliary vessels are involved, indicating a much more serious inflammatory condition of the eye. Diminished visual acuity (B) is not associated with acute conjunctivitis. If present, other serious causes of red eye should be investigated. Pain and photophobia (C) is associated with glaucoma and keratitis.

A 24-year-old woman with no past medical history presents with left wrist pain after a fall. The left extremity is grossly deformed and the patient complains of severe pain. The patient has a blood pressure of 183/100 mm Hg. While awaiting X-rays, what management is indicated for the patients elevated blood pressure? A. Arrange admission for blood pressure control B. Start an oral beta-blocker and monitor for response C. Start intravenous beta-blocker and admit to the intensive care unit D. Treat the patient's pain and reassess the blood pressure

Correct Answer ( D ) Explanation: The patient presents with a markedly elevated blood pressure in the setting of pain from a trauma and should have pain control initiated and her blood pressure rechecked. Hypertension is defined as a persistent SBP >140 mm Hg or DBP >90 mm Hg. Pain and anxiety are common causes of elevated blood pressure and heart rate in the outpatient setting. Historically, patients with elevated blood pressure and nonspecific symptoms were referred to as hypertensive urgency but this term has fallen out of use. In a patient presenting with elevated blood pressure who does not have signs or symptoms of end-organ damage, the clinician's focus should be on identifying external reasons for the elevated pressure and treating or addressing these. In this case, the reduction or relief of pain will likely lead to decreased blood pressure. (A) Patients with elevated blood pressure and an absence of end-organ damage (e.g. acute coronary syndrome, aortic dissection, encephalopathy, change in renal function) do not require admission for management. A primary care physician in the outpatient setting best manages these patients. (B) Starting a beta-blocker will not be beneficial in a patient with acute pain as the cause of elevated blood pressure. (C) Similarly, administration of an intravenous beta-blocker and admission to the intensive care unit is not indicated as the patient exhibits no end-organ damage.

A 13-year-old boy with no past medical history presents with a headache 3 days after a closed head injury. The patient states that he stood up from kneeling and hit the top of his head on a wood cabinet. There was no loss of consciousness or seizure activity. In addition to the headache, he complains of difficulty concentrating at school and dizziness. His physical examination is unremarkable. What management is indicated? A.CT scan of the head with contrast B.CT scan of the head without contrast C.MRI of the brain D.Neurology referral

Correct Answer ( D ) Explanation: The patient presents with minor head trauma and complaints consistent with a concussion and should have neurology follow up arranged. A concussion is a minor traumatic brain injury (TBI) that is often seen in MVCs and collision sports (football, hockey). It is typically caused by a rotational injury or an acceleration-deceleration injury. Patients will present with a number of non-specific symptoms including headaches, dizziness, confusion, amnesia, difficulty concentrating, and blurry vision but do not have focal neurologic findings. Despite the absence of severe intracranial injury, patients can have chronic and debilitating symptoms from concussions. Neurology referral is recommended, as patients should have functional testing and tracking of their symptoms for resolution. It is vital to council patients to avoid contact sports or activities that increased the risk of recurrent injury as these patients are at risk for more severe injury with second impact. In the absence of focal neurologic findings, absence of antiplatelet or anticoagulant use and minor trauma, imaging is not needed (A, B, C).

A 16-year-old girl presents to the ED complaining that she cannot walk up the steps. She has been well recently except for an episode of gastroenteritis two weeks ago. On exam, she has decreased strength in her bilateral lower extremities with absent patellar and ankle jerk reflexes. What is the most important next test to perform? A.Electromyelography B.Lumbar puncture C.MRI lumbar spine D.Vital capacity

Correct Answer ( D ) Explanation: This patient has Guillain-Barré syndrome (GBS), an immune-mediated peripheral neuropathy caused by myelin sheath destruction. It is often preceded by a viral illness, infection with Campylobacter jejuni, or vaccination. In the classic presentation of GBS, a viral illness is followed by an ascending symmetric weakness or paralysis with decreased or absent deep tendon reflexes. The most serious complication of GBS is respiratory failure from diaphragmatic weakness. Measuring the vital capacity or negative inspiratory force (NIF) is critical to assessing respiratory effort. These easily repeated tests predict developing diaphragmatic weakness and the need for prophylactic intubation. GBS is treated with supportive care, IV immune globulin, and possibly plasmapheresis. All patients with suspected GBS should be admitted to an intensive care unit. Electromyelography (A) (EMG) measures the action potential of a nerve and its muscle unit and provides information on the innervation status of a muscle, less likely to be abnormal in acute demyelination without axonal loss. EMG readings will show slowed action potentials in GBS because of demyelination. Although it is useful to distinguish a neuropathic etiology of weakness, it is not diagnostic of GBS, nor does it predict potential complications such as respiratory failure. Lumbar puncture (B) is sometimes performed to augment the diagnosis of GBS, but is not the test of choice; it has no predictive value. CSF will often show few WBCs, with a monocytic predominance and increased protein. Although MRI (C) of the lumbar spine may be performed to rule out a cord-compressing, space-occupying lesion that could be causing bilateral weakness, it provides no other specific information and will be negative in cases of GBS.

A 73-year-old woman with a history of hypertension presents with a unilateral headache for 3 weeks. She states that she has a throbbing pain at her right temple and has pain in her jaw with opening and closing. The vision in her right eye has worsened over the previous day. Her blood pressure is 173/100. What management is indicated? A.Carbamazepine B.Labetalol C.Non-contrast head CT scan D.Prednisone

Correct Answer ( D ) Explanation: This patient presents with a unilateral, subacute headache with associated jaw claudication and vision change; symptoms consistent with temporal arteritis. Temporal arteritis or giant cell arteritis is a systemic inflammatory process of small and medium-size arteries. The most commonly involved vessels are the ophthalmic vessels and the extracranial branches of the aortic arch. The disease typically affects patients over 70 years of age and is more common in women than in men. Patients present with a subacute headache that is throbbing in nature and may be present for weeks to months. Often, patients will have symptoms for more than 2 months. Patients may also report jaw claudication secondary to vascular insufficiency of the masseter and temporalis muscles. Physical examination may reveal tenderness over the temporal artery. Systemic symptoms may also be present including fever, joint pains, and weight loss. Diagnostic testing in the Emergency Department generally begins with an erythrocyte sedimentation rate (ESR) with a cutoff of 50 mm/hour although the level may be >100 mm/hour. However, the ESR will be normal in 10-25% of patients. The gold standard diagnostic test is a temporal artery biopsy. In patients with a high-clinical likelihood of temporal arteritis, treatment should be initiated regardless of initial diagnostic testing as delay can lead to permanent visual loss. Prednisone should be started at 60 - 120 mg/day. Carbamazepine (A) is the treatment of choice for trigeminal neuralgia, not temporal arteritis. The patient does not present with symptoms consistent with hypertensive emergency requiring emergent antihypertensive treatment with labetalol (B). A non-contrast head CT scan (C) is not helpful in temporal arteritis as the disease does not involve the intracranial contents.

A 55-year-old man presents with sudden onset of decreased vision and pain in his right eye, He reports vomiting twice prior to presentation. Examination reveals normal extraocular motions and a mid position pupil that does not react to light. What management should be pursued? A.Acetazolamide drops B.Metoprolol intravenous C.Tetracaine drops D.Timolol drops

Correct Answer ( D ) Explanation: This patient presents with signs and symptoms concerning for acute angle closure glaucoma and should be immediately treated with a topical beta blocker like timolol. Glaucoma represents an ocular neuropathy caused by increased intraocular pressure. Glaucoma occurs either as the result of increased aqueous humor production (open-angle) or due to decreased drainage of the aqueous humor (closed-angle). Open-angle glaucoma is slower in onset and progressively causes changes in vision. Primary angle closure glaucoma, on the other hand, occurs suddenly and can lead to rapid vision loss. It is typically precipitated by papillary dilation (i.e. going into a dimly lit or dark room). Symptoms of angle closure glaucoma include eye pain, headache, nausea, vomiting and change in vision. The cornea may appear cloudy and the pupil midposition and dilated. The diagnosis can be definitively made by finding an increased intraocular pressure on tonometry. Treatment should be initiated promptly with topical timolol and pilocarpine (a miotic agent). In severe cases, intravenous acetazolamide should be added. ​ Acetazolamide (A) is effective if given intravenously in severe cases. Intravenous beta blockers (B) do not play a role in management. Tetracaine (C) is a topical anesthetic and does not play a role in glaucoma management.

Which of the following murmurs is associated with an increase in right atrial pressure? A.Aortic regurgitation B.Mitral regurgitation C.Mitral stenosis D.Tricuspid regurgitation

Correct Answer ( D ) Explanation: Valve disorders are often characterized in terms of stenosis (incomplete opening of the valve, thereby increasing the resistance in blood flow) or regurgitation (an incomplete closure of a valve, resulting in a backflow of blood). The tricuspid valve is 1 of 2 atrioventricular (AV) valves and lies between the right atrium and right ventricle. In tricuspid regurgitation, there is incomplete forward flow from the right atrium into the right ventricle and blood flows from the right ventricle into the right atrium during systole. This backflow of blood results in increased right atrial pressure. Tricuspid regurgitation is caused by right ventricular dilation secondary to pulmonary hypertension, rheumatic heart disease, and infective endocarditis. Patients may complain of fatigue and dyspnea on exertion. On exam, there may be a holosystolic murmur best heard at the xiphoid area adjacent to the left sternal border. The aortic valve is one of 2 semilunar valves (the other being the pulmonary valve) and is positioned between the left ventricle and aortic trunk. Aortic regurgitation (A) results in a backflow of blood and an increase in pressure of left ventricle during diastole. The mitral valve is the 2nd AV valve and is situated between the left atria and left ventricle. Mitral regurgitation (B) causes a backflow into the left atria during systole, increasing left atrial pressure. In mitral stenosis (C), the mitral valve cannot fully open during diastole, resulting in an increase in resistance in blood flow through the valve and increasing the left atrial pressure while decreasing the filling of the left ventricle.

A 60-year-old man presents with the ECG seen above. His blood pressure is 80/60 mm Hg. Which of the following is the most likely diagnosis? A.Atrial flutter B.Atrioventricular reentry tachycardia C.Supraventricular tachycardia D.Ventricular tachycardia

Correct Answer ( D ) Explanation: Ventricular tachycardia (VT) is present when there are >3 consecutive premature ventricular contractions occurring at a rate > 100. P waves are usually absent and the QRS complexes are wide (> 120 msec) and may be bizarre appearing. Ventricular tachycardia is classified as "monomorphic" (QRS complexes look the same) or "polymorphic" (QRS complexes have varying morphology). Sometimes it is difficult to distinguish ventricular tachycardia from supraventricular tachycardia (SVT) with aberrancy (presence of a bundle branch block). Nonetheless, any patient with a wide complex tachycardia who is hemodynamically unstable should undergo immediate electrical cardioversion. (A) Atrioventricular reentry tachycardia is a type of SVT. It is more commonly associated with regular pulse and monomorphic narrow QRS complexes. (B) Atrial flutter is more commonly associated with regular tachycardia and sawtooth appearance of P waves. It is sometimes difficult to distinguish ventricular tachycardia from supraventricular tachycardia with aberrancy. Any patient with a wide complex tachycardia who is hemodynamically unstable should be treated as having ventricular tachycardia. (C) Some electrocardiographic characteristics that support VT over SVT are extreme left axis deviation, QRS concordance (all QRS complexes from V1 to V6 are either all positive or all negative), and fusion or capture beats, which indicates AV dissociation.

A 59-year-old man has been experiencing left sided hearing loss over the past several months and he is no longer able to listen to a telephone on his left side. He also has left side tinnitus and feels off-balanced when walking. An audiogram shows high tone hearing loss in the left ear and a 30% word discrimination score. Which of the following is the most likely diagnosis? A.Labyrinthitis B.Meniere's disease C.Presbycusis D.Vestibular schwannoma

Correct Answer ( D ) Explanation: Vestibular Schwannoma usually presents with a triad of progressive ipsilateral hearing loss, ataxia, and tinnitus. It is a tumor of the vestibular portion of the acoustic (VIII) cranial nerve and has historically been referred to as an acoustic neuroma (although this term is a misnomer). A typical finding is ipsilateral sensorineural hearing loss in the high tones and a poor word discrimination score (hence the frequent complaint of not being able to use a telephone in that ear). Diagnosis is made with neuroimaging (contrast enhanced MRI is most common) and treatment options include: 1) continued observation with periodic imaging 2) radiosurgery 3) surgical resection. Meniere's disease (B) is a disorder of endolymphatic hydrops. It presents with a similar triad of hearing loss, tinnitus, and vertigo. Some key distinctions include: 1) recurrent bouts of severe and often disabling vertigo (as opposed to ataxia) and 2) hearing loss will come and go and is typically in the low tones (as opposed to the high tones). Labyrinthitis (A) is inflammation of the inner ear usually caused by a viral infection and often follows an upper respiratory infection. The vertigo and hearing loss have an acute onset and are self-limited to a few weeks. Treatment is symptomatic in most cases. Presbycusis (C) is age-related hearing loss that typically begins past the age of 65 and is almost always bilateral affecting both ears equally.


Ensembles d'études connexes

biology unit 2: biological molecules

View Set

Basic Overview of GSM Communications

View Set

9.) Introduction to Body Cavities

View Set

exam 4 fundamentals practice questions

View Set